eureka 2011

148
CONTEÚDO XVI OLIMPÍADA DE MAIO Enunciados e resultado brasileiro 2 XXI OLIMPÍADA DE MATEMÁTICA DO CONE SUL Enunciados e resultado brasileiro 5 LI OLIMPÍADA INTERNACIONAL DE MATEMÁTICA (IMO) Enunciados e resultado brasileiro 7 XXV OLIMPÍADA IBEROAMERICANA DE MATEMÁTICA Enunciados e resultado brasileiro 9 ARTIGOS ASSOCIANDO UM POLINÔMIO A EXPRESSÕES ALGÉBRICAS E TRIGONOMÉTRICAS Marcílio Miranda 11 SOMAS TRIGONOMÉTRICAS: DE PROSTAFÉRESE A FÓRMULA DE EULER Rogério Possi Junior 18 UMA INTERESSANTE DEDUÇÃO PARA A FÓRMULA DE HERÃO Flávio Antonio Alves 31 RAÍZES DA UNIDADE Anderson Torres & Eduardo Tengan 33 COMO É QUE FAZ 42 SOLUÇÕES DE PROBLEMAS PROPOSTOS 45 PROBLEMAS PROPOSTOS 59 AGENDA OLÍMPICA 61 COORDENADORES REGIONAIS 62

Upload: demi-de

Post on 01-Mar-2016

275 views

Category:

Documents


0 download

DESCRIPTION

ASSOCIANDO UM POLINÔMIO A EXPRESSÕES ALGÉBRICAS E TRIGONOMÉTRICAS Marcílio Miranda 11 XVI OLIMPÍADA DE MAIO Enunciados e resultado brasileiro 2 SOLUÇÕES DE PROBLEMAS PROPOSTOS 45 XXI OLIMPÍADA DE MATEMÁTICA DO CONE SUL Enunciados e resultado brasileiro 5 UMA INTERESSANTE DEDUÇÃO PARA A FÓRMULA DE HERÃO Flávio Antonio Alves 31 XXV OLIMPÍADA IBEROAMERICANA DE MATEMÁTICA Enunciados e resultado brasileiro 9 CONTEÚDO ARTIGOS

TRANSCRIPT

Page 1: Eureka 2011

CONTEÚDO XVI OLIMPÍADA DE MAIO Enunciados e resultado brasileiro

2

XXI OLIMPÍADA DE MATEMÁTICA DO CONE SUL Enunciados e resultado brasileiro

5 LI OLIMPÍADA INTERNACIONAL DE MATEMÁTICA (IMO) Enunciados e resultado brasileiro

7 XXV OLIMPÍADA IBEROAMERICANA DE MATEMÁTICA Enunciados e resultado brasileiro

9 ARTIGOS

ASSOCIANDO UM POLINÔMIO A EXPRESSÕES ALGÉBRICAS E TRIGONOMÉTRICAS Marcílio Miranda

11 SOMAS TRIGONOMÉTRICAS: DE PROSTAFÉRESE A FÓRMULA DE EULER Rogério Possi Junior

18 UMA INTERESSANTE DEDUÇÃO PARA A FÓRMULA DE HERÃO Flávio Antonio Alves

31

RAÍZES DA UNIDADE Anderson Torres & Eduardo Tengan

33 COMO É QUE FAZ

42

SOLUÇÕES DE PROBLEMAS PROPOSTOS

45

PROBLEMAS PROPOSTOS

59

AGENDA OLÍMPICA

61

COORDENADORES REGIONAIS

62

Page 2: Eureka 2011

Sociedade Brasileira de Matemática

EUREKA! N°33, 2011

2

XVI OLIMPÍADA DE MAIO PRIMEIRO NÍVEL

PROBLEMA 1 Um recipiente fechado com formato de paralelepípedo retangular contém 1 litro de água. Se o recipiente se apoia horizontalmente sobre três faces distintas, o nível da água é de 2cm, 4cm e 5cm. Calcule o volume do paralelepípedo. PROBLEMA 2 Na etapa 0 escrevem-se os números 1, 1. Na etapa 1 intercala-se a soma dos números 1, 2, 1. Na etapa 2 entre cada par de números da etapa anterior intercala-se a soma deles: 1, 3, 2, 3, 1. Uma etapa mais: 1, 4, 3, 5, 2, 5, 3, 4, 1. Quantos números há na etapa 10? Qual é a soma de todos os números que há na etapa 10? PROBLEMA 3 É possível pintar os inteiros positivos com três cores de modo que, sempre que se somam dois números de cores distintas, o resultado da soma seja da terceira cor? (Há que usar as três cores.) Se a resposta é afirmativa, indique um possível modo de pintar; se não é possível, explique o porquê. PROBLEMA 4 Encontre todos os números naturais de 90 dígitos que são múltiplos de 13 e têm os primeiros 43 dígitos iguais entre si e distintos de zero, os últimos 43 dígitos iguais entre si, e os 4 dígitos do meio são 2, 0, 1, 0, nessa ordem. PROBLEMA 5 Num tabuleiro de 2 × 7 quadriculado em casas de 1 × 1 se consideram os 24 pontos que são vértices das casas. João e Matias jogam sobre este tabuleiro. João pinta de vermelho uma quantidade igual de pontos em cada uma das três linhas horizontais. Se Matias pode escolher três pontos vermelhos que sejam vértices de um triângulo acutângulo, Matias vence o jogo. Qual é a máxima quantidade de pontos que João pode pintar para ter certeza de que Matias não vencerá? (Para o número encontrado, dê um exemplo de pintura que impeça que Matias vença e justifique por quê Matias vence sempre se o número é maior.)

Page 3: Eureka 2011

Sociedade Brasileira de Matemática

EUREKA! N°33, 2011

3

SEGUNDO NÍVEL PROBLEMA 1 Determine o menor inteiro positivo que tenha todos seus dígitos iguais a 4, e que seja múltiplo de 169.

PROBLEMA 2 Consideramos o retângulo ABCD e a circunferência de centro D e raio DA, que corta o prolongamento do lado AD no ponto P. A reta PC corta a circunferência no ponto Q e o prolongamento do lado AB no ponto R. Demonstre que QB = BR.

PROBLEMA 3 Encontre o menor k > 2 para o qual existem k números inteiros consecutivos, tais que a soma dos seus quadrados é um quadrado.

PROBLEMA 4 Seja n um inteiro tal que 1 < n < 2010. Dado um polígono regular de 2010 lados e n moedas, devemos pintar os vértices do polígono utilizando n cores dadas, e logo colocar as n moedas em n vértices do polígono. Em seguida, a cada segundo, todas as moedas se deslocam para o vértice vizinho, girando no sentido dos ponteiros do relógio. Determine os valores de n para os quais é possível pintar e escolher as posições iniciais das moedas, de forma que em todo momento as n moedas estejam todas em vértices de cores distintas.

PROBLEMA 5 Temos as seguintes peças: um retângulo de 4 × 1, dois retângulos de 3 × 1, três retângulos de 2 × 1 e quatro quadrados de 1 × 1. Ariel e Bernardo jogam o seguinte jogo num tabuleiro de n × n , onde n é um número escolhido por Ariel. A cada movimento, Bernardo recebe de Ariel uma peça R. Em seguida Bernardo analisa se poderá colocar R no tabuleiro de modo que não tenha pontos em comum com nenhuma das peças colocadas anteriormente (nem sequer um vértice em comum). Se existe uma tal colocação para R, Bernardo deve escolher uma delas e colocar R.

Page 4: Eureka 2011

Sociedade Brasileira de Matemática

EUREKA! N°33, 2011

4

O jogo para se é impossível colocar R da forma explicada, e Bernardo vence. Ariel vence somente se estão colocadas as 10 peças no tabuleiro. a) Suponhamos que Ariel dá as peças a Bernardo em ordem decrescente de tamanho. Qual é o menor n que garante a vitória do Ariel? b) Para o n encontrado em a), se Bernardo recebe as peças em ordem crescente de tamanho. Ariel tem garantida a vitória? ESCLARECIMENTO: cada peça deve cobrir exatamente um número de quadrados unitários do tabuleiro igual ao seu próprio tamanho. Os lados das peças podem coincidir com partes da borda do tabuleiro. RESULTADO BRASILEIRO 2010: Nível 1 (até 13 anos) Nome Cidade – Estado Prêmio Murilo Corato Zanarella Amparo – SP Medalha de Ouro Daniel de Almeida Souza Brasília – DF Medalha de Prata Viviane Silva Souza Freitas Salvador – BA Medalha de Prata Carolina Lima Guimarães Vitória – ES Medalha de Bronze Pedro Henrique Alencar Costa Fortaleza – CE Medalha de Bronze Samuel Brasil de Albuquerque Fortaleza – CE Medalha de Bronze Juliana Amoedo Plácido Salvador – BA Medalha de Bronze Lucca Morais de Arruda Siaudjionis Fortaleza – CE Menção Honrosa Antonio Wesley de Brito Vieira Cocal dos Alves – PI Menção Honrosa 2010: Nível 2 (até 15 anos) Nome Cidade – Estado Prêmio Rafael Kazuhiro Miyazaki São Paulo – SP Medalha de Ouro Lucas Cauai Julião Pereira Caucaia – CE Medalha de Prata Pedro Ivo Coêlho de Araújo Caucaia – CE Medalha de Prata Francisco Markan Nobre de Souza Filho Fortaleza – CE Medalha de Bronze Fellipe Sebastiam da Silva Paranhos Pereira Rio de Janeiro – RJ Medalha de Bronze Tadeu Pires de Matos Belfort Neto Fortaleza – CE Medalha de Bronze Henrique Gasparini Fiuza do Nascimento Brasília – DF Medalha de Bronze Rafael Rodrigues Rocha de Melo Caucaia – CE Menção Honrosa Mateus Henrique Ramos de Souza Pirapora – MG Menção Honrosa

Page 5: Eureka 2011

Sociedade Brasileira de Matemática

EUREKA! N°33, 2011

5

XXI OLIMPÍADA DE MATEMÁTICA DO CONE SUL Enunciados e resultado brasileiro

O Brasil, e particularmente o Estado de São Paulo teve a honra de sediar a 21ª Olimpíada de Matemática do Cone Sul, que aconteceu até o dia 19 de junho na cidade de Águas de São Pedro, SP. A equipe foi liderada pelos professores Francisco Bruno Holanda, de Fortaleza – CE e Tertuliano Franco Santos Franco, de Rio de Janeiro – RJ.

RESULTADOS DA EQUIPE BRASILEIRA BRA1 João Lucas Camelo Sá Medalha de Ouro BRA2 Gabriel Militão Vinhas Lopes Medalha de Prata BRA3 Maria Clara Mendes Silva Medalha de Prata BRA4 Caíque Porto Lira Medalha de Bronze

PRIMEIRO DIA PROBLEMA 1 Pedro tem que escolher duas frações irrredutíveis, cada uma com numerador e denominador positivos, tais que:

• A soma das duas frações seja igual a 2. • A soma dos numeradores das duas frações seja igual a 1000.

De quantas maneiras Pedro pode fazer isso? PROBLEMA 2 Marcam-se em uma reta 44 pontos, numerados 1, 2, 3, ..., 44 da esquerda para a direita. Vários grilos saltam na reta. Cada grilo parte do ponto 1, salta por pontos marcados e termina no ponto 44. Além disso, cada grilo sempre salta de um ponto marcado a outro marcado com um número maior. Quando todos os grilos terminaram da saltar, notou-se que para cada par i, j, com 1 44,i j≤ ≤ ≤ há um grilo que saltou diretamente do ponto i para o ponto j, sem pousar em nenhum dos pontos entre eles. Determine a menor quantidade de grilos para que isso seja possível.

Page 6: Eureka 2011

Sociedade Brasileira de Matemática

EUREKA! N°33, 2011

6

PROBLEMA 3 Recortar um polígono convexo de n lados significa escolher um par de lados consecutivos AB, BC do polígono e substituí-los por três segmentos AM, MN, e NC, sendo M o ponto médio de AB e N o ponto médio de BC. Em outras palavras, corta-se o triângulo MBN e obtem-se um polígono convexo de n + 1 lados. Seja P6 um hexágono regular de área 1. Recorta-se P6 e obtém-se o polígono P7. Então recorta-se P7, de uma das sete maneiras possíveis, e obtém-se o polígono P8, e assim sucessivamente. Prove que, independentemente de como sejam feitos os recortes, a área de Pn é sempre maior do que 2 3. SEGUNDO DIA PROBLEMA 4 Pablo e Sílvia jogam em um tabuleiro 2010 × 2010. Primeiro Pablo escreve um número inteiro em cada casa. Feito isso, Sílvia repete tantas vezes quanto quiser a seguinte operação: escolher três casas que formem um L, como uma figura, e somar 1 a cada número dessas três casas. Sílvia ganha se fizer com que todos os números do tabuleiro sejam múltiplos de 10. Demonstre que Sílvia sempre pode escolher uma sequência de operações com as quais ela ganha o jogo.

PROBLEMA 5 O incírculo do triângulo ABC toca os lados BC, CA, e AB em D, E e F, respectivamente. Sejam , e a b cω ω ω os circuncírculos dos triângulos EAF, DBF e DCE, respectivamente. As retas DE e DF cortam aω em aE E≠ e

,aF F≠ respectivamente. Seja Ar a reta .a aE F Defina e B Cr r de modo análogo. Prove que as retas , e A B Cr r r determinam um triângulo cujos vértices pertencem aos lados do triângulo ABC. PROBLEMA 6 Determine se existe uma sequência infinita 0 1 2 3, , , ,...a a a a de inteiros não negativos que satisfaz as seguintes condiciones: (i) Todos os números inteiros não negativos aparecem na sequência uma única vez; (ii) A sequência , 0,n nb a n n= + ≥ é formada por todos os números primos, cada um aparecendo uma única vez.

Page 7: Eureka 2011

Sociedade Brasileira de Matemática

EUREKA! N°33, 2011

7

LI OLIMPÍADA INTERNACIONAL DE MATEMÁTICA (IMO) Enunciados e resultado Brasileiro

A LI Olimpíada Internacional de Matemática (IMO) foi realizada na cidade

de Astana, Cazaquistão entre os dias 2 e 14 de julho de 2010. A equipe foi liderada pelos professores Edmilson Luis Rodrigues Motta, de São Paulo – SP e Marcelo Mendes de Oliveira, de Fortaleza – CE.

RESULTADOS DA EQUIPE BRASILEIRA BRA1 Marcelo Tadeu de Sá Oliveira Sales Medalha de Prata BRA2 Matheus Secco Torres da Silva Medalha de Prata BRA3 Gustavo Lisbôa Empinotti Medalha de Bronze BRA4 Deborah Barbosa Alves Menção Honrosa BRA5 Hanon Lima Rossi Menção Honrosa BRA6 João Lucas Camelo Sá Menção Honrosa PRIMEIRO DIA PROBLEMA 1 Determine todas as funções :f → tais que

( ) ( ) ( )f x y f x f y=

para os números , .x y∈ ( z designa o maior inteiro que é menor ou igual a z).

PROBLEMA 2 Seja ABC um triângulo, I o seu incentro e Γ a sua circunferência circunscrita. A recta AI intersecta novamente Γ no ponto D. Sejam E um ponto do arco BDC e F um ponto do lado BC tais que

1 .2

BAF C AE B AC= <

Seja G o ponto médio do segmento IF. Mostre que as rectas DG e EI se intersectam sobre Γ . PROBLEMA 3 Seja * o conjunto dos inteiros positivos. Determine todas as funções

: * *g → tais que

Page 8: Eureka 2011

Sociedade Brasileira de Matemática

EUREKA! N°33, 2011

8

( )( ) ( )( )g m n m g n+ + é um quadrado perfeito para todos , *.m n∈

SEGUNDO DIA PROBLEMA 4 Seja Γ a circunferência circunscrita ao triângulo ABC e P um ponto no interior do triângulo. As rectas AP, BP e CP intersectam novamente Γ nos pontos K, L, e M, respectivamente. A recta tangente a Γ em C intersecta a recta AB em S. Supondo que SC = SP, mostre que MK = ML. PROBLEMA 5 Em cada uma das seis caixas 1 2 3 4 5 6, , , , ,B B B B B B há inicialmente só uma moeda. Dois tipos de operações são possíveis: Tipo 1: Escolher uma caixa não vazia ,jB com 1 5.j≤ ≤ Retirar uma moeda da jB e

adicionar duas moedas a 1.jB + Tipo 2: Escolher uma caixa não vazia ,kB com 1 4.k≤ ≤ Retirar uma moeda da kB e trocar os conteúdos das caixas (possivelmente vazias) 1kB + e 2.kB + Determine se existe uma sucessão finita destas operações que deixa as caixas

1 2 3 4 5, , , ,B B B B B vazias e a caixa 6B com exactamente 201020102010 moedas. (Observe

que ( ).cc bba a= )

PROBLEMA 6 Seja 1 2 3, , ,...a a a uma sucessão de números reais positivos. Sabe-se que para algum inteiro positivo s,

{ }max tal que 1 1n k n ka a a k n−= + ≤ ≤ − para todo n > s. Mostre que existem inteiros positivos e N, com ,s≤ tais que

n na a a −= + para todo .n N≥

Page 9: Eureka 2011

Sociedade Brasileira de Matemática

EUREKA! N°33, 2011

9

XXV OLIMPÍADA IBEROAMERICANA DE MATEMÁTICA Enunciados e resultado Brasileiro

A XXV Olimpíada Iberoamericana de Matemática foi realizada na cidade

de Assunção, Paraguai no período de 20 a 30 de setembro de 2010. A equipe brasileria foi liderada pelos professores Onofre Campos, de Fortaleza – CE e Luzinalva Miranda de Amorim, de Salvador – BA. A equipe brasileira ficou em primeiro lugar na soma dos pontos dos participantes. RESULTADOS DA EQUIPE BRASILEIRA BRA1 Marcelo Tadeu de Sá Oliveira Sales Medalha de Ouro BRA2 Deborah Barbosa Alves Medalha de Ouro BRA3 Matheus Secco Torres da Silva Medalha de Ouro BRA4 Gustavo Lisboa Empinotti Medalha de Prata PRIMEIRO DIA PROBLEMA 1 Numa fila de dez moedas indistinguíveis há duas delas que são falsas, ocupando posições consecutivas. Para cada conjunto de posições, pode-se perguntar quantas moedas falsas ele contém. É possível determinar quais são as moedas falsas fazendo apenas duas destas perguntas? Não se sabe a resposta da primeira pergunta antes de se formular a segunda. PROBLEMA 2 Determinar se existem números inteiros positivos a e b tais que todos os termos da sucesão definida por 1 22010, 2011,x x= =

2 1 1 , 1,n n n n nx x x a x x b n+ + += + + + ≥ sejam inteiros. PROBLEMA 2 A circunferência Γ inscrita ao triângulo escaleno ABC é tangente aos lados BC, CA e AB nos pontos D, E e F respectivamente. A recta EF corta a recta BC em G. A circunferência de diâmetro GD corta Γ em ( ).R R D≠ Sejam P e Q

( ), P R Q R≠ ≠ as intersecções de BR e CR com Γ , respectivamente. As rectas BQ

Page 10: Eureka 2011

Sociedade Brasileira de Matemática

EUREKA! N°33, 2011

10

e CP cortam-se em X. A circunferência circunscrita a CDE corta o segmento QR em M e a circunferência circunscrita a BDF corta o segmento PR em N. Demonstrar que as rectas PM, QN e RX são concorrentes. SEGUNDO DIA PROBLEMA 4 As médias aritmética, geométrica e harmônica de dois números inteiros positivos distintos são números inteiros. Encontrar o menor valor possível para a média aritmética. Nota: Se a e b são números positivos, suas médias aritméticas, geométrica e

harmônica são respectivamente: ,2

a b a b+⋅ e 2 .1 1

a b+

PROBLEMA 5 Seja ABCD um quadrilátero cíclico sujas diagonais AC e BD são perpendiculares. Sejam O o circuncentro de ABDC, K a intersecção das diagonais, L O≠ a intersecção das circunferências circunscritas a OAC e OBD, e G a intersecção das diagonais do quadrilátero cujos vértices são os pontos médios dos lados de ABCD. Provar que O, K, L e G são colineares. PROBLEMA 6 Ao redor de uma mesa circular sentam-se 12 pessoas e sobre a mesa há 28 vasos de flores. Duas pessoas podem ver-se uma à outra se, e somente se, não há nenhum vaso alinhado com elas. Provar que existem pelo menos duas pessoas que podem ver-se.

Page 11: Eureka 2011

Sociedade Brasileira de Matemática

EUREKA! N°33, 2011

11

ASSOCIANDO UM POLINÔMIO A EXPRESSÕES ALGÉBRICAS E TRIGONOMÉTRICAS

Marcílio Miranda, IFRN (Caicó – RN) ♦ Nível Intermediário

O objetivo deste artigo é mostrar uma técnica que pode ser bastante útil na hora de resolver problemas de olimpíadas de Matemática.Tal técnica consiste em você associar um polinômio a uma determina expressão. Com isso você pode calcular o valor de expressões trigonométricas, expressões algébricas e mostrar que um determinado número é irracional. Vejamos alguns exemplos disso: I) EXPRESSÕES TRIGONOMÉTRICAS Esse problema deixa bem clara a idéia de associarmos um polinômio a uma expressão trigonométrica: EXERCÍCIO RESOLVIDO 1 (BÉLGICA 2006): a) Encontre todos os números reais α tais que ( ) ( )cos 4 cos 3=α α

b) Determine inteiros a, b, c, d tais que 2cos ,7π 4cos ,

7π 6cos ,

7π são soluções da

equação 3 2 0.ax bx cx d+ + + = SOLUÇÃO: a) ( ) ( )cos 4 cos 3 4 3 2k= ⇔ = +α α α α π ou 4 3 2 2k k= − + ⇔ =α α π α π ou

2 ,7k

=πα logo 2 4 61,cos ,cos , cos

7 7 7π π π são as raízes dessa equação.

Por outro lado temos que ( ) 4 2cos 4 8 cos 8 cos 1= ⋅ − ⋅ +α α α e

( ) 3cos 3 4 cos 3 cos .= ⋅ − ⋅α α α Faça cos .t=α Daí temos que

( ) ( ) ( ) ( )3 2 4 3 2cos 4 cos 3 1 8 4 4 1 8 4 8 3 1 0.t t t t t t t t= ⇔ − ⋅ + − − = ⋅ − ⋅ − ⋅ + ⋅ + =α α

Assim, a equação ( )3 28 4 4 1 0t t t+ − − = tem como

soluções 2 4 6cos ,cos ,cos .7 7 7π π π

Page 12: Eureka 2011

Sociedade Brasileira de Matemática

EUREKA! N°33, 2011

12

EXERCÍCIO RESOLVIDO 2 (MOCP, JULHO DE 2003): Prove que sec 400 + sec 800 + sec 1600 = 6. SOLUÇÃO: Note que 400, 800 e 1600 satisfazem a equação

31cos3 8cos 6cos 1 0,2

= − ⇒ − + =α α α logo cos 400, cos 800, cos 1600 são as

raízes do polinômio 8cos3 α – 6 cos α +1, e assim temos que: 6cos 40 cos80 cos160 cos80 cos 40 160

8−

° ⋅ ° + ° ⋅ ° + ° ⋅ ° =

1cos 40 cos80 cos1608−

° ⋅ ° ⋅ ° =

1 1 1sec40 sec80 sec160cos40 cos80 cos160

° + ° + ° = + + =° ° °

cos 40 cos80 cos 40 cos80 cos 40 cos80 6.cos160 cos80 cos 40

° ⋅ ° + ° ⋅ ° + ° ⋅ °=

° ° ⋅ °

EXERCÍCIO RESOLVIDO 3 (IMO 1963): Prove que 2 3 1cos cos cos .7 7 7 2− + =

π π π

SOLUÇÃO: Note que 3 3 5 53 4 ,3 4 3 e 3 4 5 ,

7 7 7 7 7 7⋅ + ⋅ = ⋅ + ⋅ = ⋅ + ⋅ =π π π π π ππ π π logo

3 5, ,7 7 7π π π

são

soluções da equação cos4 cos3 .x x= −

Essa equação equivale a 7 7cos 4 cos3 0 2 cos cos 0 cos 02 2 2x x xx x+ = ⇔ ⋅ ⋅ = ⇔ = ou

cos 0.2x=

PARTE 1: Resolver a equação 7cos 02x=

7 2 3 5 9 11 13, , , , , , ,2 2 7 7 7 7 7 7 7 7x kk x x= + ⇒ = + ⇒ =

π π π π π π π π ππ π mas

13 3 11 5 9cos cos ,cos cos ,cos cos ,7 7 7 7 7 7= = =

π π π π π π logo há 4 soluções distintas

entre 0 e 2 :π 3 5, , , .7 7 7π π π π

PARTE 2: Resolver a equação cos 02x=

Page 13: Eureka 2011

Sociedade Brasileira de Matemática

EUREKA! N°33, 2011

13

2 ,2 2x k x k= + ⇒ = +

π π π π logo x =π é a única solução entre 0 e 2 .π

Por outro lado temos que 4 2cos 4 8cos 8cos 1x x x= − + e 3cos3 4cos 3cos .x x x= − 4 3 2 4 3 2cos4 cos3 8cos 4cos 8cos 3cos 1 0 8 4 8 3 1 0,x x x x x x t t t t= − ⇔ + − − + = ⇔ + − − + =

onde cos .t x= Claramente –1 é raiz desse polinômio, e temos 8t4 +4t3 – 8t2 – 3t + 1 = (t +1) · (8t3 – 4t2 – 4t + 1), donde o polinômio 8t3 – 4t2 – 4t + 1 tem como raízes

3 5cos ,cos ,cos .7 7 7π π π Logo temos pelas relações de Girard que:

3 5 4 1 2 3cos cos cos cos cos cos .7 7 7 8 2 7 7 7+ + = = = − −

π π π π π π

II) CALCULANDO O VALOR DE UMA EXPRESSÃO ALGÉBRICA:

EXERCÍCIO RESOLVIDO 4: Prove que 3 32 220 14 2 20 14 2 4.+ + − =

SOLUÇÃO: Seja 3 32 220 14 2 20 14 2 .x = + + − Temos x3 = 40 + 6x ⇒ x3 – 6x – 40 = 0. É fácil ver que 4 é raiz desse polinômio e x3 – 6x – 40 = (x – 4).(x2 + 4x + 10).

Note que as raízes de x2 + 4x + 10 não são reais e 3 32 220 14 2 20 14 2+ + − é

real, logo 3 32 220 14 2 20 14 2 4.+ + − = EXERCÍCIO RESOLVIDO 5 (CROÁCIA 2001): Se a + b + c = 0, calcule o valor da

expressão ( )7 7 7

4 4 4.a b c

abc a b c+ +

⋅ + +

SOLUÇÃO: Seja x3 + mx2 + px + q = 0. um polinômio de terceiro grau tal que suas raízes são a, b, c. Daí temos que a + b + c = – m = 0, ab + ac + bc = p e abc = – q. Assim temos que: (a + b + c)2 = a2 + b2 + c2 + 2⋅(ab + ac + bc) ⇒ a2 + b2 + c2 = –2p Por outro lado temos que: a3 + pa + q = 0 ⇒ a3 = – pa – q (i) b3 + pb + q = 0 ⇒ b3 = – pb – q (ii) c3 + pc + q = 0 ⇒ a3 = – pc – q (iii) somando (i) + (ii) + (iii), temos que a3 + b3 + c3 = – p.(a + b + c) – 3q = –3q Da mesma forma temos que: a4 + pa2 + qa = 0 ⇒ a4 = – pa2 – qa (iv)

Page 14: Eureka 2011

Sociedade Brasileira de Matemática

EUREKA! N°33, 2011

14

b4 + pb2 + qb = 0 ⇒ b4 = – pb2 – qb (v) c4 + pc2 + qc = 0 ⇒ c4 = – pc2 – qc (vi) somando (iv) + (v) + (vi), temos que a4 + b4 + c4 = – p.(a2 + b2 + c2) – q.(a + b + c) = 2p2. Analogamente temos que: a5 + pa3 + qa2 = 0 ⇒ a5 = – pa3 – qa2 (vii) b5 + pb3 + qb2 = 0 ⇒ b5 = – pb3 – qb2 (viii) c5+ pc3 + qc2 = 0 ⇒ c5 = – pc3 – qc2 (ix) somando (vii) + (viii) + (ix), temos que a5 + b5 + c5 = – p · (a3 + b3 + c3) – q · (a2 + b2 + c2) = 5pq. Proseguindo do mesmo modo, temos que: a7 + pa5 + qa4 = 0 ⇒ a7 = – pa5 – qa4 (x) b7 + pb5 + qb4 = 0 ⇒ b7 = – pb5 – qb4 (xi) c7 + pc5 + qc4 = 0 ⇒ c7 = – pc5 – qc4 (xii) somando (x) + (xi) + (xii): a7 + b7 + c7 = – p · (a5 + b5 + c5) – q · (a4 + b4 +c4) = – 7p2q.

Com isso temos que ( ) ( )7 7 7 2

4 4 4 2

7 7 .22

a b c p qabc a b c q p

+ + −= =

⋅ + + − ⋅

III) PROVANDO A IRRACIONALIDADE DE UM NÚMERO: Antes do próximo problema vamos provar o seguinte teorema:

TEOREMA (TESTE DA RAIZ RACIONAL): Se o número pq

, onde p e q são inteiros e

mdc(p, q) = 1, é uma raiz do polinômio com coeficientes inteiros 1

1 1 0... ,n nn na x a x a x a−

−⋅ + ⋅ + + ⋅ + então p é um divisor de a0 e q é um divisor de an.

PROVA: Como pq

é raiz do polinômio temos que

11 1

1 1 0 1 1 0... 0 ... 0,n n

n n n nn n n n

p p pa a a a a p a p q a p q a qq q q

− −− −

+ ⋅ + + ⋅ + = ⇒ ⋅ + ⋅ ⋅ + + ⋅ ⋅ + ⋅ =

logo temos que p é um divisor de a0 e q é um divisor de an. EXERCÍCIO RESOLVIDO 6: Prove que 2 3+ é irracional.

Page 15: Eureka 2011

Sociedade Brasileira de Matemática

EUREKA! N°33, 2011

15

Solução: Seja x = 2 3+ ⇒ x – 2 3= ⇒ x2 – 1 = 2 2 x ⇒ x4 – 2x2 + 1 = 8x2 ⇒ x4 – 10x2 + 1 = 0. Logo pelo teorema acima as raízes racionais da equação só podem ser 1 ou –1, que claramente não são soluções (em ambos os casos o valor numérico do polinômio é –8). Logo esse polinômio só possui raízes irracionais, portanto 2 3+ é irracional. EXERCÍCIOS PROPOSTOS:

1) (EUA) Prove que 2 3 7 .7 7 7 8

sen sen sen⋅ ⋅ =π π π

2) (Vietnã 1982) Ache a, b, c inteiros tais que as raízes da equação ax2 + bx + c = 0 são cos 720 e cos 1440. 3) (Prova de Seleção da Romênia para a IMO 1970): Prove que para todo inteiro positivo n:

( )12 3 ... 2 1.2 1 2 1 2 1 2 1 2 1

n ntg tg tg tg tg nn n n n n

−⋅ ⋅ ⋅ = +

+ + + + +ππ π π π

4) (Prova de Seleção da Suíça para a IMO 2004): Sejam a, b, c, d números reais distintos satisfazendo as equações:

45 21 ,a a= − − 45 21 ,b b= − − 45 21 ,c c= − − 45 21d d= − − Prove que abcd = 2004. 5) (OBM 2003): Sejam a, b, c números reais não-nulos tais que a + b + c = 0.

Calcule os possíveis valores de ( ) ( )

( )

23 3 3 4 4 4

25 5 5.

a b c a b c

a b c

+ + ⋅ + +

+ +

6) (Bélgica 1978): Encontre um polinômio com coeficientes inteiros tal que 2 3+ é raiz.

7) (Moldávia 2000): Os números a, b, c satisfazem a relação a + b + c = 0. Mostre que o número 2a4 + 2b4 +2c4 é um quadrado perfeito. 8) Prove que 32 3+ é irracional.

9) Prove que x = 2cos 7π satisfaz a equação: x3 + x2 – 2x + 1 = 0.

Use este fato para provar que cos 7π é irracional.

10) Prove que tg2 10 + tg2 30 +....+ tg2 870 + tg2 890 = 4005.

Page 16: Eureka 2011

Sociedade Brasileira de Matemática

EUREKA! N°33, 2011

16

11) Prove que cos 200. cos 400.cos 800 = 18

.

12) Prove que:

a) 2 3 77 7 7

tg tg tg⋅ ⋅ =π π π

.

b) 2 3 4 5 6 1313 13 13 13 13 13

tg tg tg tg tg tg⋅ ⋅ ⋅ ⋅ ⋅ =π π π π π π

.

13) Prove que cossec 6° + cossec 78° − cossec 42° − cossec 66° = 8. 14) Calcule as expressões:

a) 2 2 22 37 7 7

tg tg tg⋅ ⋅π π π

.

b) 2 2 22 37 7 7

tg tg tg+ +π π π

.

c) 2 2 2 2 2 22 3 2 3 .7 7 7 7 7 7

tg tg tg tg tg tg⋅ + ⋅ + ⋅π π π π π π

15) Prove que 2 4 1cos cos cos .7 7 7 8⋅ ⋅ = −

π π π

16) Ache uma equação do terceiro grau cujas raízes são 3 5cos ,cos ,cos .7 7 7π π π

17) Calcule as expressões:

a) 3 5cos cos cos .7 7 7⋅ ⋅

π π π

b) 3 5cos cos cos7 7 7⋅ +

π π π 3 5cos cos cos .7 7 7+ ⋅

π π π

c) 3 5cos cos cos .7 7 7+ +

π π π

d) 2 2 23 5cos cos cos .7 7 7+ +

π π π

e) 1 1 1 .3 5cos cos cos7 7 7

+ +π π π

18) Prove que tg 810 – tg 630 + tg 90 – tg 270 = 4.

Page 17: Eureka 2011

Sociedade Brasileira de Matemática

EUREKA! N°33, 2011

17

19) Sejam u, v, w as raízes do polinômio x3 – 10x + 11. Determine o valor de arctg u + arctg v + arctg w.

20) Prove que cossec 18π + cossec 5

18π + cossec 13

18π = 6.

21) Prove que tg 200. tg400. tg 600. tg 800 = 3. 22) Sejam a, b, c números reais tais que a + b + c = 0, prove que: a) a3 + b3 + c3 = 3abc.

b) 2 2 2 5 5 5 7 7 7

.2 5 5

a b c a b c a b c+ + + + + +⋅ =

23) Prove que 3sen20 sen40 sen80 .8

° ⋅ ° ⋅ ° =

24) Prove que 2 2 22 3cot cot cot 5.7 7 7

g g g+ + =π π π

25) Calcule o valor da expressão 4 7 .9 9 9

tg tg tg+ +π π π

REFERÊNCIAS [1] MIRANDA, Marcílio. Problemas Selecionados de Matemática ITA-IME – Olimpíadas, Volume 1, Fortaleza (CE), Editora Vestseller, 2010. [2] ANDREESCU, Titu; FENG , Zuming. 103 Trigonometry Problems from the Training of the USA IMO Team, Birkhauser, 2004. [3] ANDREESCU, Titu; GELCA, Razvan. Putnam and Beyond. New York: Springer-Verlag, 2006. [4] DOMINGUES, Hygino. Fundamentos de Aritmética, São Paulo, Atual Editora, 1991. SITES ACESSADOS [1] The IMO Compendium, Disponível em <http://www.imomath.com/index.php?options=oth|other&p=0>, Acesso em: 10/08/2009. [2] Treinamento do Cone Sul. Disponível em: < http://treinamentoconesul.blogspot.com/>, Acesso em: 12/08/2009. [3]Notas de Aula de Kin Yin Li. Disponível em: <http://www.math.ust.hk/~makyli/190_2003Fa/lect-notes_03fa.pdf>, Acesso em: 15/08/2009. [4] Página de Olimpíada da Sociedade Canadense de Matemática. Disponível em: < http://www.cms.math.ca/Olympiads/ >, Acesso em: 20/07/2009. [5] Matemática Nick Puzzles. Disponível em: < http://www.qbyte.org/puzzles/>, Acesso em : 15/11/2009. [6] Olimpíada Brasileira de Matemática. Disponível em: <http://www.obm.org.br >, Acesso em: 20 /11/2009.

Page 18: Eureka 2011

Sociedade Brasileira de Matemática

EUREKA! N°33, 2011

18

SOMAS TRIGONOMÉTRICAS: DE PROSTAFÉRESE À FÓRMULA DE EULER

Rogério Possi Junior ♦ Nível Intermediário

INTRODUÇÃO

São apresentados fundamentos básicos da matemática elementar, cujos

conceitos somados podem auxiliar na resolução de problemas mais elaborados, como os que podem aparecer quando se depara com o início do estudo das Variáveis Complexas e o uso dos teoremas de De Moivre. Seja através das fórmulas de Transformação de Soma em Produto, conhecidas como Fórmulas de Prostaférese, ou através da Relação de Euler, são calculados alguns exemplos de somas de funções trigonométricas aparentemente complexas. AS FÓRMULAS DE TRANSFORMAÇÃO TRIGONOMÉTRICAS.

Admitamos conhecidas as fórmulas da soma e diferença de arcos para as funções “seno” e “cosseno”, isto é

abbaba cossencossen)sen( +=+ (a) abbaba cossencossen)sen( −=− (b) bababa sensencoscos)cos( −=+ (c) bababa sensencoscos)cos( +=− (d)

Somando-se (a) e (b) tem-se

bababa cossen2)sen()sen( =−++ (e)

Subtraindo-se (a) de (b) tem-se abbaba cossen2)sen()sen( =−−+ (f)

Somando-se (c) e (d) teremos

bababa coscos2)cos()cos( =−++ (g)

E por fim, subtraindo-se (c) de (d)

Page 19: Eureka 2011

Sociedade Brasileira de Matemática

EUREKA! N°33, 2011

19

bababa sensen2)cos()cos( −=−−+ (h)

Fazendo α=+ ba e β=−ba teremos que 2βα +

=a e 2βα −

=b , cujos

valores substituídos nas relações (e), (f), (g) e (h) fornecerão as seguintes relações

+

=+2

cos2

sen2sensen βαβαβα

+

=−2

cos2

sen2sensen βαβαβα

+

=+2

cos2

cos2coscos βαβαβα

+

−=−2

sen2

sen2coscos βαβαβα , que são as conhecidas

Fórmulas de Transformação de soma em produto ou Fórmulas de Prostaférese. A FÓRMULA DE EULER Segundo GUIDORIZZI (1987), seja )(xf uma função derivável até a ordem n em um intervalo aberto I e seja Ix ∈0 . Define-se o polinômio )(xP a seguir como o

polinômio de Taylor, de ordem n , de )(xf em torno do ponto 0x , isto é

( ) ( ) ( )( ) ( ) ( ) ( ) ( ) ( ) ( )0 00 0 0 0 0

1

n knn k

k

x x x xP x f x f x x x f x f x

n! k!=

− −′= + − + + =∑… (i)

que, se fixado em torno de 0 0x ,= também pode ser chamado de polinômio de Mac-Laurin. Tomando-se (i) ( ) xf x e= e 0 0x ,= pode-se demonstrar que

…… ++++=

+++++=

∞→ !321

!!321lim

3232 xxxnxxxxe

n

n

x (j)

A expressão da direita pode ser usada para definir xe para x para x complexo. Analogamente demonstra-se que

Page 20: Eureka 2011

Sociedade Brasileira de Matemática

EUREKA! N°33, 2011

20

…… −+−=

−+−+−=

∞→ !5!3!)1(

!5!3limsen

532

153 xxxnxxxxx

nn

n (k)

e que

…… −+−=

−+++−=

∞→ !4!21

)!2()1(

!421limcos

42242 xxn

xxxxn

n

n (l)

Para RYiYZx ∈== , e observando-se (j), (k) e (l) teremos que

YiYYYYiYYeiY sencos!5!3!42

15342

+=

−+−+

−+−= …… (m)

que é a conhecida fórmula de Euler. Não obstante, também se demonstra que se iYXZ ee += , onde 0≠X , então

( )YiYee XZ sencos += (n) Se, alternativamente, adotássemos a expressão de (n) como definição de Ze , não é difícil mostrar que Z W Z We e e , Z ,W+ = ⋅ ∀ ∈ de fato, se 1 1 1Z X iY= + e

2 2 2Z X iY ,= + ( ) ( )( )1 2 1 2 1 21 2 1 2

Z Z x x x xe e cos Y Y isen Y Y e+ + += + + + =

( )( )1 2 1 2 1 2 2 1cos cos sen sen sen cos sen cosY Y Y Y i Y Y Y Y− + + =

( ) ( )1 2 1 21 1 2 2cos sen cos senx x Z Ze Y i Y e Y i Y e e .= + ⋅ + = ⋅

PROBLEMAS DE APLICAÇÃO

PROBLEMA 1: Começaremos com um exemplo de problema análogo ao proposto em um exame de admissão ao Instituto Militar de Engenharia (IME). O problema pede que se calcule as somas a seguir.

nxxxxS sen3sen2sensen1 ++++= … (1)

2 2S cos x cos x ... cos nx= + + + (2)

Utilizaremos a transformação de somas de funções trigonométricas em produto, conhecidas como “Fórmulas de Prostaférese”. Observamos que

Page 21: Eureka 2011

Sociedade Brasileira de Matemática

EUREKA! N°33, 2011

21

nxxxnxn

xnxxnxn

xxxx

xxxx

xxxx

cos2

sen22

)12(sen2

)12(sen

)1cos(2

sen22

)32(sen2

)12(sen

3cos2

sen22

5sen2

7sen

2cos2

sen22

3sen2

5sen

cos2

sen22

sen2

3sen

=−

−+

−=−

−−

=−

=−

=−

(3)

Somando-se as linhas acima encontraremos uma “Soma Telescópica”, cujo valor será dado por

jxxxxn n

j∑=

⋅=−+

1

cos2

sen22

sen2

)12(sen

2sen

2)1(cos

2sen

cos1

2 x

xnnx

jxSn

j

+⋅

==⇒ ∑=

(4)

Analogamente, para a soma das funções “seno” 1S pode-se escrever que:

nxxxnxn

xnxxnxn

xxxx

xxxx

xxxx

sen2

sen22

)12(cos2

)12(cos

)1sen(2

sen22

)32(cos2

)12(cos

3sen2

sen22

5cos2

7cos

2sen2

sen22

3cos2

5cos

sen2

sen22

cos2

3cos

−=−

−+

−−=−

−−

−=−

−=−

−=−

(5)

Somando-se as linhas acima encontraremos outra “Soma Telescópica”, cujo valor é

Page 22: Eureka 2011

Sociedade Brasileira de Matemática

EUREKA! N°33, 2011

22

jxxxxn n

j∑=

⋅−=−+

1sen

2sen2

2cos

2)12(cos

2sen

2)1(sen

2sen

sen1

1 x

xnnx

jxSn

j

+⋅

==⇒ ∑=

(6),

que é a soma procurada.

Não obstante, este problema também pode ser resolvido utilizando-se a conhecida

Relação de Euler. Seja 2

sen2

cos2 xixCeix

+== , onde 12 −=i ; assim tem-se que

nxinxxixC

xixxixC

xixxixC

nn sencos

2sen

2cos

2sen2cos2

sen2

cos

sencos2

sen2

cos

22

44

22

+=

+=

+=

+=

+=

+=

(7)

( )( ) ( )∑

=−

+=−−

⋅=−−

=+++⇒n

j

nnnnn jxijx

CCCCCC

CCCCCC

112

22242 sencos

)1()1(…

( ) 121

2sen

2sen

2sen

2cos

2sen

2cossencos iSSx

nxnxinxxixjxijx

n

j+=

+

+=+⇒∑

=

2sen

2sen

2)1(sen

2sen

2sen

2)1(cos

12 x

nxxn

ix

nxxn

iSS⋅+

+⋅+

=+⇒ (8)

de onde tiramos os valores de interesse 1S e 2S igualando-se as partes reais e imaginárias da igualdade acima respectivamente.

Page 23: Eureka 2011

Sociedade Brasileira de Matemática

EUREKA! N°33, 2011

23

PROBLEMA 2: Considere agora o problema de se determinar as somas dadas por

∑=

=n

jjxS

1

21 sen e ∑

=

=n

jjxS

1

22 cos . Para tal, observa-se inicialmente, da

Trigonometria que xx 2cos21

21sen2 −= e xx 2cos

21

21cos2 += , assim pode-

se reescrever 1S e 2S como sendo

( )nxxxjxSn

n

j2cos4cos2cos

21

21

21

21sen

""1

21 +++−

+++==∑

=

…… (9)

onde nxxxS 2cos4cos2cos´ +++= … . Com o auxílio da Relação de Euler, seja xixC sencos += , assim

( )( ) ( )∑

=−

+=−−

=+++n

j

nnnn jxijx

CCCCCCCCC

11

242 2sen2cos.…

( ) ´´´sen

sen.)1sen(sen

sen.)1cos(2sen2cos1

iSSx

nxxnix

nxxnjxijxn

j+=

++

+=+⇒∑

=

(10)

onde ∑=

=n

jjxS

12sen´´ . Sendo ´

21

21 SnS −=

+−=∴

xnxxnnS

sensen.)1cos(

21

1 (11)

Outra solução para o cálculo da soma ´S consiste em transformá-la segundo as fórmulas de Prostaférese. Para este caso tem-se que,

nxxxnxnxnxxnxn

xxxxxxxx

2cossen2)12sen()12sen()22cos(sen2)32sen()12sen(

4cossen23sen5sen2cossen2sen3sen

=−−+−=−−−

=−=−

(12)

Page 24: Eureka 2011

Sociedade Brasileira de Matemática

EUREKA! N°33, 2011

24

Somando-se todas as linhas acima tem-se que ´sen2sen)12sen( Sxxxn ⋅=−+

xxnnxS

sen)1cos(.sen´ +

=∴ , que é exatamente o valor encontrado da parte real do

somatório dado por (10).

Observando-se (9) e que xx 2cos21

21cos2 +=

[ ]

++=+==⇒ ∑

= xnxxnnSnjxS

n

j sensen.)1cos(

21´

21cos

1

22 (13)

que resolve o problema do cálculo de 2S . PROBLEMA 3: Considere a seguir o problema do cálculo das somas dadas por

∑=

=n

kkxS

1

31 sen e ∑

=

=n

kkxS

1

32 cos .

Seja cos senC x i x.= + Sendo kxikxC K sencos ±=± pode-se escrever que

2cos

kk CCkx−+

= (14)

iCCkx

kk

2sen

−−= (15)

Elevando-se a relação (15) ao cubo tem-se que

( ) ( )i

CCCCiCCkx

kkkkkk

83

2sen

3333

−−−−

=

−=

−−−

43sensen3sen3 kxkxkx −

=⇒ (16)

∑ ∑∑= ==

−==∴

n

k

n

k

n

kkxkxkxS

1 11

31 3sensen3

41sen (17)

Por (6) tem-se que ∑=

+⋅

=n

k x

xnnx

kx1

2sen

2)1(sen

2sen

sen e observando-se que se

xixD 3sen3cos += teremos que:

Page 25: Eureka 2011

Sociedade Brasileira de Matemática

EUREKA! N°33, 2011

25

nxinxD

xixDxixD

n 3sen3cos

9sen9cos6sen6cos

3

2

+=

+=

+=

=++++=⇒−

21

21

22221

32

.

DD

DDDDDDDDS

nnn

nD …

23sen

23sen

2)13(sen

2)13(cos

nxx

xnixn

SD ⋅

+++

=⇒ (18)

Tomando-se a parte imaginária da relação (18) tem-se que

⋅+

=∑=

23sen

23sen

2)13(sen

3sen1 x

nxxn

kxn

k

(19)

Logo, por (6) e (19) teremos que

⋅+

+⋅==∑

=

23sen

23sen

2)13(sen

2sen

2)1(sen

2sen3

41sen

1

31 x

nxxn

x

xnnx

kxSn

k

(20)

Vale lembrar que a soma ∑=

=n

kkxS

13sen também poderá ser calculada

observando-se as igualdades a seguir, isto é

Page 26: Eureka 2011

Sociedade Brasileira de Matemática

EUREKA! N°33, 2011

26

nxxxnxn

xnxxnxn

xxxx

xxxx

3sen2

3sen22

)12(3cos2

)12(3cos

)13sen(2

3sen22

)32(3cos2

)12(3cos

6sen2

3sen22

9cos2

15cos

3sen2

3sen22

3cos2

9cos

⋅−=−

−+

−⋅−=−

−−

⋅−=−

⋅−=−

cuja soma resultará em

∑=

⋅−=−+ n

kkxxxxn

13sen

23sen2

23cos

2)12(3cos .

23sen

2)1(3sen

23sen

3sen1 x

xnnx

kxn

k

+⋅=∴∑

=

, que é exatamente a expressão (19).

Para o cálculo de ∑=

=n

kkxS

1

32 cos elevando-se a expressão (14) ao cubo teremos

que ( ) ( )

83

2cos

3333

kkkkkk CCCCCCkx−−− +++

=

+=

43coscos3cos3 kxkxkx +

=⇒ (21)

+==∴ ∑ ∑∑

= ==

n

k

n

k

n

k

kxkxkxS1 11

32 3coscos3

41cos (22)

Utilizando-se a relação (4) e a parte real da relação (18) e substituindo-as em (22) tem-se que

⋅+

+

+⋅==∑

=

23sen

23sen

2)13(cos

2sen

2)1(cos

2sen3

41cos

1

32 x

nxxn

x

xnnx

kxSn

k

(23)

Page 27: Eureka 2011

Sociedade Brasileira de Matemática

EUREKA! N°33, 2011

27

Ressaltamos que a soma ∑=

n

k

kx1

3cos também pode ser calculada através das

fórmulas de Prostaférese, ou seja, fazendo

nxxxnxn

xnxxnxn

xxxx

xxxx

3cos2

3sen22

)12(3sen2

)12(3sen

)13cos(2

3sen22

)32(3sen2

)12(3sen

6cos2

3sen22

9sen2

15sen

3cos2

3sen22

3sen2

9sen

⋅=−

−+

−⋅=−

−−

⋅=−

⋅=−

e somando-se as linhas teremos uma “Soma Telescópica”, cujo valor será

∑=

⋅=−+ n

k

kxxxxn1

3cos2

3sen22

3sen2

)12(3sen

23sen

2)1(3cos

23sen

3cos1 x

xnnx

kxn

k

+⋅

=∴∑=

, que é exatamente a parte real da

expressão (18). PROBLEMA 4 (IMO-62): Aqui é proposto resolvermos a equação a seguir (observamos que o segundo problema resolvido trata desta questão de forma generalizada).

13cos2coscos 222 =++ xxx (A)

Notando que xx 2cos21

21cos2 += segue que

( )xxxxxx 6cos4cos2cos21

233cos2coscos 222 +++=++

Sendo xixZ sencos +=

xixZxixZxixZ

6sen6cos4sen4cos2sen2cos

6

4

2

+=⇒

+=⇒

+=⇒

Page 28: Eureka 2011

Sociedade Brasileira de Matemática

EUREKA! N°33, 2011

28

( )xxxix

ZZZZZZ

sen3sen4sen4cos

)1()1(

2

62642 +=

−−

=++⇒ (B)

Tomando-se a parte real de (B) tem-se que

( )x

xxxxxsen

3sen4cos6cos4cos2cos ⋅=++ (C)

Como xxxx 4cos.3sen)sen7(sen21

=− , então teremos que a equação (A)

reduz-se a 0sen7sen =+ xx .

03cos.4sen2 =⇒ xx

03cos04sen =∨=∴ xx Logo, a solução da equação proposta será dada pelo conjunto

2 1 2 12 4 6

k ( k ) ( k )S x x x x ,k π + π + π= ∈ = ∨ = ∨ = ∈

PROBLEMA 5: Determinaremos agora o valor das somas a) nxnxxx cos3cos32cos2cos ++++ … e b) sen 2sen2 3sen3 senx x x n nx+ + + +… Sejam nxnxxxS cos3cos32cos2cos1 ++++= … e

2 sen 2sen2 3sen3 senS x x x n nx= + + + +… ( ) ( ) ( )1 2 cos sen 2 cos2 sen2 cos senS iS x i x x i x n nx i nx⇒ + = + + + + + +…

Sendo cos senZ x i x= + nnZZZZiSS ++++=+⇒ …3221 32 . Multiplicando-

se ambos os termos por )1( Z− teremos

221

21

21

21

212

132

21

)(

)1(

)()1( −−

++

−−

−=

−−+++

=+ZZ

Z

ZZ

nZZ

nZZZZZiSSn

nnn…

( )1 2

2 2

2 1 2 1cos sencos sen 12 2

2 sen 4 sen2 2

n nn x i xnx i nx

S iS x xi i

+ + + + − ⇒ + = − (1)

Page 29: Eureka 2011

Sociedade Brasileira de Matemática

EUREKA! N°33, 2011

29

Observando-se que a parte real de (1) nos dará o valor de 1S e que a parte imaginária nos dará o valor de 2S tem-se, após alguma manipulação algébrica que

( ) ( )1

21

1 cos cos 1 1cos

4sen2

n

j

n nx n n xS j jx x=

+ − + −= =∑ , e

( ) ( )2

21

1 sen sen 1sen

4sen2

n

j

n nx n n xS j jx x=

+ − += =∑

PROBLEMAS PROPOSTOS: 1) (URSS) Calcule o valor das somas a) xnCxCx n

nn )1cos(2coscos 1 ++++ …

b) ( )1sen sen2 sen 1nn nx C x C n x+ + + +…

Obs: kn

nC

k

=

denota o binomial “n escolhe k”.

2) (URSS) Mostre que

21

122cos

126cos

124cos

122cos −=

+++

++

++

+ nn

nnnππππ … .

3) (URSS) Prove que

a) ( ) ( ) ( )

( )1sen sen

2 2sen sen sen 2 sensen

2

n n

n

+ α α ϕ+ ϕ+ ϕ+α + ϕ+ α + + ϕ+ α =

α…

b) ( ) ( ) ( )

( )1sen cos2 2cos cos cos 2 cos

sen2

n n

n

+ α α ϕ+ ϕ+ ϕ+α + ϕ+ α + + ϕ+ α =

α…

4) Calcule o valor da soma n

n

S2

4cos

24

2cos

24

cos2

πππ

+++= … .

5) Mostre que

Page 30: Eureka 2011

Sociedade Brasileira de Matemática

EUREKA! N°33, 2011

30

a) 1cos2

1cos)1cos(coscos2coscos1 2

122

+−+−+−

=++++++

ϕϕϕϕϕϕϕ

aaakakakaaa

kkk…

b) ( ) ( )

( ) ( ) ( )2 1

2

sen sen sen

sen sen 1 sen sen2 cos 1

k

k k

a h a kh

a kh a k h a ha a h

+ +

ϕ+ ϕ+ + + ϕ+ =

ϕ+ − ϕ+ + − ϕ− + ϕ − +

6) Mostre que 072 é o menor ângulo positivo que satisfaz simultaneamente às equações:

1 cos cos2 cos3 cos4 0sen sen2 sen3 sen4 0

x x x xx x x x

+ + + + = + + + =

7) (IME-92) Mostre que

( )2 1sen1 2cos cos2 cos

2 2sen2

n x

x x nx .x

+

+ + + + =…

REFERÊNCIAS [1] FADDEEV, D.; SOMINSKY, I. Problems in Higher Algebra, Moscou: Ed. MIR, 1968. [2] GREITZER, S.L. International Mathematical Olympiads 1959-1977, Fifth Printing, Washington D.C.: The Mathematical Association of America, 1978. [3] GUIDORIZZI, H.L. Um curso de cálculo – Vol. 1, 2a Edição, São Paulo: Ed. Livros Técnicos e Científicos, 1987. [4] IEZZI, G. Fundamentos de Matemática Elementar, Vol. 3 (Trigonometria), 6a Edição, São Paulo: Editora Moderna, 1985. [5] IEZZI, G. Fundamentos de Matemática Elementar, Vol. 6 (Complexos – Polinômios - Equações), 4a Edição, São Paulo: Editora Moderna, 1983. [6] LIDSKI, V. B.; OVSIANIKOV, L. V.; TULAIKOV, A. N.; SHABUNIN M. I. Problemas de Matematicas Elementales, Moscou: Ed. MIR, 1972. [7] MORGADO, A. C; WAGNER, E.; DO CARMO, M. P., Trigonometria e Números Complexos, 4a Edição, Rio de Janeiro: Publicação da Sociedade Brasileira de Matemática, 2001. [8] SHKLARSKY, D.O., CHENTZOV, N.N., YAGLOM, I.M. The USSR Olympiad Problem Book, New York, Dover Publications, Inc., 1994.

Page 31: Eureka 2011

Sociedade Brasileira de Matemática

EUREKA! N°33, 2011

31

UMA INTERESSANTE DEDUÇÃO PARA A FÓRMULA DE HERÃO

Flávio Antonio Alves, Amparo – SP ♦ Nível Intermediário Nesta nota sugerimos uma dedução para a fascinante fórmula de Herão por meio de aplicações dos números complexos à geometria. Sejam biaz +=1 e dicz +=2 dois números complexos não nulos e distintos. Vamos considerar o triângulo de vértices o, 1z e 2z (veja a figura abaixo).

o

z2

z1

Re

Im

Ө2

Ө1

A área S do triângulo acima é dada por:

( ) { }1 2 2 1 2 11 12 2

S z z sen Im z z= θ − θ = .

Vamos multiplicar essa expressão, membro a membro, por 2 e elevar ao quadrado

ambos os termos da igualdade. Assim,

{ } ( )( ) ( )

22 2 22 1 1 22 2 2 2 2

2 1 2 1 2 1 1 22

14 242

z z z zS Im z z z z z z z z

i

−= = = − −

Page 32: Eureka 2011

Sociedade Brasileira de Matemática

EUREKA! N°33, 2011

32

( )[ ] ( )[ ] ( )[ ]2112122112122

21122

12

2 22414

41 zzzzzzzzzzzzzzzzzz +++−=+−=

( )[ ] ( )[ ]221

221

221

2214

1 zzzzzzzz −−+−−−= .

Notemos que:

i) ( ) ( ) ( )212112212

212

21 zzzzzzzzzzzz −+−−+−=−−− ,

E, do mesmo modo, temos que:

ii) ( ) ( ) ( )212121212

212

21 zzzzzzzzzzzz −++−−+=−−+ .

Substituindo (i) e (ii) na expressão acima, vem:

( ) ( ) ( ) ( )212121212121122141 zzzzzzzzzzzzzzzz −++−−+−+−−+−=

Nesse caso, pondo-se ( )

22121 zzzz

p−++

= , onde p é o semi-perímetro,

concluímos que:

( ) ( ) ( ) ( )⇒−−−−= pzzpzpzpS 2222222414 2121

2

( ) ( ) ( ) ( )⇒−−−−= pzzpzpzpS 21212 44

( ) ( ) ( ) ( )⇒−−−−= pzzpzpzpS 21212

( ) ( ) ( ) ( )2121 zzpzpzppS −−−−= , que é a fórmula de Herão.

Page 33: Eureka 2011

Sociedade Brasileira de Matemática

EUREKA! N°33, 2011

33

RAÍZES DA UNIDADE Anderson Torres & Eduardo Tengan

♦ Nível Intermediário

Para θ∈ a Fórmula de Euler nos permite escrever cos senie i .⋅θ = θ + ⋅ θ Ela nos fornece uma maneira prática de multiplicar números complexos. Por exemplo, o Teorema de De Moivre, normalmente escrito ( )cos sen cos senni n i n ,θ + ⋅ θ = θ + ⋅ θ na notação exponencial fica bem mais

conciso: ( ) ( )n i nie e .θθ = Mas, e as raízes da unidade? Elas são os complexos que

zeram o polinômio ( ) 1nP z z .= − Por De Moivre, sabemos que 2k i nk e πζ = são

raízes deste polinômio (com 0 k n≤ < ), e, como são n no total, elas são todas as raízes. E assim temos o primeiro resultado do artigo:

( )0

1n k

k n

z z ,≤ <

− = − ζ∏

em que 2k i ne .πζ = Raízes da unidade têm um monte de aplicações. Uma das mais imediatas é simplificar contas com funções trigonométricas, usando estas fórmulas aqui:

cos sen2 2

i i i ie e e e;i

θ − θ θ − θ+ −θ = θ =

PROBLEMA 1: calcule a soma tenebrosa

0sen

k n

kn≤ <

π∑

SOLUÇÃO: Usando a nossa recente descoberta, esta soma se transforma numa progressão geométrica! Sendo i ne ,πζ = temos

( )1

0 0 0 0

12 2

k k kk

k n k n k n k n

ksenn i i

−−

≤ < ≤ < ≤ < ≤ <

π ζ − ζ = = ζ − ζ

∑ ∑ ∑ ∑

Page 34: Eureka 2011

Sociedade Brasileira de Matemática

EUREKA! N°33, 2011

34

( )1

10

11 1sen2 1 1

nn

k n

kn i

−≤ <

ζ −π ζ − = − ζ − ζ −

Talvez você deva estar pensando: “uma diferença de complexos dando um

real? Mas como??” Simples: 1 ,−ζ = ζ logo a soma acima é uma diferença de conjugados dividida por 2i. É por isso que o resultado é real...

1 2 1 2

1 1 2 1 20

1 2 22 1 1 2k n i

ksen i cotgn n

− −≤ <

π − ζ + ζ π= + = ⋅ = ζ − ζ − ζ + ζ

Agora, uma aplicação da fatoração de 1nz − : PROBLEMA 2: Prove que, para todo inteiro positivo n existem polinômios

[ ]n nf ,g x∈ tais que

( )( ) ( )( )2 21 1 2nr

n nf x x g x x+ + + =

SOLUÇÃO: Primeiro, testar alguns casos pequenos: n = 1

( )( ) ( )( )2 21 11 1 2f x x g x x+ + + =

Para eliminar 1g , podemos aplicar x = i, o que nos dá

( )( ) ( )( )

21 1 2

21 21

f i i f i ii

+ = ⇔ = = −+

Podemos tomar ( )1f x x.= − Mas e quanto a ( )1g x ? Calma, coisas são feitas para funcionar! Veja que

( )( ) ( )2 212 1 2 1f x x x x− + = + +

tem i com zero, e automaticamente –i (conjugados, a-há!). Portanto o polinômio acima é múltiplo de 2 1x + e basta efetuar a divisão com Briot-Ruffini para achar

1g .

Page 35: Eureka 2011

Sociedade Brasileira de Matemática

EUREKA! N°33, 2011

35

Para o caso geral, vamos considerar os zeros de 2 1n

x .+ Mas os zeros de 12

22

111

nn

n

xxx

+

−+ =

− são justamente as raízes 12n+ -ésimas da unidade que não são

raízes 2n -ésimas da unidade. Logo, se escolhermos 12 2nie+πζ = uma raiz 12n+ -ésima

primitiva da unidade (isto é, que não é raiz t-ésima da unidade para nenhum t menor que 12n+ ), temos

( )( )

2

1 2 11 mod2

1n

n

k

kk

x x≤ ≤ +≡

+ = − ζ∏

Escrevendo x = –1,

( ) ( )( )

( )( )

2

1 2 1 1 2 1 1 mod2 1 mod2

1 1 1 2 1n

n n

k k

k kk k≤ ≤ + ≤ ≤ +≡ ≡

− + = − − ζ ⇔ = − − ζ∏ ∏

Basta demonstrar que cada 1 k+ ζ “é múltiplo” de 1+ ζ . Moleza:

( )( )2 3 2 11 1 1k k k... − −+ ζ = + ζ − ζ + ζ − ζ + − ζ + ζ

Portanto, podemos escolher nf tal que ( )( )22 1

n

nf x x− + admite raízes k ,kζ

ímpar. Portanto, é divisível por 2 1n

x ,+ o que acaba a demonstração. Agora, um problema de Geometria: PROBLEMA 3: ABCDE é um pentágono cíclico de circuncentro O. Os ângulos internos do pentágono são 70 120 120 130 100A , B , C , D , E .∠ = ° ∠ = ° ∠ = ° ∠ = ° ∠ = ° Demonstre que as diagonais BD e CE encontram-se em um ponto pertencente à reta AO. SOLUÇÃO: Como em qualquer problema de geometria, um bom arrastão para começar. Inicialmente, vamos ligar o centro aos vértices do pentágono. Esta é a melhor maneira de aproveitar a conciclicidade dos pontos. Assim sendo, 80 40 80 20 140AOB , BOC , COD , DOE , EOA .∠ = ° ∠ = ° ∠ = ° ∠ = ° ∠ = °

Mas ( )80 40 20 140 20MDC , , , = e portanto os vértices do pentágono estão entre os

vértices de um 18-ágono regular (afinal, 360 1820

= )! Agora, vamos colocar as

coisas nos eixos: inicialmente, 0 1O ,A= = (podemos fazer isto por homotetia: se

Page 36: Eureka 2011

Sociedade Brasileira de Matemática

EUREKA! N°33, 2011

36

1OA ,≠ aplicamos uma homotetia de centro O e razão 1 OA ). Seja 2 18ie πω = uma raiz 18-ésima (primitiva, por sinal) da unidade. Com isto, os vértices estão determinados. Vamos usar minúsculas para os números complexos associados aos pontos.

4 6 10 111a ,b ,c ,d ,e= =ω =ω =ω =ω Temos que provar que AO,BD,CE são concorrentes. Dada a escolha esperta que fizemos, basta demonstrar que as retas BD e CE se intersectam em um ponto real puro. Ou, em outras palavras, que se z é o complexo comum a BD e CE então z z= . Bem, para calcular equações de retas, vamos a uma técnica, ou melhor, um teorema, bastante útil (e que fica como exercício para o leitor, haha!): Dados os complexos p, q do círculo unitário, a reta pq tem equação dada por

z pqz p q+ = + Temos então:

AO : z z

BD : z bd z b d

CE : z cez c e

=

+ = +

+ = +

que equivale a

14 4 1 10

17 6 11

AO : z z

BD : z z

CE : z z

=

+ω =ω +ωω

+ω =ω +ω

Basta provar que 4 10 6 11

14 171 1AO BD : z ; AO CE : zω +ω ω +ω

∩ = ∩ =+ω +ω

Antes de começar a calculeira, vamos estudar algumas propriedades interessantes de .ω Bem, sabemos que ele é zero do polinômio 18 1x ,− e 218 2 3 .= ⋅ A ideia será fatorar este polinômio até a exaustão... ( )( )18 9 91 1 1x x x .− = − + Como ω é raiz 18-

ésima primitiva da unidade, o primeiro fator não contém ω como raiz. Assim

sendo, vamos pensar no outro fator: ( ) ( )( )39 3 3 6 31 1 1 1x x x x x .+ = + = + − + Pode-

Page 37: Eureka 2011

Sociedade Brasileira de Matemática

EUREKA! N°33, 2011

37

se demonstrar (mas não será necessário) que este último fator é irredutível. Então 6 3 1 0,ω −ω + = e de quebra 9 1.ω = −

Depois dessa volta toda, vamos ao que interessa: comparar as duas expressões de z:

( )( ) ( )( )( )( ) ( )( )

4 10 6 11

14 17

4 10 17 6 11 14

4 1 8 6 2 5

4 1 12 9 6 2 11 7

4 1 3 6 2 2 7

4 1 3 3 7

4 1 7

1 11 1

1 1

1 1 1

ω +ω ω +ω=

+ω +ωω +ω +ω = ω +ω +ω

ω −ω −ω = ω −ω −ω

ω −ω − ω + ω = ω −ω − ω + ω

ω −ω + ω − = ω −ω + ω + ω

ω −ω + ω − = ω − + ω

ω −ω = ω3 6 1

0 0ω − = ω

=

E fim! Outra aplicação interessante das raízes da unidade é como “marcadores”. Veja este problema: PROBLEMA 4: Determine uma fórmula fechada para

3 k

nk

SOLUÇÃO: Bem, alguém aí conhece algo parecido? Que tal o Binômio de Newton?

( )3

1 nk

k

nz z

k

= +

Agora, já tem alguma ideia do que se pode fazer? Temos que filtrar os múltiplos de 3 desta expansão, e nada melhor que usar uma raiz cúbica da unidade 2 3ie .πω= Substituindo z por 1, ω e 2 ,ω temos

Page 38: Eureka 2011

Sociedade Brasileira de Matemática

EUREKA! N°33, 2011

38

( )

( ) ( ) ( ) ( )

( )

2 2

2 2

1 1

1 1 2 1 1

1

n

k

nn nk k k nk k

nkk

nk

n nk k

nk

= +

ω = +ω ⇒ +ω +ω = + +ω + +ω

ω = +ω

∑ ∑

Agora, se k é múltiplo de 23 1 3k k, ;+ω +ω = caso contrário, temos uma progressão

geométrica de razão 1k ,ω ≠ e portanto 3

2 11 01

kk k

k .ω −+ω +ω = =

ω −

Ou seja, matamos todos os não múltiplos de 3!

( ) ( ) ( ) ( ) ( )2 2

3

3 2 1 1 2 2 2 12

n nn nn n nn n n

k

nk

− ω + ω= + + ω + + ω = + −ω + −ω = + −

( )

3

22 2 13

3

nn

k

ncosnk

π + − =

Esta última técnica tem um nome chique: multisecção. Vamos usá-la em um problema de, adivinha só, Combinatória Enumerativa! PROBLEMA 5: (IMO 1995, Canadá) Seja p um primo ímpar, e seja { }1 2 3 2S , , ... p .= Determine o total de subconjuntos A S∈ que satisfazem as condições a seguir:

• A p;=

• x A

p x.∈∑

SOLUÇÃO: Este foi o problema 6 da Olimpíada Internacional de 1995, em Montreal, Canadá. Ela foi tida como uma das mais interessantes pela riqueza de problemas “legais e divertidos” daquele ano, algo comparável apenas à IMO da Argentina, que aconteceria dois anos depois. A solução aqui apresentada é uma pequena modificação daquela dada por Nikolai Nikolov, ganhador de um Special Prize (prêmio especial, dado pela originalidade).

Page 39: Eureka 2011

Sociedade Brasileira de Matemática

EUREKA! N°33, 2011

39

Vamos pensar em uma raiz p-ésima da unidade, primitiva por sinal: 2 i pe .πε = Veja que kω= ε também é uma raiz p-ésima da unidade, para { }1 2 3 1k , , ,..., p .∈ − Excluímos o 1 propositalmente, pois ele não terá propriedades tão interessantes quanto as outras raízes (logo verás o porquê). Os complexos { } ( ){ }10 1 2 1 21 k pp k k, , ,..., , , ,..., −−ω ω ω ω = ε ε ε são raízes p-ésimas da

unidade. Elas são distintas: de fato, se ik jkε = ε para 0 i j p,≤ ≤ < temos

( )0 1j ie k e p j i k− = = ⇔ − e, como ( )0 0k p, p j i j i .< < − ⇔ − =

Agora vamos ao bom e velho polinômio ( ) ( ) ( )0 1 11p j j

j p j pf z z z z .

≤ ≤ − ≤ ≤= − = −ε = −ω∏ ∏

Pensando em Séries Formais, conseguimos trabalhar com este polinômio os elementos de 1 a p. Como podemos “alcançar” 2p? Oras, eleva ao quadrado!

( )( ) ( ) ( ) ( ) ( ) ( )22

0 1 0 1 1 1 2

1p j j j j

j p j p j p p j p

f z z z z z z≤ ≤ − ≤ ≤ − ≤ ≤ + ≤ ≤

= − = −ω ⋅ −ω = −ω ⋅ −ω =∏ ∏ ∏ ∏

( )1 2

j

j p

z≤ ≤

= −ω∏

Vamos abrir ( )( )2f z : ( )( )2 2 2 1 2

0 1 2 2 1 2p p p

p p pf z a a z a z ... a z ... a z a z−−= + + + + + + +

Agora, vamos observar como o pa é produzido de uma maneira combinatória. Primeiramente, escolhemos arbitrariamente p fatores, e coletamos o termo z deles; isto nos dará o expoente 2p. Já dos outros p fatores, escolhemos o termo ( )j .−ω O

resultado será então

( )( ) ( )1 2

1 2 0 11 2

p

p

jj j rp r

r pj j ... j p

a ... c≤ ≤ −≤ ≤ < < ≤

= −ω −ω −ω =− ω∑∏

em que rc é o total de p-tuplas 1 2 pj j ... j< < < tais que

( )1 2 mod pj j ... j r p .+ + + ≡ A nossa tarefa é achar 0c !

Mas ( )( )2 2 2 1 2p ppf z z z a .= − + ⇒ = − Assim,

10 1 2 1 2p

pc c c ... c −−+ ω+ ω+ + ω =

Em outras palavras, ω é zero do polinômio

Page 40: Eureka 2011

Sociedade Brasileira de Matemática

EUREKA! N°33, 2011

40

( ) ( ) 2 10 1 2 12 p

pg z c c z c z ... c −−= − + + + + ω

Lembre-se que todo o raciocínio usado até aqui foi puramente combinatório, e é válido para qualquer ω que seja raiz p-ésima da unidade (exceto o 1). Logo, todas as raízes p-ésimas primitivas da unidade são raízes de g. Mas g tem grau p – 1, portanto:

( ) ( ) ( )2 11 1 1

1p

p p

f zg z c c z z ... z

z−

− −= ⋅ = + + + +−

Igualando os coeficientes, 0 1 2 12 pc c c ... c .−− = = = =

Mas 0 1 2 1

2p

pc c c ... c .

p−

+ + + + =

Contagem dupla: cada p-subconjunto de S é

contado em exatamente um dos ic , justamente aquele correspondente à soma de seus elementos módulo p. Resolvendo as equações acima, concluímos que

0

212 2

pc

pp

= + −

E fim! Bem, que tal uns exercícios? EXERCÍCIOS PROPOSTOS: 1) Determine o valor numérico da série

1 1n j n

jcosn≥ ≤ ≤

π∑∏

2) Sejam x, y, z, A, B, C reais tais que A B C+ +π

é inteiro.

Defina ( ) ( ) ( )r r rrK x sen rA y sen rB z sen rC .= + +

Prove que se 1 2 0K K= = então 0nK = para todo 0n .> 3) Fixe um dos vértices de um n-ágono regular inscrito numa circunferência de raio 1, e considere os segmentos que ligam este vértice a todos os outros. Prove que o produto das medidas de todos estes n – 1 segmentos é n.

Page 41: Eureka 2011

Sociedade Brasileira de Matemática

EUREKA! N°33, 2011

41

4) Calcule 2 4 87 7 7

sen sen sen .π π π+ +

Dica: sejam 2 2 4 3 5 67

ie , p ,q .

πζ = = ζ + ζ + ζ = ζ + ζ + ζ O que queremos é calcular a

parte real de p. Calcule p + q e p · q e seja feliz! 5) Se P ,Q , R , S são polinômios tais que

( ) ( ) ( ) ( ) ( )5 5 2 5 4 3 2 1P x xQ x x R x x x x x S x ,+ + = + + + + prove que ( )1 0P .=

6) Fórmula de Multisecção: Sendo ( ) 2

0 1 2n

np x a a x a x ... a x ,= + + + + e l ,m ,∈

com 0 l m,≤ ≤ temos( )

( )

0

mod

lk kk m

kk l m

pa

m

−≤ ≤

ω ω=∑∑ em que

2ime .

πω =

7) Mostre que ( )2

02

2 2k n

n ncos k cos cosk≤ ≤

θ θ θ =

8) (Irlanda) Sabe-se que a, b, c são complexos tais que as raízes da equação

3 2 0x ax bx c+ + + = têm módulo 1. Prove que as raízes de 3 2 0x a x b x c+ + + = também têm módulo 1. 9) Seja ( )4962 3 4 2 1984

0 1 2 19841 x x x x a a x a x ... a x .+ + + + = + + + +

• Determine MDC ( )3 8 1983a ,a ,...,a

• Prove que 340 34799210 10a< <

10) Determine todos os polinômios P tais que ( ) ( ) ( )2 1P x P x P x .= −

11) Determine o número de polinômios de grau 5 com coeficientes entre 1 e 9 inclusive e que sejam divisíveis por 2 1x x .− +

12) Prove que o número 30

2 12

2 1k

k n

nk≤ ≤

+ +

∑ não é múltiplo de 5 para qualquer

0n .≥

Page 42: Eureka 2011

Sociedade Brasileira de Matemática

EUREKA! N°33, 2011

42

COMO É QUE FAZ? Resolvermos aqui, a pedidos, três problemas propostos na seção “Olimpíadas ao redor do

mundo”.

1) (Problema 109 – Suíça, 2000, proposto na Eureka! 11) Seja ( )q n a soma dos

algarismos de n. Calcule ( )( )( )20002000q q q (Proposto por Cícero Soares Furtado,

de Reriutaba – CE).

SOLUÇÃO: Como 2000 2000 60002000 2 10 ,= ⋅ sua representação decimal é a representação decimal de 20002 seguida de 6000 zeros, e logo

( ) ( )2000 20002000 2 .q q= Como ( )6673 2000 2001 3 6672 10,2 2 2 10 ,< < = < donde 20002

tem no máximo 667 dígitos. Como cada dígito é no máximo 9, ( ) ( )2000 20002000 2 9 667 6003.q q= ≤ ⋅ =

Portanto, ( )( )20002000 6 9 9 9 33,q q ≤ + + + = e logo ( )( )( )20002000 3 9 12.q q q ≤ + =

Por outro lado, como n e q(n) sempre deixam o mesmo resto na divisão por 9, o resto da divisão de ( )( )( ) ( )( )( )2000 20002000 2000q q q q q q= por 9 é igual ao resto

da divisão de 20002 por 9. Mas, como 62 64= deixa resto 1 quando dividido por 9,

( )3332000 6 333 2 6 22 2 2 2 4⋅ += = ⋅ = quando dividido por 9. Como

( )( )( )20002000 12q q q ≤ e 4 + 9 = 13 > 12, concluímos que necessariamente

( )( )( )20002000 4.q q q =

2) (Problema 110 – Grécia, 2000, proposto na Eureka! 11) Determine os números primo p para os quais o número 2 3 41 p p p p+ + + + é um quadrado perfeito. (proposto por Cícero Soares Furtado, de Rariutaba – CE).

Vamos encontrar todos os naturais n tais que 2 3 41 n n n n+ + + + é quadrado

perfeito. Note que 2 2

2 4 3 4 3 21 5 1 12 4 2 4

n n nn n n n n n n+ + = + + + + > + + + +

para

Page 43: Eureka 2011

Sociedade Brasileira de Matemática

EUREKA! N°33, 2011

43

todo 3n > (pois 2 3 0

4 2 4n n

− − > para todo n > 3). Por outro lado, para todo

,n∈2 2

2 4 3 4 3 2 1.4 4n nn n n n n n n + = + + < + + + +

Como, para todo ,k∈ temos 2

2nk n≤ + ou 2 1,

2nk n +

≥ + se n > 3 temos

2 4 3 2 1k n n n n< + + + + ou 2 4 3 2 1k n n n n> + + + + Assim, basta olhar os casos { }0,1, 2,3 .n ∈ Para n = 0, 4 3 2 21 1 1 .n n n n+ + + + = = Para n = 1,

4 3 2 1 5,n n n n+ + + + = que não é quadrado perfeito. Para n = 2, 4 3 2 1 31,n n n n+ + + + = que não é quadrado perfeito, e, para n = 3, 4 3 2 21 121 11 .n n n n+ + + + = = Assim, o único primo p tal que 2 3 41 p p p p+ + + +

é quadrado perfeito é p = 3.

3) (Problema 188 – Rússia, 2002, proposto na Eureka! 15) No intervalo ( )2 22 ,3n n

são escolhidos 2 12 1n− + números ímpares. Mostre que podemos encontrar entre estes números dois números tais que o quadrado de cada um deles não é divisível pelo outro. (Proposto por Anderson Torres, de Santana de Parnaíba – SP).

SOLUÇÃO: Se x y< são ímpares e y divide 2x , então 2

.2

y xy −

Em particular,

2

,2

y x y− ≥

donde 2 ,y x y− ≥ e logo ( )21 2 1 ,y y y x− = − + > donde

1.y x> + Assim, se 2 12 2

0 1 22 ... 3n

n nx x x −< < < < < são os números em questão,

temos 1 1,j jx x+ > + para todo 0,j ≥ e logo 2 , 0.njx j j> + ∀ ≥

Em particular, 2 12 1

22 2 ,n

n nx −−> + donde ( )2 1

22 2 12

3 2 2 ,nn n nx −

−> > + e logo 2 13 2 2 ,n n n−> + mas isso é falso para todo 1n ≥ (para n = 1, 3 < 4, para

2, 9 4 8n = < + e, para 2 13,3 2n nn −≥ < : com efeito, 3 527 3 2 32= < = e, se ( )2 1 12 1 1 2 1 2 1 2 13 2 ,3 3 3 3 2 4 2 2 2 ),nn n n n n n n + −− + − − +< = ⋅ < ⋅ < ⋅ = = absurdo.

4) (Problema 113 – Polônia, 2000, proposto na Eureka! 11) Uma sequência 1 2,...,p p de números primos satisfaz à seguinte condição: para 3, nn p≥ é o maior divisor

Page 44: Eureka 2011

Sociedade Brasileira de Matemática

EUREKA! N°33, 2011

44

primo de 1 2 2000.n np p− −+ + Mostre que a sequência ( )np é limitada. (Proposto por Anderson Torres, de Santana de Parnaíba – SP). SOLUÇÃO: Vamos mostrar a seguinte afirmação, que implica o resultado: Para todo 0k ≥ existe j com 1 40j≤ ≤ tal que

( ) ( )1 12max , max , 400003k j k j k kp p p p+ + + +≤ + (de fato, a afirmação implica que

existe 0n ∈ tal que 160000,np ≤ para todo 0 ;n n≥ note que ( )2 1max , 1000, 1).n n np p p n+ +≤ + ∀ ≥

Suponhamos inicialmente que, para todo r, com 0 35, k rr p +≤ ≤ é um primo ímpar.

Então, para todo r, com 1 2 20002 36, .2

k r k rk r

p pr p + − + −+

+ +≤ ≤ ≤ Definindo

0 ,kq p= 1 1kq p += e 1 2 20002

j jj

q qq − −+ +

= para 2 36,j≤ ≤ temos ,k r rp q+ ≤ para

0 36.r≤ ≤ Se, para algum 36, ,k r rr p q+≤ ≠ tomando um tal r mínimo temos

1 2 2000 ,6

k r k rk r

p pp + − + −+

+ +≤ e a afirmação vale para j = r. Temos ainda que jq é

dado pela expressão 1 12 2 24000 4000 13 9 3 9 2

jk k k k

jp p p pq + ++ − = − + + ⋅ − +

2000 ,3

j para

0 42.j≤ ≤ Assim,

( ) ( ) ( )6

6 1 119 3 6 4000 6 6 4000 6000 9 6000 mod7 .2

j

j k k k k kq p p p p j p j+ + = + − + − + ⋅ − + ≡ +

Portanto, existe s com 0 6s≤ ≤ tal que 69 sq (e logo 6sq ) é múltiplo de 7. Se s = 0, a afirmação já vale para j = 1. Se tivéssemos k r rp q+ = para 0 36,r≤ ≤ tomamos s

com 1 6s≤ ≤ tal que 6sq é múltiplo de 7, e teríamos 66 ,

7s

k jqp + ≤ absurdo.

Se 2,kp = a afirmação já vale para j = 1. Finalmente, se 2k rp + = para algum r com 1 35,r≤ ≤ teremos 1 1 2002k r k rp p+ + + −≤ + e 2 2 1 1 4004,k kp p+ + + −≤ + mas um desses números ( )1 1 1, +2002 e 4004k r k r k rp p p+ − + − + − + é múltiplo de 3, logo a afirmação vale para j = r + 2.

Page 45: Eureka 2011

Sociedade Brasileira de Matemática

EUREKA! N°33, 2011

45

SOLUÇÕES DE PROBLEMAS PROPOSTOS Publicamos aqui algumas das respostas enviadas por nossos leitores.

131) a) Considere o seguinte jogo: no início um jogador A entrega um número

2k ≥ ao jogador B . Quando A entrega um número 2m ≥ a B, B pode devolver m – 1 ou 1m + a A. Quando A recebe um número 2n ≥ deve, se n for ímpar

devolver 3n a B; se n for par mas não múltiplo de 4, pode devolver 2n ou 3n a B, e,

se n for múltiplo de 4, pode devolver ,4 2n n ou 3n a B. Qualquer jogador ganha o

jogo se devolver 1 ao adversário. Caso algum jogador devolva ao adversário um número maior que 1000k, o jogo empata. Determine, para cada valor de 2k ≥ , se algum dos jogadores tem estratégia vencedora, e, nesses casos, qual deles. b) Resolva o item anterior supondo que A, ao receber um número 2,n ≥ deve

devolver 3n a B se n for ímpar, deve devolver 2n a B se n for par mas não múltiplo

de 4 e deve devolver 4n a B se n for múltiplo de 4.

SOLUÇÃO DE JOSÉ DE ALMEIDA PANTERA (RIO DE JANEIRO – RJ) a) Naturalmente A perde se entregar o número 2k = ao jogador B, pois B poderá devolver 1 imediatamente a A.

Vamos mostrar que A ganha se entregar a B um número de k da forma 2 4 13

n⋅ + ou

da forma 4 1,3

n − para algum 1n ≥ , e se 2k > não for de nenhuma dessas formas

nenhum dos jogadores tem estratégia vencedora.

Para isso, note que, se A entrega 12 4 133

⋅ += a B, B pode devolver 2 ou 4 a A, e,

em qualquer caso, A pode devolver 1 a B e ganhar o jogo. Se A entrega 1 14 15

3

+ −=

a B, B pode devolver 4 ou 6 a A. Se devolve 4, A pode devolver 1 a B e ganhar. Se devolve 6, A pode devolver 3 a B, e ganhar a seguir, como vimos antes.

Page 46: Eureka 2011

Sociedade Brasileira de Matemática

EUREKA! N°33, 2011

46

Em geral, podemos argumentar por indução: se A entrega 12 4 1

3

n+⋅ + a B, com

1,n ≥ B pode devolver 12 4 2

3

n+⋅ − a A, caso em que A pode devolver 14 13

n+ − a B,

ganhando o jogo, ou B pode devolver 12 4 4

3

n+⋅ + a A, caso em que A pode devolver

2 4 13

n⋅ + a B, ganhando o jogo. De modo similar, se A entrega 24 13

n+ − a B, com

1,n ≥ B pode devolver 24 43

n+ − a A, caso em que A pode devolver 14 13

n+ − a B,

ganhando o jogo, ou B pode devolver 24 23

n+ + a A , caso em que A pode devolver

12 4 13

n+⋅ + a B, ganahndo o jogo.

Notemos que A sempre pode no mínimo empatar o jogo se 3.k ≥ De fato, se em algum momento do jogo A entrega 3m ≥ a B, B devolve no mínimo m – 1, e A pode devolver o triplo, que é no mínimo ( )3 1 .m m− > Assim, A pode devolver números cada vez maiores, que em algum momento ultrapassarão 1000k, empatando o jogo. Veremos agora que B pode garantir o empate se A entrega um número que não é das formas descritas anteriormente. Mais precisamente, veremos que, se A envia um número que não pertence ao conjunto

12 4 1 4 1: , 0 , 1 ,3 3

n n

X n n+ ⋅ + −

= ≥ ∪ ≥

então B pode devolver um número a

partir do qual A não pode devolver nenhum número pertencente a X (note que 1 X∈ ). Temos { }1,3,5,11,21,... .X = Se A envia a B um número par m, B pode devolver m – 1 ou m + 1, que são ímpares, a A, que deve devolver o triplo a B. Como não é possível que ( )3 1m − e ( )3 1m + pertençam ambos a X, isso mostra nossa afirmação no caso m par. Se A envia a B um número ímpar m, que não pertence a X, não é difícil ver que { } { }1 2 , 4 ,m k k X k k X− ∉ ∈ ∪ ∈ ou

{ } { }1 2 , 4 , .m k k X k k X+ ∉ ∈ ∪ ∈ Isso implica a afirmação no caso m par. b) No meio do jogo, B só recebe um número par se A tiver acabado de dividir um número (necessariamente múltiplo de 8) por 4. E, se B recebe um ímpar, devolverá um par, o que forçará A a dividi-lo por 2 ou por 4. Assim, os números tendem a

Page 47: Eureka 2011

Sociedade Brasileira de Matemática

EUREKA! N°33, 2011

47

decrescer, e , nos casos que empatavam, B ganha o jogo. Por outro lado, A ganha o jogo, com a mesma estratégia, nos mesmos casos que no item A, pois nesses casos sempre devolve números ímpares. 132) a) Considere uma família ℑ de 2000 círculos de raio 1 no plano tal que dois círculos de ℑ nunca são tangentes e cada círculo de ℑ intersecta pelo menos dois outros círculos de ℑ . Determine o número mínimo possível de pontos do plano que pertencem a pelo menos dois círculos de ℑ . SOLUÇÃO DE ZOROASTRO AZAMBUJA NETO (RIO DE JANEIRO – RJ) Mostraremos que esse número mínimo é igual a 2000. Para isso, consideramos um triângulo equilátero de lado 3 e os seguintes quatro círculos: o círculo circunscrito ao triângulo e os três círculos que contêm o circuncentro do triângulo e dois de seus vértices. Esses quatro círculos têm raio 1, e cada um deles intersecta os outros três. Considerando 500 cópias disjuntas dessa configuração de círculos, obtemos 2000 círculos como no enunciado tais que há 2000 pontos que pertencem a pelo menos dois deles.

Para concluir, vamos mostrar que, numa configuração de n círculos de raio 1 no plano ( )2n ≥ em que cada círculo intersecta pelo menos outro círculo e não há dois círculos tangentes, há sempre pelo menos n pontos que pertencem a pelo menos dois dos círculos. Vamos mostrar, por indução em n, que, na situação acima, não apenas há pelo menos n pontos que pertencem a pelo menos dois dos círculos, mas também que existe uma função injetiva do conjunto dos n círculos no conjunto dos pontos que pertencem a pelo menos dois dos círculos tal que a imagem de cada círculo pertence a ele.

Page 48: Eureka 2011

Sociedade Brasileira de Matemática

EUREKA! N°33, 2011

48

Para isso, note que se n = 2 isso é claramente verdadeiro. Suponha agora que 2m ≥ e que isso vale para todo n com 2 ,n m≤ ≤ e considere uma configuração

de m + 1 círculos como antes. Suponha inicialmente que algum desses círculos, digamos 1,C intersecta só um dos outros círculos, digamos 2C , seja { }1 2 , .C C p q∩ = Temos dois casos: no primeiro, 2C só intersecta 1.C Então associamos p a 1,C q a

2C e usamos a hipótese de indução para os m – 1 círculos restantes. No segundo caso, 2C intersecta algum dos outros círculos. Então associamos P a 1C e usamos a hipótese de indução para os m círculos 2 3 1, ,..., .mC C C + Se, por outro lado, cada um desses m + 1 círculos intersecta pelo menos dois dos outros, temos de novo dois casos: Se há no total pelo menos m + 1 pontos que pertencem a pelo menos dois dos círculos, podemos separar um dos círculos, digamos 1,mC + e fixar uma injecção de { }1 2, ,..., mC C C no conjunto dos pontos que pertencem a pelo menos dois dos círculos ,jC j m≤ tal que a imagem de cada iC , que chamaremos de ,iP pertence a

.iC Para cada { }1,..., ,mX C C⊂ o conjunto dos pontos que pertencem a pelo menos

dois círculos de { }1mX C +∪ tem pelo menos 1X + elementos. Se algum ponto

{ }1,..., mP P P∉ pertence a 1mC + e a algum dos outros círculos, simplesmente estendemos a injeção associando P a 1mC + . Senão, construímos uma sequência de conjuntos 1 2, ,...A A do seguinte modo: { }1 1 .i mA i m P C += ≤ ∈ Se 1 2, ,..., rA A A já

estão definidos, se o conjunto (de pelo menos 1rA + ) pontos que pertencem a pelo

menos dois círculos de { } { }1 ,m j rC C j A+ ∪ ∈ está contido em { }1,..., ,mP P

definimos {1r r jA A j m P+ = ∪ ≤ pertence a pelo menos dois círculos de

{ } { }}1 , .m j rC C j A+ ∪ ∈ Note que 1 .r rA A+ > Em algum momento, haverá um

ponto P fora de { }1,..., mP P que pertence a pelo menos dois círculos de

{ }1 , ,m j rC C j A+ ∪ ∈ e logo a dois círculos jC e ,j

C com 1, , .r rj j A j A −∈ ∉

Podemos então alterar a injeção associando jC a P; como existe algum

1 1 2\r r rj A A− − −∈ tal que jP pertence a 1,

rjC

− associamos

1rjC

− a jP , e, em geral,

para cada s com 1 1,s r< ≤ − se já definimos 1\ ,s s sj A A −∈ existe 1 1 2\s s sj A A− − −∈

Page 49: Eureka 2011

Sociedade Brasileira de Matemática

EUREKA! N°33, 2011

49

tal que sj

P pertence a 1;

sjC

− associamos então

1sjC

−a .

sjP Fazemos isso até associar

1jC a

2.jP Como 1 1,j A∈ podemos associar 1mC + a

1,jP estendendo nossa injeção a

{ }1 2 1, ,..., , ,m mC C C C + o que prova nossa afirmação. Finalmente, suponhamos que há apenas m pontos que pertencem a pelo menos dois dos círculos. Observamos que, como os círculos têm raio 1, se um par de pontos está contido em dois círculos de família, não estará contido em nenhum outro círculo de família, e portanto, se um ponto pertence a 2r ≥ círculos da família, cada um desses r círculos intersecta os outros r – 1 em outros r – 1 pontos distintos, e distintos do ponto comum aos r círculos. Assim, cada um desses r círculos contém pelo menos r pontos que pertecem a pelo menos dois círculos da família. Podemos considerar uma injeção que leva { }1 2, ,..., mC C C no conjunto desses pontos, a qual será uma bijeção. Sendo iP a imagem de ,iC podemos considerar a

matriz ( ) ,1 1,1 ,ija i m j m≤ ≤ + ≤ ≤ onde 1ija = se jP pertence a iC e 0,ija = caso

contrário. Se, para ,j m≤ { } { }1j j i ijn i P C i a= ∈ = = e, para

{ } { }1, : 1 ,i j i iji m s j P C j a≤ + = ∈ = = temos, pelo que observamos acima,

, ,i is n i m≥ ∀ ≤ donde 1 1

,m m

i ii i

s n= =

≥∑ ∑ mas ( ){ }1

1 1, 1 ,

m m

i j iji j

s n i j a+

= =

= = =∑ ∑ e 1 0,ms + >

pois 1mC + intersecta outros círculos, absurdo. 133) Considere um n–ágono regular inscrito em um círculo unitário, fixe um vértice i e denote por dj a distância entre este vértice i e o vértice j. Prove que

( )1

2 2

0

5n

j nj ij

d F−

≠=

− =∏ onde 1 10, 1F F= = e 1 2,n n nF F F− −= se 2.n ≥

SOLUÇÃO DE ASDRUBAL PAFÚNCIO SANTOS (BOTUCATU – SP) Podemos supor sem perda de generalidade que 0i = e que o vértice j é 2 ,j i ne π para

0 1.j n≤ ≤ − Queremos então provar que 1

2

1

23 2cos .n

nj

j Fn

=

+ =

∏ π

Temos ( )( ) 222 2 2 2 21 1 1 2 2 2cos ;j i nj i n j i n j i n j i n je e e e e

n− − − = − − = − − = −

ππ π π π π

queremos provar portanto que

Page 50: Eureka 2011

Sociedade Brasileira de Matemática

EUREKA! N°33, 2011

50

1

2

1

23 2cos .n

nj

j Fn

=

+ =

∏ π Considere agora a sequência de polinômios

( )( ) 0n nf x

≥ dada por ( ) ( )0 12,f x f x x= = e ( ) ( ) ( )1 1 , 1.n n nf x xf x f x n+ −= − ∀ ≥

Temos, para todo 0n ≥ e todo ( ) ( ), 2cos 2cos .nf n∈ =θ θ θ De fato isso vale para n = 0 e n = 1 e, por indução,

( ) ( ) ( ) ( ) ( )( )1 12cos 2cos 2cos 2cos 4cos cos 2cos 1n n nf f f n n+ −= − = − − =θ θ θ θ θ θ θ

( )( )2cos 1 .n= + θ Além disso, para todo ( )1, nn f x≥ é um polinômio mônico de

grau n. Como as solução de ( )2cos 2n =θ são dadas por 2 , ,k kn

= ∈πθ temos

( )1

0

22 2cos , 1,n

nk

kf x x nn

=

− = − ∀ ≥

∏ π donde ( )1

1

222cos .2

nn

j

f xjxn x

=

− − = − ∏ π

O que queremos provar equivale a

( ) ( ) ( )( )1

1 2

1

3 22 11 3 2cos 3 2 ,3 2 5

nn n

n nj

fjF fn

−−

=

− − − = − − = = − − − − − ∏ π o que é

equivalente a ( ) ( ) 23 2 1 5 .nn nf F− = + − ⋅ Como

1 1 5 1 5 , 0,2 25

n n

nF n + − = − ∀ ≥

temos

( ) 2 3 5 3 52 1 5 .2 2

n nn

nF − + − −

+ − ⋅ = +

Por outro lado, a sequência

( )3n nx f= − satisfaz 0 12, 3x x= = − e 1 13 , 1,n n nx x x n+ −= − − ∀ ≥ e logo (usando o

fato de as raízes de 2 3 1 0x x+ + = serem 3 52

− ± ), 3 5 3 5 , 0,2 2

n n

nx n − + − −

= + ∀ ≥

o que prova a igualdade desejada. 136) Sejam R, 1 2,r r e 3r os raios dos círculos de centro ,O 1 2,O O e 3 ,O

respectivamente, conforme a figura abaixo. Prove que: 1 2 1 3 2 3R r r r r r r= + + .

Page 51: Eureka 2011

Sociedade Brasileira de Matemática

EUREKA! N°33, 2011

51

O1

O2 O3

R

O R2

SOLUÇÃO DE ANDERSON TORRES (SANTANA DE PARNAÍBA – SP) A boa e velha trigonometria...

O

TA

TB OC

OA

OB

r Y

X ra

A

B C Como sempre, , A B∠ = ∠ =α β e , + + = .C∠ = γ α β γ π Vamos calcular ar , para começar:

A A A A A A A AAO O T T O AO AO O T AO OT+ + = ⇔ + = −

: ; ; .

2 2

aA A

r rAO X AO AOY AOsen sen

∆ = ∆ =α α

Substituindo:

Page 52: Eureka 2011

Sociedade Brasileira de Matemática

EUREKA! N°33, 2011

52

12 ,

12 2 2

aa a

senr rr r r rsen sen sen

− + = − ⇔ = ⋅

+

α

α α α ou

12

12

asenr

r sen

−=

+

α

α

Como precisaremos de um “quadrado”, vamos aplicar um truque: a tangente do meio arco.

22 2

2

2

2 tan41

1 1 tan 1 2 tan tan 1 tan2 4 4 4 4 .

1 2 tan 1 2 tan tan 1 tan2 4 4 4 41

1 tan4

sen

sen

− − + − + −

= = = + + + + +

+

α

α α α α α

α α α α α

α

Com isto, já podemos substituir na igualdade que queremos demonstrar:

1 tan 1 tan 1 tan 1 tan 1 tan 1 tan4 4 4 4 4 4 1

1 tan 1 tan 1 tan 1 tan 1 tan 1 tan4 4 4 4 4 4

− − − − − −⋅ + ⋅ + ⋅ =

+ + + + + +

α β α γ β γ

α β α γ β γ

Para escrever menos, seja tan , tan , tan .4 4 4

a b c= = =α β γ Abrindo os

denominadores, ( )( )( ) ( )( )( ) ( )( )( ) ( )( )( )1 1 1 1 1 1 1 1 1 1 1 1a b c a b c a b c a b c− − + + − + − + + − − = + + +

( ) ( ) ( ) ( ) ( ) ( )3 3 1a b c ab ac bc abc a b c ab ac bc abc− + + − + + + = + + + + + + +

( ) ( ) ( )1 0a b c ab ac bc abc− + + − + + + = . Mas isto é fácil?

tan tan4 4 41 tan tan

4 4 4 4 1 tan tan4 4 4

+ + = = + + = = − +

α β γπ α β γ

α β γ

Page 53: Eureka 2011

Sociedade Brasileira de Matemática

EUREKA! N°33, 2011

53

tan tan4 4tan

4 1 tan tan tan tan tan tan tan tan4 4 4 4 4 4 4 4

tan tan 1 tan tan tan tan tan tan4 4 4 2 4 4 4 41 tan

4 1 tan tan4 4

++

− + + −= =

+ − − −−

β γα

β γ α β γ α β γ

β γ α β α γ β γα

β γ

( )1 ,a b c abc ab ac bc⇒ + + − = − + + como esperado.

137) Seja A um conjunto de quinze pontos de 2 tal que a distância de cada ponto à origem é positiva e menor do que 1 e que quaisquer dois deles nunca sejam colineares com a origem. Mostre que existe um triângulo com dois vértices em A e

um na origem cuja área é menor que 1 .4

SOLUÇÃO DE ITAMAR SALES DE OLIVEIRA FILHO (CEDRO – CE)

1

– 1 1 0

x

y 3α

2α1α

15α

– 1

Distribuímos aleatoriamente os 15 pontos. Como a distância à origem é sempre menor do que 1, com certeza todos esses pontos são interiores à circunferência de raio 1 e centro na origem.

Page 54: Eureka 2011

Sociedade Brasileira de Matemática

EUREKA! N°33, 2011

54

Pelo fato de não existerem dois colineares com o centro, temos os 15 ângulos representados na figura ( )1 2 15, ,..., .α α α Obviamente:

1 2 15, ,..., 360 .= °α α α Vamos provar que existe pelo menos um ângulo menor do que ou igual a 24° . Para isso, suponha o contrário, ou seja, 24,n >α para todo n Então:

1 2 15... 15 24 360 absurdo.+ + + > × ° = °→α α α Então realmente existe pelo menos um ângulo 24 .≤ °α Suponha .i XOY= ∠α Olhando para o triângulo XOY:

Área 11 sen .2

XOY OX OY= ⋅ ⋅ ⋅ α Contudo, e OX OY são menores do que 1 e

sen sen24 sen30 ,i ≤ ° < °α substituindo: 1 1 1 11 1 sen30 .2 2 2 4

A A A< ⋅ ⋅ ⋅ °→ < ⋅ → <

Então existe um triângulo como no enunciado cuja área é menor do que 1 .4

138) Calcule o máximo divisor comum entre todos os números da forma ,⋅ ⋅x y z onde ( ), ,x y z percorre todas as soluções inteiras da equação 2 2 2x y z+ = com

0.⋅ ⋅ ≠x y z SOLUÇÃO DE MARCÍLIO MIRANDA DE CARVALHO (TERESINA – PI) Seja d mdc entre todos os inteiros da forma x y z⋅ ⋅ onde (x, y, z) percorre todas as soluções inteiras da equação 2 2 2x y z+ = com 0.⋅ ⋅ ≠x y z Note que (3, 4, 5) é solução, logo temos que 60.d ≤ AFIRMAÇÃO 1: Se uma tripla ( ), ,x y z é solução da equação 2 2 2x y z+ = então x y z⋅ ⋅ é múltiplo de 3. PROVA: Suponhamos, por absurdo, que x e y não são múltiplos de 3. Então

( )2 2 2 2 mod3z x y= + ≡ , absurdo!. Logo x ou y tem que ser múltiplo de 3. Assim temos que x y z⋅ ⋅ é múltiplo de 3. AFIRMAÇÃO 2: Se uma tripla ( , , )x y z é solução da equação 2 2 2x y z+ = então x y z⋅ ⋅ é múltiplo de 5.

Page 55: Eureka 2011

Sociedade Brasileira de Matemática

EUREKA! N°33, 2011

55

PROVA: Suponhamos, por absurdo, que x e y não são múltiplos de 5. Então 2 2 0x y+ ≡ ou 2 ou 3 (mod 5). No primeiro caso temos que z é múltiplo de 5,

portanto x y z⋅ ⋅ é múltiplo de 5. No segundo e terceiro casos temos um absurdo, pois um número quadrado perfeito só pode deixar restos 0, 1 ou 4 módulo 5. Assim, temos que se ( ), ,x y z é solução da equação 2 2 2x y z+ = então x y z⋅ ⋅ é múltiplo de 5. AFIRMAÇÃO 3: Se uma tripla ( ), ,x y z é solução da equação 2 2 2x y z+ = então x y z⋅ ⋅ é múltiplo de 4. PROVA: Suponhamos, por absurdo, que x e y são ímpares. Então

( )2 2 2 mod 4 ,x y+ ≡ absurdo. Portanto x ou y tem que ser par. Se x for par, mas não

for múltiplo de 4, então ( )2 2 24 mod8 4x x y≡ ⇒ + ≡ , 5 ou 0 (mod 8). No primeiro e terceiro casos temos que y é par, portanto x y z⋅ ⋅ é múltiplo de 4. No segundo caso temos um absurdo, pois um número quadrado perfeito só pode deixar restos 0, 1 ou 4 módulo 8. E se y for par é análogo. Assim, temos que x y z⋅ ⋅ é múltiplo de 4. Portanto d é múltiplo de 3 4 5 60,⋅ ⋅ = logo d = 60. 139) Determine todos os inteiros positivos x, y, z satisfazendo 3 3 2x y z− = , onde y é primo, z não é divisível por 3 e z não é divisível por y. SOLUÇÃO DE ADRIANO CARNEIRO TAVARES (CAUCAIA – CE) Suponha que exista uma solução. Então 2 3 3 2 2 2( ) ( ) ( )(( ) 3 )z x y x y x xy y x y x y xy= − = − + + = − − + (I) Como z não é divisível por 3 e nem por y, e y é um número primo, teremos por (I) mdc( , ) 1x y = e mdc( , 3) 1.x y− = Então mdc 2 2( , ) mdc (3 , ) 1x xy y x y xy x y+ + − = − = (II) Agora (I) e (II) implicam que 2 2 2 2 , e x y m x xy y n z mn− = + + = = , para certos inteiros positivos m e n. Temos 2 2 2 2 24 4 4 4 (2 ) 3 .n x xy y x y y= + + = + + Então 23 (2 2 )(2 2 ).y n x y n x y= + + − − Sendo y um primo, então existem três possibilidades: a) 22 2 3 ,2 2 1n x y y n x y+ + = − − =

Page 56: Eureka 2011

Sociedade Brasileira de Matemática

EUREKA! N°33, 2011

56

b) 2 2 3 ,2 2n x y y n x y y+ + = − − = c) 22 2 , 2 2 3n x y y n x y+ + = − − = Em (a), após a subtração das equações temos:

2 23 1 2(2 ) 2(2 3 ).y x y m y− = + = + Daí, 2 2 21 3 6 3 0 (mod 3).m y y m+ = − − ≡ Por outro lado, temos sempre 2 1 1 ou 2 (mod 3).m + ≡ Nós chegamos a uma contradição. Em (b), subtraindo as equações chegamos x = 0, o que é absurdo! Subtraindo as equações em (c), chegamos em 2 23 2(2 ) 2(2 3 ),y x y m y− = + = + que pode ser escrito assim:

2 2( 3) 4 12,y m− − = ou seja, ( 3 2 )( 3 2 ) 12.y m y m− + − − = Da equação chegamos a y = 7 e m = 1, pois devemos ter 3 2 6y m− + = e

3 2 2y m− − = . Segue que x = y + m2 = 8 e z = 13 2mn m x xy y= + + .

Veja que de fato 3 3 28 7 13 .− = Esta é a única solução. 140) Mostre que 2903 803 464 261n n n n− − + é divisível por 1897, para todo .n∈ SOLUÇÃO DE MARCELO RIBEIRO DE SOUZA (RIO DE JANEIRO – RJ) LEMA: Sejam 1 2,p p dois números inteiros primos entre si. Então, se 1p a e

2 ,p a ter-se-á 1 2 .p p a DEMONSTRAÇÃO: 1 1.p a k a kp⇔∃ ∈ = No entanto, deve-se ter 2 1,p a kp= ora,

como ( )1 2, 1,p p = conclui-se que 2 1 1 2.kp k k k p⇔∃ ∈ = Finalmente,

1 1 2 1 2 .a k p p p p a= ⇒ Note-se, inicialmente, que 1897 7 271.= × Escreva-se, então:

( ) ( ) ( ) ( ) ( )2903 464 803 261 78 78 10 10 0 mod271 ,n n n nn n n n n− − + ≡ − − − − − + − ≡ ∀ ∈ (i)

( )2903 464 803 261 5 2 5 2 0 mod7 ,n n n n n n n n n− − + ≡ − − + ≡ ∀ ∈ (ii) Assim, temos, pelo Lema, que ( )2903 464 803 261 0 mod1897 ,n n n n n− − + ≡ ∀ ∈ , como se quis demonstrar.

Page 57: Eureka 2011

Sociedade Brasileira de Matemática

EUREKA! N°33, 2011

57

141) Dado { }0,1,2,3,4,5,6,7,8,9 ,∈a seja ≠ ∅X um conjunto finito de inteiros positivos, tal que nenhum dos seus elementos possui o algarismo a em sua

representação decimal. Prove que 1 80.∈

<∑n X n

SOLUÇÃO DE FABRÍCIO VASCONCELLOS PUPPI (SÃO PAULO – SP) Por um simples raciocínio combinatório, nota-se que a quantidade de inteiros positivos com k algarismos que não apresentam algum dígito a em sua representação decimal é 18 9 ,k−⋅ caso 0, e 9 ,ka ≠ caso a = 0. Seja max( )N X= e d o número de dígitos de N. Seja S(T) a operação definida sobre um subconjunto finito T qualquer de *, tal que:

( ) 1n T

S Tn∈

=∑

Pela definição de S, como cada elemento de T tem contribuição positiva no valor da soma que caracteriza a operação, claramente S é monótona em relação ao seu argumento, de tal modo que ( )( ) .T Q S T S Q⊆ ⇒ ≤ Assim, para

{ } { }1,2,3,..., ,...,10 1 não é dígito de , .dX N n a n X X= − ∩ ⊆ Para todo inteiro

positivo n de k dígitos, 1 110 1 1 10 ,k kn n− −≥ ∴ ≤ sendo que a desigualdade estrita vale para todos os n de k dígitos exceto para um deles. Considerando inicialmente o caso em que 0,a ≠ tem-se:

( ) ( ) ( )1

1 1 2 1 11 1 2 1 1 1

1

1 1 1 1 8 98 9 8 9 ... 8 910 10 10 10

kdd

d kkn X n

−− − −

− − − −=∈

⋅ < ⋅ ⋅ + ⋅ ⋅ + + ⋅ ⋅ = ∑ ∑

( ) ( )1

11

9810

kd

kk

S X S X−

−=

∴ ≤ < ∑

Como a somatória à direita é uma série geométrica, trivialmente tem-se que:

( ) ( )( ) ( )1 9 /10

8 80 1 9 10 801 9 10

ddS X

− < ⋅ = ⋅ − < −

Page 58: Eureka 2011

Sociedade Brasileira de Matemática

EUREKA! N°33, 2011

58

Para o caso a = 0 é necessário um refinamento de análise, pois o uso de processo idêntico ao acima só permitiria afirmar que ( ) 90.S X < Pra este caso, separou-se cada conjunto dos números de k algarismos em dois subconjuntos disjuntos: um dos 14 9k−⋅ inteiros que satisfazem 1 110 5 10k kn− −≤ < ⋅ e outro dos 15 9k−⋅ inteiros que satisfazem 15 10 10 .k kn−⋅ ≤ < Assim, por um raciocionio análogo ao do caso anterior:

( ) ( )1 1 1

1 1 11 1 1

9 9 94 5 510 5 10 10

k k kd d d

k k kk k k

S X S X− − −

− − −= = =

≤ < + =⋅∑ ∑ ∑

( ) ( )( ) ( )1 9 10

5 50 1 9 10 50 801 9 10

ddS X

− ∴ < ⋅ = ⋅ − < < −.

Agradecemos o envio de soluções e a colaboração de: Adriano Carneiro Tavares (Caucaia – CE) Prob. 140 Anderson Torres (Santana de Parnaíba – SP) Prob. 133, 137, 138, 139, 140, 141 Douglas Oliveira de Lima (Brasília – DF) Prob. 140 Fabrício Vasconcellos Puppy (São Paulo – SP) Prob. 137, 138, 140. Flávio Antonio Alves (Amparo – SP) Prob. 136 Itamar Sales de Oliveira Fiolho (Cedro – CE) Prob. 136 Jean Pierre Youyoute (Rio de Janeiro – RJ) Prob. 138 Lucas Alves, Douglas Oliveira de Lima, Danillo Leal, Gustavo Campelo, Júlio Castro (Brasília – DF)

Prob. 136

Lucas Colucci Prob. 138 e 140 Marcelo Ribeiro de Souza (Rio de Janeiro – RJ) Prob. 136 Marcílio Miranda de Carvalho (Teresina – PI) Prob. 140 Marcos Martinelli (Brasília – DF) Prob. 133 e 136 Matheus Henrique Alves Moura (Fortaleza – CE) Prob. 136 Renato Carneiro (Belo Horizonte – MG) Prob. 140 Continuamos aguardando soluções para os problemas 134 e 135.

Page 59: Eureka 2011

Sociedade Brasileira de Matemática

EUREKA! N°33, 2011

59

PROBLEMAS PROPOSTOS Convidamos o leitor a enviar soluções dos problemas propostos e sugestões de novos

problemas para próximos números. 142) Seja { }4,8,9,16,25,27,36,64,...A = o conjunto das potências não triviais

(números da forma ,ba com 2,a ≥ 2b ≥ naturais). Prove que, para todo natural 1,n ≥ existe um natural k tal que todos os termos da progressão aritmética

,2 ,3 ,...,k k k nk pertencem a A. 143) Determime todas as funções , , :f g h → tais que

( ) ( ) ( )3 3 , , .f xy g x y h x y x y= + + + ∀ ∈

144) Seja 1x ≥ um número racional tal que existe uma constante 0c ≠ e uma sequência ( ) 1n na

≥ de inteiros tal que ( )lim 0.n

nncx a

→∞− = Prove que x é inteiro.

145) Encontre todos os números racionais p, q, r de modo que

2 3 cos cos cos 17 7 7

p q r+ + =π π π .

146) Determine todos os subconjuntos não-vazios A, B, C de de modo que: a) .A B B C C A∩ = ∩ = ∩ =∅ b) .A B C∪ ∪ = c) para quaisquer ,a A b B∈ ∈ e ,c C∈ temos: , e .a c A b c B a b C+ ∈ + ∈ + ∈

147) Demonstre que ( )( )

( )2

0

1 1 1,

22 14 2

k nn

kn

ksen

n

=

− − −= +

+ −

∑ πpara todo inteiro 2.n ≥

148) Sejam m e n inteiros positivos. Calcule 1

0.

n

k

n km

=

149) a) Deseja-se organizar um torneio de futebol com n times ( )2n ≥ em que cada time joga uma vez contra cada um dos outros, dividido em um certo número de rodadas. Em cada rodada cada time joga no máximo uma partida.

Page 60: Eureka 2011

Sociedade Brasileira de Matemática

EUREKA! N°33, 2011

60

Prove que, se n é ímpar, é possível organizar um tal torneio com n rodadas e, se n é par, é possível organizar um tal torneio com n – 1 rodadas. b) Uma matriz n n× é preenchida com elementos do conjunto

{ }1,2,3,...,2 1 .S n= − Sabe-se que, para todo { }1,2,..., ,i n∈ a i-ésima linha e a i-ésima coluna contêm juntas todos os elementos de S. Quais os possíveis valores de n? 150) Sejam a, b e c números reais tais que ( ) ( ) ( )3 3 3 9.a b b c c a− + − + − =

Prove que ( ) ( ) ( )

32 2 2

1 1 1 3.a b a b c a

+ + ≥− − −

Problema 142 adaptado de um problema proposto por Anderson Torres (Santana de Parnaíba – SP); 143 proposto por Anderson Torres (Santana de Parnaíba – SP); 144, 145 e 146 propostos por Carlos da Silva Ramos (Belém – PA); 147 e 148 propostos por Marcos Martinelli; 149 adapatado de um problema proposto por Anderson Torres (Santana de Parnaíba – SP); 150 adaptado de um problema proposto por Adriano Carneiro (Caucaia – CE).

Page 61: Eureka 2011

Sociedade Brasileira de Matemática

EUREKA! N°33, 2011

61

AGENDA OLÍMPICA

XXXIII OLIMPÍADA BRASILEIRA DE MATEMÁTICA

NÍVEIS 1, 2 e 3 Primeira Fase – sábado, 18 de junho de 2011

Segunda Fase – sábado, 3 de setembro de 2011 Terceira Fase – sábado, 15 de outubro de 2011 (níveis 1, 2 e 3)

domingo, 16 de outubro de 2011 (níveis 2 e 3 - segundo dia de prova)

NÍVEL UNIVERSITÁRIO Primeira Fase – sábado, 3 de setembro de 2011

Segunda Fase – sábado, 15 e domingo, 16 de outubro de 2011

IV ROMANIAN MASTER OF MATHEMATICS (RMM) 23 a 28 de fevereiro de 2011(Bucareste, Romênia)

ASIAN PACIFIC MATH OLYMPIAD (APMO)

12 de março de 2011

XVII OLIMPÍADA DE MAIO 7 de maio de 2011

XXII OLIMPÍADA DE MATEMÁTICA DO CONE SUL

14 a 20 de agosto de 2011(La Paz, Bolívia)

LII OLIMPÍADA INTERNACIONAL DE MATEMÁTICA 13 a 24 de julho de 2011(Amsterdam, Holanda)

I OLIMPÍADA DE MATEMÁTICA DA LUSOFONIA

20 a 31 de julho de 2011(Coimbra, Portugal)

XVII OLIMPÍADA INTERNACIONAL DE MATEMÁTICA UNIVERSITÁRIA (IMC) 24 a 30 de julho de 2011(Blagoevgrad, Bulgária)

XXV OLIMPÍADA IBEROAMERICANA DE MATEMÁTICA

23 de setembro a 1 de outubro de 2011(São José, Costa Rica)

II COMPETIÇÃO IBEROAMERICANA INTERUNIVERSITÁRIA DE MATEMÁTICA 2 a 8 de outubro de 2011(Quito, Equador)

XIII OLIMPÍADA IBEROAMERICANA DE MATEMÁTICA UNIVERSITÁRIA

Page 62: Eureka 2011

Sociedade Brasileira de Matemática

EUREKA! N°33, 2011

62

COORDENADORES REGIONAIS

Alberto Hassen Raad (UFJF) Juiz de Fora – MG Américo López Gálvez (USP) Ribeirão Preto – SP Antonio Carlos Nogueira (UFU) Uberlândia – MG Benedito Tadeu Vasconcelos Freire (UFRN) Natal – RN Bruno Holanda (CAEN – UFC) Fortaleza – CE Carmen Vieira Mathias (UNIFRA) Santa María – RS Claus Haetinger (UNIVATES) Lajeado – RS Cláudio de Lima Vidal (UNESP) S.J. do Rio Preto – SP Denice Fontana Nisxota Menegais (UNIPAMPA) Bagé – RS Disney Douglas Lima de Oliveira (UFAM) Manaus – AM Edson Roberto Abe (Colégio Objetivo de Campinas) Campinas – SP Edney Aparecido Santulo Jr. (UEM) Maringá – PR Fábio Brochero Martínez (UFMG) Belo Horizonte – MG Florêncio Ferreira Guimarães Filho (UFES) Vitória – ES Francinildo Nobre Ferreira (UFSJ) São João del Rei – MG Genildo Alves Marinho (Centro Educacional Leonardo Da Vinci) Taguatingua – DF Herivelto Martins (USP – São Carlos) São Carlos – SP Gilson Tumelero (UTFPR) Pato Branco – PR Ivanilde Fernandes Saad (UC. Dom Bosco) Campo Grande – MS João Benício de Melo Neto (UFPI) Teresina – PI João Francisco Melo Libonati (Grupo Educacional Ideal) Belém – PA Diogo Diniz (UFPB) Campina Grande – PB José Luiz Rosas Pinho (UFSC) Florianópolis – SC José Vieira Alves (UFPB) Campina Grande – PB José William Costa (Instituto Pueri Domus) Santo André – SP Krerley Oliveira (UFAL) Maceió – AL Licio Hernandes Bezerra (UFSC) Florianópolis – SC Luciano G. Monteiro de Castro (Sistema Elite de Ensino) Rio de Janeiro – RJ Luzinalva Miranda de Amorim (UFBA) Salvador – BA Marcelo Dias (Grupo Educacional Etapa) São Paulo – SP Marcelo Antonio dos Santos FACOS Osório – RS Marcelo Rufino de Oliveira (Grupo Educacional Ideal) Belém – PA Newman Simões (Cursinho CLQ Objetivo) Piracicaba – SP Nivaldo Costa Muniz (UFMA) São Luis – MA Osnel Broche Cristo (UFLA) Lavras – MG Uberlândio Batista Severo (UFPB) João Pessoa – PB Raul Cintra de Negreiros Ribeiro (Colégio Anglo) Atibaia – SP Reginaldo de Lima Pereira (Escola Técnica Federal de Roraima) Boa Vista – RR Reinaldo Gen Ichiro Arakaki (UNIFESP) SJ dos Campos – SP Ricardo Amorim (Centro Educacional Logos) Nova Iguaçu – RJ Ronaldo Alves Garcia (UFGO) Goiânia – GO Rogério da Silva Ignácio (Col. Aplic. da UFPE) Recife – PE Rosangela Ramon (UNOCHAPECÓ) Chapecó – SC Sérgio Cláudio Ramos (IM-UFRGS) Porto Alegre – RS Seme Gebara Neto (UFMG) Belo Horizonte – MG Tadeu Ferreira Gomes (UEBA) Juazeiro – BA Tomás Menéndez Rodrigues (U. Federal de Rondônia) Porto Velho – RO Valdenberg Araújo da Silva (U. Federal de Sergipe) São Cristóvão – SE Vânia Cristina Silva Rodrigues (U. Metodista de SP) S.B. do Campo – SP Wagner Pereira Lopes (CEFET – GO) Jataí – GO Wanderson Breder (CEFET – RJ) Nova Friburgo – RJ William Serafim dos Reis (UFT – TO) Arraias – TO

Page 63: Eureka 2011

CONTEÚDO XXXII OLIMPÍADA BRASILEIRA DE MATEMÁTICA Problemas e soluções da Primeira Fase

3

XXXII OLIMPÍADA BRASILEIRA DE MATEMÁTICA Problemas e soluções da Segunda Fase

15

XXXII OLIMPÍADA BRASILEIRA DE MATEMÁTICA Problemas e soluções da Terceira Fase

34

XXXII OLIMPÍADA BRASILEIRA DE MATEMÁTICA Problemas e soluções da Primeira Fase Nível Universitário

59

XXXII OLIMPÍADA BRASILEIRA DE MATEMÁTICA Problemas e soluções da Segunda Fase Nível Universitário

67

XXXII OLIMPÍADA BRASILEIRA DE MATEMÁTICA Premiados

78

AGENDA OLÍMPICA

85

COORDENADORES REGIONAIS

86

Page 64: Eureka 2011

Sociedade Brasileira de Matemática

EUREKA! N°34, 2011

2

Esta edição é dedicada à memória do professor Sergio Plaza Salinas da Universidad de Santiago de Chile, que colaborou como membro do comitê editorial da revista Eureka! desde 1998, e que nos deixou neste ano de 2011.

Os editores

Page 65: Eureka 2011

Sociedade Brasileira de Matemática

EUREKA! N°34, 2011

3

XXXII OLIMPÍADA BRASILEIRA DE MATEMÁTICA Problemas e soluções da Primeira Fase

PROBLEMAS – NÍVEL 1 1. Qual dos números a seguir não é múltiplo de 15? A) 135 B) 315 C) 555 D) 785 E) 915 2. Ana, Esmeralda e Lúcia têm, juntas, 33 reais. Ana e Esmeralda, juntas, têm 19 reais e Esmeralda e Lúcia, juntas, têm 21 reais. Quantos reais tem Esmeralda? A) 6 B) 7 C) 10 D) 12 E) 14 3. Aumentando 2% o valor um número inteiro positivo, obtemos o seu sucessor. Qual é a soma desses dois números? A) 43 B) 53 C) 97 D) 101 E) 115 4. Qual é o maior número de fichas que podemos colocar em um tabuleiro 5 5× , no máximo uma em cada casa, de modo que o número de fichas em cada linha e cada coluna seja múltiplo de 3? A) 6 B) 9 C) 12 D) 15 E) 24 5. Carlos tem 2010 blocos iguais de 10 cm de largura por 20 cm de comprimento e 1,5 cm de espessura e resolveu empilhá-los formando uma coluna de 20 cm de largura por 40 cm de comprimento, como na figura. Qual dos valores a seguir, em metros, é o mais próximo da altura dessa coluna?

A) 7 B) 7,5 C) 8 D) 8,5 E) 9 6. Qual das alternativas apresenta um divisor de 5 4 33 4 5⋅ ⋅ ? A) 42 B) 45 C) 52 D) 85 E) 105

7. Dividindo-se o número ( )244 por 44 obtemos o número: A) 2 B) 43 C) 44 D) 48 E) 412

Page 66: Eureka 2011

Sociedade Brasileira de Matemática

EUREKA! N°34, 2011

4

8. As quatro faces de um dado são triângulos equiláteros, numerados de 1 a 4, como no desenho. Colando-se dois dados iguais, fazemos coincidir duas faces, com o mesmo número ou não. Qual dos números a seguir não pode ser a soma dos números das faces visíveis? A) 12 B) 14 C) 17 D) 18 E) 19

9. Quantos divisores positivos de 120 são múltiplos de 6? A) 4 B) 5 C) 6 D) 8 E) 12 10. O desenho mostra dois quadrados de papel sobrepostos, um de lado 5 cm e outro de lado 6 cm. Qual é o perímetro da figura formada (linha grossa no contorno do desenho), em centímetros? A) 31 B) 34 C) 36 D) 38 E) 41

11. O horário indicado pelo relógio ao lado está correto. A partir desse momento, porém, o relógio começa a atrasar exatamente 5 minutos a cada hora real. Depois de quantos dias o relógio voltará a apresentar um horário correto? A) 1 B) 2 C) 4 D) 6 E) 12 12. No reticulado a seguir, pontos vizinhos na vertical ou na horizontal estão a 1 cm de distância.

1cm

1cm

Qual é a área da região sombreada? A) 7 B) 8 C) 8,5 D) 9 E) 9,5

Page 67: Eureka 2011

Sociedade Brasileira de Matemática

EUREKA! N°34, 2011

5

13. Um jornal publicou a tabela de um campeonato de futebol formado por quatro times, apresentando os gols marcados e os gols sofridos por cada time. Por uma falha de impressão, a tabela saiu com dois números borrados, conforme reprodução a seguir.

Sabe-se que o time Esmeralda FC sofreu dois gols a mais que o time EC Boleiros. Quantos gols sofreu o time Esmeralda FC? A) 2 B) 3 C) 4 D) 5 E) 6 14. Ana começou a descer uma escada no mesmo instante em que Beatriz começou

a subi-la. Ana tinha descido 34

da escada quando cruzou com Beatriz. No

momento em que Ana terminar de descer, que fração da escada Beatriz ainda terá que subir?

A) 14

B) 13

C) 1

12 D)

512

E) 23

15. Alguns números inteiros positivos, não necessariamente distintos, estão escritos na lousa. A soma deles é 83 e o produto é 1024. O menor número é igual a: A) 1 B) 2 C) 4 D) 8 E) 16 16. Numa sala do 6º ano, todos gostam de pelo menos uma das duas matérias:

Matemática ou Português. Sabe-se que 34

dos alunos gostam de Matemática e 57

dos alunos gostam de Português. A sala tem 56 alunos. Quantos alunos gostam dessas duas matérias ao mesmo tempo? A) 4 B) 8 C) 13 D) 24 E) 26

Gols marcados Gols sofridos Craques do Momento 8 4 Independentes 1 6 EC Boleiros 4 *** Esmeralda FC 5 ***

Page 68: Eureka 2011

Sociedade Brasileira de Matemática

EUREKA! N°34, 2011

6

17. O desenho representa um canto de um tabuleiro retangular convencional, formado por quadradinhos de lado 1 cm. Nesse tabuleiro, 17 quadradinhos são brancos. Qual é a área do tabuleiro, em centímetros quadrados? A) 29 B) 34 C) 35 D) 40 E) 150

18. A figura representa uma barra de chocolate que tem um amendoim apenas num pedaço. Elias e Fábio querem repartir o chocolate, mas nenhum deles gosta de amendoim. Então combinam dividir o chocolate quebrando-o ao longo das linhas verticais ou horizontais da barra, um depois do outro e retirando o pedaço escolhido, até que alguém tenha que ficar com o pedaço do amendoim. Por sorteio, coube a Elias começar a divisão, sendo proibido ficar com mais da metade do chocolate logo no começo. Qual deve ser a primeira divisão de Elias para garantir que Fábio fique com o amendoim ao final?

A) Escolher a primeira coluna à esquerda.

B) Escolher as duas primeiras colunas à esquerda.

C) Escolher a terceira linha, de cima para baixo.

D) Escolher as duas últimas linhas, de cima para baixo.

E) Qualquer uma, já que Fábio forçosamente ficará com o amendoim. 19. Quatro amigos, Arnaldo, Bernaldo, Cernaldo e Dernaldo estão jogando cartas. São 20 cartas diferentes, cada carta tem uma entre 4 cores (azul, amarelo, verde, vermelho) e um número de 1 a 5. Cada amigo recebe cinco cartas, de modo que todas as cartas são distribuídas. Eles fazem as seguintes afirmações: Arnaldo: “Eu tenho quatro cartas com o mesmo número.” Bernaldo: “Eu tenho as cinco cartas vermelhas.” Cernaldo: “As minhas cinco cartas são de cores que começam com a letra V.” Dernaldo: “Eu tenho três cartas de um número e duas cartas de outro número.” Sabe-se que somente uma das afirmações é falsa. Quem fez essa afirmação? A) Arnaldo B) Bernaldo C) Cernaldo D) Dernaldo E) Não é possível definir.

Page 69: Eureka 2011

Sociedade Brasileira de Matemática

EUREKA! N°34, 2011

7

20. A figura a seguir foi recortada em cartolina e depois dobrada para formar um icosaedro. As faces em branco foram numeradas de modo que ao redor de cada vértice (pontas do sólido) apareçam os números de 1 a 5. Qual número está na face com a interrogação?

ICOSAEDRO

A) 1 B) 2 C) 3 D) 4 E) 5

PROBLEMAS – NÍVEL 2

1. Veja o problema No. 6 do Nível 1. 2. Aumentando em 2% o valor do menor de dois números consecutivos, obtém-se o maior deles. Qual é a soma desses números? A) 43 B) 53 C) 97 D) 101 E) 115 3. Veja o problema No. 7 do Nível 1 4. Cecília pegou uma cartolina e recortou as 8 peças à direita, formadas por quadradinhos de mesmo tamanho.

De quantas maneiras diferentes ela pode escolher 3 dessas peças para montar o quadrado

33× à esquerda?

A) 3 B) 4 C) 5 D) 6 E) 7

5. Os números x e y são distintos e satisfazem y

yx

x 11−=− . Então xy é igual a

A) 4 B) 1 C) –1 D) –4 E) é preciso de mais dados.

Page 70: Eureka 2011

Sociedade Brasileira de Matemática

EUREKA! N°34, 2011

8

6. Sônia calculou a média aritmética de dois diferentes números de dois dígitos e obteve 98. Qual é a diferença entre esses números? A) 1 B) 2 C) 3 D) 4 E) um número maior que 4 7. Veja o problema No. 17 do Nível 1. 8. Quantos inteiros da lista 100, 101, 102, ..., 999 não possuem algarismos iguais a 2, 5, 7 ou 8? A) 160 B) 170 C) 180 D) 190 E) 200 9. No triângulo ABC, m(BÂC) = 140o. Sendo M o ponto médio de BC, N o ponto médio de AB e P o ponto sobre o lado AC tal que MP é perpendicular a AC, qual é a medida do ângulo PMN ˆ ? A) 40o B) 50o C) 70o D) 90o E) 100o 10. Veja o problema No. 4 do Nível 1

11. Para quantos inteiros n o número n

n−100

é também inteiro?

A) 1 B) 6 C) 10 D) 18 E) 100 12. Ana começou a descer uma escada de 24 degraus no mesmo instante em que

Beatriz começou a subi-la. Ana tinha descido 43 da escada quando cruzou com

Beatriz. No momento em que Ana terminar de descer, quantos degraus Beatriz ainda terá que subir? A) 2 B) 6 C) 8 D) 10 E) 16 13. Veja o problema 19 do Nível 1. 14. No desenho, o retângulo cinza tem seus vértices sobre os lados do triângulo equilátero de área 40 cm2. O menor lado do retângulo é um quarto do lado do triângulo. A área do retângulo em cm2 é:

Page 71: Eureka 2011

Sociedade Brasileira de Matemática

EUREKA! N°34, 2011

9

A) 5 B) 10 C) 15 D) 18 E) 22 15. Veja o problema No. 15 do Nível 1. 16. De quantas maneiras é possível desenhar a figura a seguir sem tirar o lápis do papel (ou qualquer outro utensílio, se você preferir!) começando de P e sem passar sobre o mesmo ponto mais de uma vez, com exceção do ponto comum aos três triângulos?

P

A) 48 B) 24 C) 16 D) 108 E) 27 17. Os pontos P, Q, R, S e T são vértices de um polígono regular. Os lados PQ e TS são prolongados até se encontrarem em X, como mostra a figura, e SXQ ˆ mede 140o. Quantos lados o polígono tem?

A) 9 B) 18 C) 24 D) 27 E) 40 18. Veja o Problema No. 20 do Nível 1. 19. O professor Piraldo tem dois relógios, ambos digitais de 24 horas. Nenhum dos dois funciona: um muda de horário com o dobro da velocidade normal e o outro vai

Page 72: Eureka 2011

Sociedade Brasileira de Matemática

EUREKA! N°34, 2011

10

de trás para frente, na velocidade normal. Ambos mostram corretamente 13:00. Qual é a hora certa na próxima vem em que os dois relógios mostrarem o mesmo horário? A) 05:00 B) 09:00 C) 13:00 D) 17:00 E) 21:00 20. Uma figura no formato de cruz, formada por quadrados de lado 1, está inscrita em um quadrado maior, cujos lados são paralelos aos lados do quadrado tracejado, cujos vértices são vértices da cruz. Qual é a área do quadrado maior?

A) 9 B)

549 C) 10 D)

881 E)

332

21. Quantos são os pares (x, y) de inteiros positivos tais que x2 – y2 = 22010? A) 1000 B) 1001 C) 1002 D) 1003 E) 1004 22. Quatro números inteiros positivos a < b < c < d são tais que o mdc entre quaisquer dois deles é maior do que 1, mas mdc(a, b, c, d) = 1. Qual é o menor valor possível para d? A) 10 B) 12 C) 15 D) 30 E) 105 23. Veja o problema No. 8 do Nível 1. 24. Na figura, BC = 2BH.

B

H30o

80o x

A C

A) 10o B) 15o C) 16o D) 20o E) 25o

Page 73: Eureka 2011

Sociedade Brasileira de Matemática

EUREKA! N°34, 2011

11

25. Os números a e b são reais não negativos tais que a3 + a < b – b3. Então A) b < a < 1 B) a = b = 1 C) a < 1 < b D) a < b < 1 E) 1 < a < b

PROBLEMAS – NÍVEL 3

1. Dividindo-se o número ( )244 por 44 obtemos o número: A) 2 B) 43 C) 44 D) 48 E) 412 2. Qual dos seguintes números é um divisor de 345 543 ⋅⋅ ? A) 42 B) 45 C) 52 D) 85 E) 105 3. Veja o Problema No. 8 do Nível 1. 4. Veja o Problema No. 14 do Nível 1. 5. Um quadrado PQRS tem lados medindo x. T é o ponto médio de QR e U é o pé da perpendicular a QS que passa por T. Qual é a medida de TU?

A) 2x B)

3x C)

2x D)

2 2x E)

4x

6. Os números x e y são distintos e satisfazem 1 1x yx y

− = − . Então xy é igual a

A) 4 B) 1 C) –1 D) –4 E) é preciso de mais dados 7. Considere todos os números de três algarismos distintos, cada um igual a 0, 1, 2, 3 ou 5. Quantos desses números são múltiplos de 6? A) 4 B) 7 C) 10 D) 15 E) 20

Page 74: Eureka 2011

Sociedade Brasileira de Matemática

EUREKA! N°34, 2011

12

8. O máximo divisor comum de todos os números que são o produto de cinco ímpares positivos consecutivos é A) 1 B) 3 C) 5 D) 15 E) 105 9. Veja o problema 17 do Nível 2. 10. Veja o problema 19 do Nível 1. 11. Esmeralda ia desenhar o gráfico de y = 2x + 6 mas trocou os eixos de lugar. Como fica o desenho dessa relação com os eixos trocados de lugar?

12. Qual das seguintes frações é mais próxima de 7 ?

A) 31

B) 52

C) 83

D) 135

E) 187

13. No triângulo ABC, m(BÂC) = 140º. Sendo M o ponto médio de BC, N o ponto médio de AB e P o ponto sobre o lado AC tal que MP é perpendicular a AC, qual é a medida do ângulo ˆNMP ? A) 40º B) 50º C) 70º D) 90º E) 100º 14. Veja o problema 16 do Nível 2. 15. Veja o problema No. 20 do Nível 1. 16. Os números a e b são reais não negativos tais que a3 + a < b – b3. Então A) b < a < 1 B) a = b = 1 C) a < 1 < b D) a < b < 1 E) 1 < a < b 17. Quantos são os pares (x, y) de inteiros positivos tais que x2 – y2 = 22010? A) 1000 B) 1001 C) 1002 D) 1003 E) 1004

Page 75: Eureka 2011

Sociedade Brasileira de Matemática

EUREKA! N°34, 2011

13

18. Veja o problema No. 8 do Nível 1. 19. Seja ABC um triângulo e X, Y e Z pontos sobre os lados BC, CA, AB tais que

2===ZABZ

YCAY

XBCX .

A razão entre as áreas do triângulo XYZ e do triângulo cujos lados são congruentes às medianas de ABC é: Obs.: as medianas de um triângulo são os segmentos que ligam os vértices do triângulo aos pontos médios dos lados opostos.

A) 23

B) 12

C) 49

D) 13

E) 14

20. Para cada subconjunto A de {1;2;3;4;5;6;7;8;9;10}, seja p(A) o produto de seus elementos. Por exemplo, p({1;2;4;5}) = 40 e p(A) = 10! = 1 2 3 10⋅ ⋅ ⋅…⋅ . Por convenção, adote ( ) 1.p ∅ = A soma de todos os 210 produtos p(A) é igual a: A) 211 B) 11! C) 1111 D) 211! E) 112! 21. Sendo n = 20102010 e log n é igual ao número m tal que 10m = n, então A) n! < nlog n < (log n)n

B) nlog n < n! < (log n)n

C) (log n)n < nlog n < n! D) (log n)n < n! < nlog n E) nlog n < (log n)n < n! 22. Quatro números inteiros positivos a < b < c < d são tais que o mdc entre quaisquer dois deles é maior do que 1, mas mdc(a, b, c, d) = 1. Qual é o menor valor possível para d? A) 10 B) 12 C) 15 D) 30 E) 105 23. Qual é o maior valor de xy2 se x e y são reais positivos cuja soma é 3? A) 3 B) 4 C) 5 D) 6 E) 7

A

B CX

Y

Z

Page 76: Eureka 2011

Sociedade Brasileira de Matemática

EUREKA! N°34, 2011

14

24. Um ponto P é escolhido ao acaso no interior de um quadrado QRST. Qual é a probabilidade do ângulo ˆRPQ ser agudo?

A) 34

B) 2 1− C) 12

D) 4π E) 1

25. Qual é o menor valor positivo de 21m2 – n2 para m e n inteiros positivos? A) 1 B) 2 C) 3 D) 5 E) 7

GABARITO

NÍVEL 1 (6º. ou 7º. Anos do Ensino Fundamental)

1) D 6) B 11) D 16) E 2) B 7) E 12) B 17) C 3) D 8) E 13) D 18) A 4) D 9) C 14) E 19) B 5) B 10) D 15) A 20) D

NÍVEL 2 (8º. ou 9º. Anos do Ensino Fundamental)

1) B 6) B 11) D 16) A 21) E 2) D 7) C 12) E 17) D 22) C 3) E 8) C 13) B 18) D 23) E 4) E 9) D 14) C 19) E 24) Anulada 5) C 10) D 15) A 20) B 25) D

NÍVEL 3 (Ensino Médio)

1) E 6) C 11) E 16) D 21) E 2) B 7) B 12) C 17) E 22) C 3) E 8) D 13) D 18) A 23) B 4) E 9) D 14) A 19) C 24) E 5) D 10) B 15) D 20) B 25) C

Page 77: Eureka 2011

Sociedade Brasileira de Matemática

EUREKA! N°34, 2011

15

XXXII OLIMPÍADA BRASILEIRA DE MATEMÁTICA Problemas e soluções da Segunda Fase

PROBLEMAS – NÍVEL 1 – PARTE A (Cada problema vale 5 pontos)

01. Uma jarra contém 14

de sua capacidade em água. Despejando um copo cheio de

água na jarra, o volume de água atinge 13

da sua capacidade. Quantos copos cheios

mais ainda serão necessários para acabar de encher a jarra?

02. Joãozinho tem que fazer uma multiplicação como lição de casa, mas a chuva molhou o caderno dele, borrando alguns algarismos, que estão representados por (cada algarismo borrado pode ser diferente dos outros).

1 × 2 3 4 4 2 +

0 1 0 0 2

Qual é a soma dos algarismos que foram borrados? 03. Soninha pintou as seis faces de um cubo da seguinte maneira: uma face preta e a face oposta vermelha, uma face amarela e a face oposta azul, uma face branca e a oposta verde. Ao olhar para o cubo, de modo a ver três faces, como na figura, e considerando apenas o conjunto das cores das três faces visíveis, de quantas maneiras diferentes pode ser visto esse cubo?

04. Esmeralda foi escrevendo os quadrados dos números inteiros positivos um em seguida ao outro formando o número 149162536... e parou quando chegou no centésimo algarismo. Qual foi o último algarismo que ela escreveu?

Page 78: Eureka 2011

Sociedade Brasileira de Matemática

EUREKA! N°34, 2011

16

05. Carlinhos escreve números inteiros positivos diferentes e menores do que 1000 em várias bolas e coloca-as numa caixa, de modo que Mariazinha possa pegar ao acaso duas dessas bolas. Quantas bolas no máximo Carlinhos irá colocar na caixa se os números das duas bolas deverão ter um divisor comum maior do que 1?

06. Num concurso com 10 questões, cada resposta correta valia 3 pontos, cada resposta errada valia 1 ponto negativo e cada questão não respondida valia 0 ponto. Não houve dois candidatos que apresentassem a mesma nota, feitas as correções. Quantos candidatos no máximo fizeram essa prova? PROBLEMAS – NÍVEL 1 – PARTE B (Cada problema vale 10 pontos)

PROBLEMA 1 Com cinco quadrados com lados de 27 cm, formamos uma sequência de figuras, das quais as quatro primeiras são:

a) Na 4ª figura, qual é a área do quadrado cinza? b) Na 5ª figura, qual é a área do quadrado cinza? PROBLEMA 2 Maria tem 90 cartões. Ela numerou os cartões de 10 a 99 numa das faces e, para cada número escrito, escreveu a soma dos seus algarismos na outra face. Por exemplo, o cartão de número 43 tem o número 7 escrito no verso. Em quais cartões um número de uma face é o dobro do número escrito na outra face?

Page 79: Eureka 2011

Sociedade Brasileira de Matemática

EUREKA! N°34, 2011

17

PROBLEMA 3 Fazendo três cortes num quadrado 3 3× e juntando as quatro partes resultantes a um quadrado 4 4× , obtemos um quadrado 5 5× , conforme indicado na figura. Os cortes devem ser paralelos aos lados dos quadrados e não pode haver sobreposição de figuras para a realização dos cortes.

a) Transforme um quadrado de lado 8 cm e um quadrado de lado 15 cm num único quadrado de lado 17 cm, fazendo quatro cortes apenas no quadrado de 8 cm. b) Qual é o menor número de cortes para transformar três quadrados, de áreas respectivamente iguais a 4 cm2, 9 cm2 e 36 cm2, num único quadrado? PROBLEMAS – NÍVEL 2 – PARTE A (Cada problema vale 5 pontos) 01. Seja N o menor número inteiro positivo que multiplicado por 33 resulta em um número cujos algarismos são todos iguais a 7. Determine a soma dos algarismos de N.

02. Na figura seguinte, os triângulos ABC e ABD são retângulos em A e D, respectivamente. Sabendo que AC = 15 cm, AD = 16 cm e BD = 12 cm, determine, em cm2, a área do triângulo ABE.

03. Sejam p, q números reais satisfazendo as relações 2p2 – 3p – 1 = 0, q2 + 3q – 2 =

0 e pq ≠ 1. Ache o valor de qppq 1++ .

A B

C

D

E

Page 80: Eureka 2011

Sociedade Brasileira de Matemática

EUREKA! N°34, 2011

18

04. Em uma cidade arbitrária o prefeito organizou uma rifa com bilhetes numerados de 100 a 999. O prêmio de cada bilhete é determinado pela soma dos algarismos do número do bilhete. Para que ninguém leve três prêmios iguais, estabeleceu-se que quem retirar três bilhetes com as três somas iguais tem direito a um superprêmio. Qual é o menor número de bilhetes que um cidadão deve comprar para ter a certeza de que vai receber um superprêmio?

05. Sejam r e s números inteiros. Sabe-se que a equação do segundo grau

x2 – (r + s)x + rs + 2010 = 0 tem as duas soluções inteiras. Quantos são os possíveis valores de |r – s|?

PROBLEMAS – NÍVEL 2 – PARTE B (Cada problema vale 10 pontos) PROBLEMA 1 Joãozinho deseja colorir um tabuleiro 2 × 2010 com duas cores A e B. Uma coloração é dita legal se não é possível encontrar um L-triminó, como na figura abaixo, com todos os seus quadradinhos de mesma cor. Determine o número de colorações legais.

L – Triminó Veja abaixo duas colorações que não são legais:

PROBLEMA 2 Determine todos os números primos m e n tais que 0 < m < n e os três números

2m + n, m + 2n e m + n – 18 sejam também primos.]

PROBLEMA 3 Chamaremos de imagem de um número natural de dois algarismos o número que se

Page 81: Eureka 2011

Sociedade Brasileira de Matemática

EUREKA! N°34, 2011

19

obtém trocando a ordem de seus algarismos. Por exemplo, a imagem de 34 é 43. Quais são os números de dois algarismos que somados com sua imagem resultam em um quadrado perfeito? PROBLEMA 4 As bissetrizes internas dos ângulos A e C do triângulo ABC cortam-se no ponto I. Sabe-se que AI = BC e que )ˆ(2)ˆ( CAImACIm = . Determine a medida do ângulo

CBA ˆ .

PROBLEMAS – NÍVEL 3 – PARTE A (Cada problema vale 4 pontos)

01. Seja N o menor número inteiro positivo que multiplicado por 33 resulta em um número cujos algarismos são todos iguais a 7. Determine a soma dos algarismos de N. 02. Sejam r e s números inteiros. Sabe-se que a equação do segundo grau

x2 – (r + s)x + rs + 2010 = 0

tem as duas soluções inteiras. Quantos são os possíveis valores de |r – s|? 03. Na figura a seguir, as três circunferências em traço contínuo são tangentes às retas r e s e a circunferência tracejada passa pelos pontos A, B, C e D. Além disso, a circunferência menor é tangente também a AD e a circunferência maior é também tangente a BC. Se os raios das circunferências externas ao quadrilátero ABCD são 8 e 18, calcule o raio R da circunferência inscrita em ABCD.

AB

CD

8

18

R

r

s

Page 82: Eureka 2011

Sociedade Brasileira de Matemática

EUREKA! N°34, 2011

20

04. Cada uma das oito casas de um retângulo de duas linhas e quatro colunas é pintada de uma entre três cores. Uma coluna é chamada de corte se as suas duas casas são da mesma cor. De quantas maneiras é possível pintar o retângulo de modo que haja exatamente um corte? 05. Calcule

( )( )( ) ( )( )( )( ) ( )13131155133111

1323216614412224242424

24242424

++++++++++++++++

……

PROBLEMAS – NÍVEL 3 – PARTE B (Cada problema vale 10 pontos)

PROBLEMA 1 As bissetrizes internas dos ângulos A e C do triângulo ABC cortam-se no ponto I. Sabe-se que AI = BC e que )ˆ(2)ˆ( CAImACIm = . Determine a medida do ângulo

CBA ˆ . PROBLEMA 2 Diamantino gosta de jogar futebol, mas se jogar dois dias seguidos ele fica com dores musculares. De quantas maneiras Diamantino pode escolher em quais de dez dias seguidos ele vai jogar bola sem ter dores musculares? Uma maneira é não jogar futebol em nenhum dos dias. PROBLEMA 3 Resolva o sistema

=+++=++

94677

xyzzxyzxyzyx

sendo x ≤ y ≤ z inteiros não negativos. PROBLEMA 4 Uma mesa de bilhar tem o formato de um quadrado ABCD. SuperPablo tem uma missão especial: ele deve dar uma tacada em uma bola de bilhar, inicialmente colocada no vértice A, de modo que, após bater exatamente 2010 vezes nos lados do quadrado, a bola chegue, pela primeira vez, a um vértice do quadrado.

Page 83: Eureka 2011

Sociedade Brasileira de Matemática

EUREKA! N°34, 2011

21

Quantos são os possíveis valores do ângulo formado pelo lado AB com a trajetória inicial da bola? Observação: ao bater nos lados do quadrado, a bola sofre reflexão perfeita, ou seja, o ângulo de incidência é igual ao ângulo de reflexão. Suponha também que a bola seja um ponto.

SOLUÇÕES NÍVEL 1 – SEGUNDA FASE – PARTE A

Problema 01 02 03 04 05 06 Resposta 8 60 8 9 499 38

01. Volume de um copo de água é igual a 1 1 4 3 13 4 12 12

−− = = do volume da jarra.

Falta encher 1 213 3

− = da jarra. Para isso são necessários

223 12 81 3

12

= × = copos.

02. Como o algarismo das unidades do produto é 2, o algarismo das unidades do multiplicando é 4. Assim, obtemos o algarismo da direita da 3ª linha do algoritmo e

também os dois últimos algarismos da 5ª linha, conforme figura à direita. Como o algarismo das dezenas do produto é 0, o algarismo da direita na 4ª linha do algoritmo deve ser 6. Logo o algarismo das dezenas do

multiplicador é 9, conforme figura à esquerda. Como o 2º algarismo à direita 5ª linha é 0, o algarismo das centenas

1 4 × 2 3 4 2 4 2 +

0 2 8 1 0 0 2

1 4 × 2 9 3 4 2 4 2 6 +

0 2 8 1 0 0 2

α α

Page 84: Eureka 2011

Sociedade Brasileira de Matemática

EUREKA! N°34, 2011

22

do multiplicando é 5. A partir do algoritmo completo, mostrado à direita, concluímos que a soma dos algarismos que foram borrados é 5 4 9 1 5 2 6 6 1 2 8 5 6 60+ + + + + + + + + + + + = 03. Cada 3 faces que podem ser vistas ao mesmo tempo compartilham um vértice. Como o cubo tem 8 vértices, o número de composições de cores percebidas visualmente é 8. 04. Os números 12, 22, 32 possuem um algarismo. Os números 42, 52, ..., 92 possuem dois algarismos. Os números 102, 112, ..., 312 possuem três algarismos. Assim, ao escrever o quadrado do número 31, o número de algarismos escritos é 1 3 2 6 3 22 81× + × + × = , faltando escrever 19 algarismos. Com os quadrados de 32, 33, 34 e 35, temos mais 4 4 16× = algarismos, faltando ainda escrever apenas três algarismos. Como o quadrado de 36 é 1296, concluímos que o último algarismo escrito foi o 9, o centésimo algarismo escrito por Esmeralda. 05. Não podemos colocar o número 1 em nenhuma bola, pois o MDC entre 1 e qualquer outro número é 1, assim temos 998 números disponíveis. Além disso, se forem usadas 500 bolas ou mais, haverá duas com números consecutivos, sempre primos entre si, então não podemos colocar mais que 499 bolas. Mas existe uma forma de colocar 499 bolas, usando os números pares de 2 a 998. 06. Quem acerta a questões e erra b obtém 3a – b pontos, com 10≤+ ba . Obtemos os números de 0 a –10 com a = 0, ao todo 11 inteiros. Obtemos os números de 1 a 30 usando os valores 0, 1 ou 2 para b, não obtendo apenas 3.9 – 2 = 25, 3.10 – 1 = 29 e 3.10 – 2 = 28, pois nesses casos ficamos com a + b > 10, ao todo 30 – 3 = 27 inteiros. Logo, o número máximo de candidatos nas condições apresentadas é 11 + 27 = 38. SOLUÇÕES NÍVEL 1 – SEGUNDA FASE – PARTE B SOLUÇÃO DO PROBLEMA 1: Em cada figura, a área do quadrado cinza é uma fração da área do quadrado original. Nas figuras apresentadas, a partir da segunda, as áreas são iguais,

5 1 4 × 2 9 3 1 5 4 2 4 6 2 6 + 1 0 2 8 1 5 0 6 0 2

Page 85: Eureka 2011

Sociedade Brasileira de Matemática

EUREKA! N°34, 2011

23

respectivamente, a

4 27 2794 4 27 279 94 4 4 27 27 649 9 9

× ×

× × ×

× × × × =

a) Na 4ª figura, a área do quadrado cinza é igual a 64, segundo os produtos acima. b) Na 5ª figura, admitindo que a obtenção do quadrado cinza seja feita da mesma

maneira, a sua área é igual a 49

da área do quadrado cinza da 4ª figura, ou seja, é

igual a 4 256649 9

× = cm2.

SOLUÇÃO DO PROBLEMA 2: Para um número cujo algarismo das dezenas é a e cujo algarismo das unidades é b, temos ( )10a b 2 a b+ = + ou ( )a b 2 10a b+ = + . A segunda equação não tem soluções positivas, e na primeira equação temos

( )10a b 2 a b 10a b 2a 2b 8a b+ = + ⇔ + = + ⇔ = . Necessariamente temos a = 1 e b = 8. De fato, no cartão de número 18 a soma dos algarismos é 9. SOLUÇÃO DO PROBLEMA 3: a) Bastam 4 cortes no quadrado de lado 8 cm, conforme ilustrado nos desenhos à direita.

Ou ainda, como a figura a seguir.

1º, 2º e 3º cortes 4º

corte

Page 86: Eureka 2011

Sociedade Brasileira de Matemática

EUREKA! N°34, 2011

24

1º, 2º e 3º cortes

4ºco

rte

b) Uma possibilidade (exemplo 1) é juntar ao quadrado maior pedaços dos quadrados menores, obtendo-se um quadrado de área 4 + 9 + 36 = 49 cm2. Para isso, dividimos o quadrado de lado 3 em três tiras 3 1× com dois cortes e o quadrado de lado 2 em duas tiras 2 1× com um corte, num total de 3 cortes, conforme desenho à esquerda. Menos que 3 cortes não formam peças que se encaixam na região sombreada.

(exemplo 1)

Outras maneiras (exemplos 2 e 3) demontar o quadrado também com três cortes são apresentadas ao lado.

(exemplo 2)

(exemplo 3)

Page 87: Eureka 2011

Sociedade Brasileira de Matemática

EUREKA! N°34, 2011

25

SOLUÇÕES NÍVEL 2 – SEGUNDA FASE – PARTE A

Problema 01 02 03 04 05

Resposta 25 75 1 53 8 01. O critério de divisibilidade por 11 nos diz que se o número 33N possui todos os seus algarismos iguais e é divisível por 11, então ele deve possuir um número par da algarismos. O critério de divisibilidade por 3 também nos diz que a soma dos algarismos deve ser múltipla de 3 e isso obriga que a quantidade de algarismos 7 seja divisível por 3. O menor número que cumpre essas condições é 777777, ou seja, N = 777777/33 = 23569. 02. Pelo teorema de Pitágoras, temos que 2 2 20AB AD BD= + = e que

2 2 25CB AC AB= + = . Os triângulos ABC e ADB são semelhantes pois os seus lados são proporcionais e consequentemente temos EAB EBA= e

90 90ACB EBA EAB CAE.= − = − = . Concluímos assim que E é o ponto médio de

CB e a área procurada é metade da área do triângulo CAB, ou seja, 15 20 754

.⋅=

03. Como p não pode ser zero, podemos dividir a primeira equação por 2p− e

obter 2

1 13 2.p p

+ − Isto nos diz que as raízes da primeira equação são os inversos

das raízes da segunda equação. Como 1pq .≠ , p é igual ao inverso da outra raíz da

segunda equação que é diferente de q, ou seja, 13

pq

=− −

pois a soma das raízes

da segunda equação é igual a –3. Substituindo na expressão procurada:

2 2

1 1 3 2 2 13 3 2

pq p q qq q q q q

+ + − − + += = = =

+ +

04. A soma dos dígitos dos bilhetes é no mínimo 1 e no máximo 27. Para as somas 1 e 27 existem apenas dois bilhetes, enquanto que para qualquer outro valor existem pelo menos três bilhetes. Então retirando ( )1 1 2 27 2 1 53+ + × − + = iremos

escolher pelo menos três números com mesma soma.

Page 88: Eureka 2011

Sociedade Brasileira de Matemática

EUREKA! N°34, 2011

26

05. Para que as soluções sejam inteiras, o discriminante da equação do segundo grau deve ser o quadrado de um inteiro positivo, digamos 2t . Assim

( )2 24 4 2010r s rs t+ − − × =

( )2 2 4 2010r s t− − = ×

( )( ) ( )( )2010

2 2r s t r s t− + − −

× =

Como os números ( )( )r s t− + e ( )( )r s t+ + possuem a mesma paridade e 2010 é

inteiro, concluímos que os termos no produto anterior são inteiros. A cada para de

divisores do tipo 2010d ,d

do número 2010, temos uma solução para t e r s−

na última equação. Como 2010 possui 16 divisores, o número de soluções é 8.

SOLUÇÕES NÍVEL 2 – SEGUNDA FASE – PARTE B SOLUÇÃO DO PROBLEMA 1: A pintura da primeira coluna 2 x 1 do tabuleiro limita o número de maneiras de pintarmos o restante do tabuleiro. Temos dois casos a considerar: Primeiro caso: As casas desta coluna são pintadas com a mesma cor. Necessariamente a próxima coluna terá ambas casa da cor oposta à aquela da primeira coluna e. Pela mesma razão, teremos que as cores das colunas do tabuleiro devem ser alternadas. Assim, neste caso, temos apenas 2 pinturas diferentes.

Figura 1 …

Segundo caso: As casas desta coluna são pintadas com cores diferentes. Necessariamente a próxima coluna é igual à primeira ou tem as cores opostas. O mesmo se passará com as próximas colunas. Como para cada coluna sempre

teremos duas escolhas a fazer, incluindo a coluna inicial, temos 22010pinturas

diferentes.

Page 89: Eureka 2011

Sociedade Brasileira de Matemática

EUREKA! N°34, 2011

27

Figura 2

O total de pinturas é: 20102 2+

SOLUÇÃO DO PROBLEMA 2: Como os primos 2m + n e , m + 2n são maiores que dois, temos que ambos são ímpares e consequentemente 2 2 3 3m n m n m n+ + + = + é um número par. Assim m n+ é par e 18m n+ − é um primo par, ou seja, dois. O único par de primos ( , )m n que cumpre 20m n+ = e satisfaz o enunciado é ( , ) (3,17)m n =

SOLUÇÃO DO PROBLEMA 3: A soma de um número de dois algarismos com a sua imagem é da forma

( )(10 ) (10 ) 11 ,a b b a a b+ + + = + onde a e b são seus algarismos. Se ( )11 a b+ é um quadrado perfeito, devemos ter outro fator primo 11 na soma a + b. Além disso, como a e b são menores que 10, concluímos que a + b é um múltiplo de 11 menor que 20 e maior que 0, ou seja, é igual à 11. Os pares de dígitos ( ),a b que

verificam 11a b+ = são: ( ) ( ) ( ) ( ) ( ) ( ) ( )2,9 , 3,8 , 4,7 , 5,6 , 6,5 , 7, 4 , 8,3 e ( )9, 2 .

Portanto, existem 8 números de dois algarismos que cumprem o enunciado.

PRIMEIRA SOLUÇÃO DO PROBLEMA 4:

Seja N o ponto de encontro da bissetriz do ângulo ACD∠ com o lado AB . Pelo caso . .A L A , os triângulos NCA e ADC são congruentes. Consequentemente NC AD BC= = .

Pelo teorema do ângulo externo, BNC NAC ACN NCB∠ = + = ∠ . portanto BN BC NC= = e BNC é equilátero. Daí 60, 80ABC BCA∠ = ∠ = e

40BAC∠ = SEGUNDA SOLUÇÃO DO PROBLEMA 4:

Lema 1: Se ABC é um triângulo com 2ACB BAC∠ = ∠ então 2( )a a b c+ = .

Page 90: Eureka 2011

Sociedade Brasileira de Matemática

EUREKA! N°34, 2011

28

Lema 2: A bissetriz do vértice C do triângulo ABC tem comprimento

2 cos2

ACBab

a b

+

Sejam BADα = ∠ e P o ponto de encontro da bissetriz do ângulo C∠ com o

lado AB . Pelo segundo lema temos 2 cos 2cb ab

a b a bα

=+ + e daí 2 cos 2c a α= .

Pelo lema 1 temos 2 2 2( ) 4 cos 2a a b c a α+ = = e daí

2(4cos 2 1)b a α= − . Como AD BC= temos que cos cosp a AD aα α− = = e daí

2 (cos 1)a b c a α+ + = + . Substituindo os valores encontrados anteriormente temos

2(4cos 2 1) 2 cos 2 2 (cos 1)1 2cos 4 1 2cos 2 2cos 2cos 4 cos 2 cos

1cos (1 2cos3 ) 0 cos32

a a a aα α αα α α

α α α

α α α

+ − + = + ⇒+ + + = + ⇒

+ = ⇒

− = ⇒ =

E consequentemente 3 60ABC α∠ = = . SOLUÇÕES NÍVEL 3 – SEGUNDA FASE – PARTE A Problema 01 02 03 04 05 Resposta 25 8 12 2592 1057 01. Queremos o menor múltiplo de 33 formado apenas por algarismos 7. Teremos

33⋅N = 7777...77, com k algarismos 7. Para ser múltiplo de 33, deve ser múltiplo de 11 e de 3. Assim, k deve ser par (pelo critério de divisibilidade por 11) e, também, k deve ser múltiplo de 3, pois a soma dos algarismos de 33N é 7k. Logo, o menor N procurado satisfaz 33⋅N = 777.777, o que nos dá N = 23.569. A soma dos algarismos de N é 2 + 3 + 5 + 6 + 9 = 25. 02. A expressão x2 – (r + s)x + rs pode ser escrita como (x – r)(x – s). Logo, devemos ter (r – x)(x – s) = 2010.

Page 91: Eureka 2011

Sociedade Brasileira de Matemática

EUREKA! N°34, 2011

29

Fazendo r – x = a e x – s = b, a e b com o mesmo sinal, devemos encontrar a + b = r – s sabendo que a e b são inteiros positivos tais que a⋅b = 2010. O número de pares {a, b} que satisfazem esta equação é igual a oito, sendo {a, b} = {1, 2010}, {2, 1005}, {3, 670}, {5, 402}, {6, 335}, {10, 201}, {15, 134}, {30, 67}. 03. Seja O o ponto de interseção entre as retas AB e CD. Veja que os triângulos ODA e OBC são semelhantes, pois ∠OAD = 180° − ∠DAB = ∠BCA. Logo, podemos igualar a razão de semelhança à razão entre os raios das circunferências inscritas, bem como das ex-inscritas, obtendo:

.1214418

8 2 =⇔=⇔= RRRR

04. Em primeiro lugar, escolhemos a coluna que conterá o corte. Isso pode ser feito de 4 modos. Em seguida, escolhemos a cor das casas do corte, o que pode ser feito de 3 modos. Ficamos, então, com três colunas restantes para preencher. Preencheremos primeiramente as casas da primeira linha. Temos 3 modos de colorirmos cada casa da primeira linha, ou seja, 33 modos. Finalmente, resta-nos colorir as casas da segunda linha, o que pode ser feito de 23 modos, já que as cores das casas dessas colunas devem ser diferentes das cores das casas imediatamente superiores. O total de colorações é 4⋅3⋅33⋅23 = 2592. 05. Em primeiro lugar, veja que cada termo do produto é do tipo

.1

1)1()1(24

24

++++++

kkkk Além disso, podemos escrever

).1)(1()1()12(1 2222222424 +++−=−+=−++=++ xxxxxxxxxxx Assim, ficamos com

.)1)(1(

]1)1()1].[(1)1()1[(1

1)1()1(22

22

24

24

+++−++++++−+

=++

++++kkkk

kkkkkkkk Agora, veja que

(k + 1)2 – (k + 1) + 1 = k2 + k + 1 e k2 – k + 1 = (k – 1)2 + (k – 1) + 1. Logo, a última expressão fica

.1)1()1(1)1()1(

11)1()1(

2

2

24

24

+−+−++++

=++

++++kkkk

kkkk

Logo, o produto pedido é igual a

1057132321303013232

144166

122144

100122 2

2

2

2

2

2

2

2

2

=++=++++

⋅⋅++++

⋅++++

⋅++++ … .

Page 92: Eureka 2011

Sociedade Brasileira de Matemática

EUREKA! N°34, 2011

30

SOLUÇÕES NÍVEL 3 – SEGUNDA FASE – PARTE B

PROBLEMA 1:

Seja α = m(IÂC). Então α2)ˆ( =ACIm . Prolongue a reta CI até encontrar o lado AB em D. Como m(CÂD) = 2m(IÂC) = 2α, o triângulo ACD é isósceles e, portanto, suas bissetrizes AI e CE são congruentes.

A

C

B

I

DE

α α

α α 2α

Logo, sendo m(CÊB) = α + 2α = 3α = )ˆ( BCEm e CE = AI = BC, o triângulo BCE

é equilátero. Assim, 60)ˆ( =CBAm .

Outra solução: Considere a mesma figura acima. Aplicando a lei dos senos nos triângulos ACI e ABC, obtemos

αα

ααα 2sen3sen

2sen)2180sen(=⇔=

−− AIACAIAC

e αα

ααα 2sen6sen

2sen)42180sen(=⇔=

−− BCACBCAC

Como AI = BC e 0 < 3α < 6α < 180º,

20180636sen3sen =⇔=+⇔= ααααα . Logo 606180)ˆ( =−= αCBAm .

Page 93: Eureka 2011

Sociedade Brasileira de Matemática

EUREKA! N°34, 2011

31

PROBLEMA 2: Note que Diamantino pode jogar futebol no máximo 5 vezes; caso contrário ele necessariamente joga dois dias seguidos. Suponha que ele joga k dias. Então os k dias em que ele joga devem ser imediatamente seguidos por dias em que ele não joga. Assim, acrescentando um dia ao período, podemos dividir os 11 dias em k blocos de dois dias e 11 – 2k blocos de um dia. Podemos permutar os k + 11 – 2k =

11 – k blocos de

−=

−−

kk

kkk 11

)!211(!)!11( maneiras.

Assim, o total de maneiras de Diamantino escolher os dias em que vai jogar é

14463556361015

5114

4113

3112

2111

1110

011=+++++=

−+

−+

−+

−+

−+

. Outra solução: Seja an o número de maneiras de Diamantino escolher os dias em que vai jogar entre n dias. Se ele jogar no dia n ele não pode ter jogado no dia n – 1, mas não há restrições aos demais n – 2 dias; assim, nesse caso há an–2 maneiras de escolher os dias em que vai jogar; se ele não jogar no dia n não há restrições aos demais n – 1 dias, então nesse caso há an–1 maneiras de escolher os dias. Assim, an = an–1 + an–2, com a0 = 1 (a única opção é não jogar) e a1 = 2 (ele joga ou não no único dia). Dessa forma, podemos encontrar os valores de an a partir dos anteriores:

n 0 1 2 3 4 5 6 7 8 9 10 an 1 2 3 5 8 13 21 34 55 89 144

Logo Diamantino pode escolher os dias de 144 maneiras. Comentários:

• Temos que an = Fn+2, em que Fn é a famosa sequência de Fibonacci (clique no link para saber algumas de suas muitas propriedades!)

• Comparando e generalizando as duas soluções você pode obter a identidade

+

−+

−+

−+

=+ 3

32

21

101

nnnnFn

que soma as outras diagonais do triângulo de Pascal.

Page 94: Eureka 2011

Sociedade Brasileira de Matemática

EUREKA! N°34, 2011

32

PROBLEMA 3: Observando que (1 + x)(1 + y)(1 + z) = 1 + x + y + z + xy + yz + zx + xyz,

==+++=+++++

=+++++++=++

=+++=++

1021024)1)(1)(1(80)1()1()1(

1024177

94677

zyxzyx

xyzzxyzxyzyxzyx

xyzzxyzxyzyx

Como x, y e z são inteiros não negativos, 1 + x, 1 + y e 1 + z são potências de 2. Considerando que 80 = 26 + 24 > 3⋅24, 80 < 27 e x ≤ y ≤ z, temos 24 < 1 + z < 27, ou seja, 1 + z = 25 = 32 ou 1 + z = 26 = 64. Se 1 + z = 32, temos 1 + x + 1 + y = 48 e (1 + x)(1 + y) = 25 = 32. Mas, sendo 1 + x e 1 + y potências de 2 com soma par, temos 1 + x ≥ 2 e, portanto, 1 + y ≤ 16. Então 1 + x ≤ 16 e 1 + x + 1 + y ≤ 32 < 48, e não há soluções nesse caso. Se 1 + z = 64, temos 1 + x + 1 + y = 16 e (1 + x)(1 + y) = 24 = 16. Desse modo, 1 + x e 1 + y são soluções da equação do segundo grau t2 – 16t + 16 = 0, que não tem soluções inteiras. Logo não há soluções. PROBLEMA 4: Como a bola sofre reflexão perfeita, ao refletir a mesa em relação a cada lado em que a bola bate obtém-se uma linha reta. Repetindo as reflexões obtemos a seguinte figura, em que a trajetória da bola é reta:

A

Assim, o problema é equivalente a encontrar uma trajetória em um retângulo de dimensões inteiras m e n, dividido em mn quadradinhos unitários, que começa em um vértice, termina no vértice oposto e corte os lados dos quadradinhos unitários 2010 vezes, sem passar por nenhum dos vértices internos dos quadrados unitários (pois se passasse, a bola chegaria a um vértice do quadrado antes de 2010 rebatidas nos lados).

Page 95: Eureka 2011

Sociedade Brasileira de Matemática

EUREKA! N°34, 2011

33

Como a bola deve atravessar m – 1 quadrados em um sentido e n – 1 no outro, m – 1 + n – 1 = 2010 ⇔ m + n = 2012; como a bola não passa por vértices do quadrado unitário, mdc(m, n) = 1 ⇔ mdc(m, m + n) = 1 ⇔ mdc(m, 2012) = 1. Assim, o número pedido é a quantidade de números coprimos com 2012 menores do que

2012, que é φ(2012) = φ(22 ⋅ 503) = 100450311

2112012 =

− .

Page 96: Eureka 2011

Sociedade Brasileira de Matemática

EUREKA! N°34, 2011

34

XXXII OLIMPÍADA BRASILEIRA DE MATEMÁTICA Problemas e soluções da Terceira Fase

TERCEIRA FASE – NÍVEL 1 PROBLEMA 1 Esmeralda tem muitos triângulos retângulos iguais aos da figura.

5 cm

4 cm

3 cm

a) Fazendo coincidir partes dos lados, sem sobrepor triângulos, Esmeralda montou a figura a seguir. Qual é a área e qual é o perímetro dessa figura?

b) Usando o mesmo processo, Esmeralda montou o menor quadrado possível com lado de medida inteira. Mostre, através de uma figura, como Esmeralda pode fazer isso. PROBLEMA 2 As casas de um tabuleiro 3 × 3 são numeradas de 1 a 9, cada número sendo utilizado exatamente uma vez. Em cada linha horizontal, pintamos de vermelho a casa com o maior número e, de verde, a casa com o menor número. Seja A o menor dos números das casas vermelhas e B o maior dos números das casas verdes.

Page 97: Eureka 2011

Sociedade Brasileira de Matemática

EUREKA! N°34, 2011

35

a) Mostre uma maneira de preencher o tabuleiro de forma que A – B = 4. b) Mostre uma maneira de preencher o tabuleiro de forma que A – B = –3. c) É possível obter A = 4 e B = 3? Não se esqueça de justificar a sua resposta. PROBLEMA 3 Dado um sólido formado por cubos de 1 cm de aresta, como mostra a figura 1, podemos indicar a quantidade de cubos em cada direção, como mostra a figura 2.

1

00

1

0

2

2

0

1

3

2

3

3

0

1

3

1

2

3

1

1

1

1

2

3

2

3

Esmeraldino montou um sólido com cubos de 1 cm de aresta e fez uma figura similar à figura 2.

d

12

1

2

a

2

2

x

2

1

2

1

e

b

3

f

c

1

3

2

3

3

1

m

2

2

Encontre os valores de a, b, c, d, e, f, x e m. PROBLEMA 4 Dizemos que um número inteiro positivo n é abestado se ao lermos da direita para esquerda obtivermos um inteiro maior que n. Por exemplo, 2009 é abestado porque 9002 é maior que 2009, por outro lado, 2010 não é abestado pois 0102, que é o

Page 98: Eureka 2011

Sociedade Brasileira de Matemática

EUREKA! N°34, 2011

36

número 102, é menor que 2010 e 3443 não é abestado pois quando lido da direita para esquerda é exatamente igual ao original. Quantos inteiros positivos de quatro algarismos são abestados? PROBLEMA 5 a) Exiba um número inteiro positivo menor ou igual a 1000 com pelo menos 20 divisores positivos. b) Existe um número inteiro positivo menor ou igual a 11000 com pelo menos 200 divisores positivos? Não se esqueça de justificar a sua resposta. TERCEIRA FASE – NÍVEL 2 PRIMEIRO DIA PROBLEMA 1 Dizemos que um número inteiro positivo n é abestado se ao lermos da direita para esquerda obtivermos um inteiro maior que n. Por exemplo, 2009 é abestado porque 9002 é maior que 2009, por outro lado, 2010 não é abestado pois 0102, que é o número 102, é menor que 2010 e 3443 não é abestado pois quando lido da direita para esquerda é exatamente igual ao original. Quantos inteiros positivos de quatro algarismos são abestados?

PROBLEMA 2 Seja ABCD um paralelogramo e Γ a circunferência circunscrita ao triângulo ABD. Se E e F são as interseções de Γ com as retas BC e CD respectivamente, prove que o circuncentro do triângulo CEF está sobre Γ. PROBLEMA 3 Arnaldo e Bernaldo participam do seguinte jogo em um tabuleiro m × n, m, n ≥ 2. Arnaldo começa escolhendo uma casinha e colocando um cavalo na casinha escolhida; em seguida, Bernaldo e Arnaldo movem alternadamente o cavalo, começando por Bernaldo, com a restrição de que o cavalo não pode cair em casinhas que já foram visitadas. Perde quem não poder mover o cavalo. Determinar, em função de m e n, qual jogador tem uma estratégia para ganhar o jogo, não importando os movimentos do outro jogador e mostrar como ele deve jogar para ganhar. Observação: Cada movimento de um cavalo consiste em ir duas casas na vertical ou na horizontal e, em seguida, uma casa na direção perpendicular.

Page 99: Eureka 2011

Sociedade Brasileira de Matemática

EUREKA! N°34, 2011

37

SEGUNDO DIA

PROBLEMA 4 Sejam a, b e c reais tais que a ≠ b e a2(b + c) = b2(c + a) = 2010. Calcule c2(a + b). PROBLEMA 5 As diagonais de um quadrilátero inscritível ABCD se intersectam em O. Os círculos circunscritos aos triângulos AOB e COD intersectam as retas BC e AD, pela segunda vez, nos pontos M, N, P e Q. Prove que o quadrilátero MNPQ está inscrito em um círculo de centro O. PROBLEMA 6 Os três lados e a área de um triângulo são números inteiros. Qual é o menor valor da área desse triângulo? TERCEIRA FASE – NÍVEL 3 PRIMEIRO DIA PROBLEMA 1 Encontre todas as funções f do conjunto dos reais nos conjuntos dos reais tais que

f(a + b) = f(ab) para todos a, b irracionais. PROBLEMA 2 Seja P(x) um polinômio com coeficientes reais. Prove que existem inteiros positivos n e k tais que k tem n dígitos e mais de P(n) divisores positivos. PROBLEMA 3 Qual é a maior sombra que um cubo sólido de aresta 1 pode ter, no sol a pino? Observação: Entende-se “maior sombra de uma figura no sol a pino” como a maior área possível para a projeção ortogonal da figura sobre um plano. SEGUNDO DIA

PROBLEMA 4 Seja ABCD um quadrilátero convexo e M e N os pontos médios dos lados CD e AD, respectivamente. As retas perpendiculares a AB passando por M e a BC

Page 100: Eureka 2011

Sociedade Brasileira de Matemática

EUREKA! N°34, 2011

38

passando por N cortam-se no ponto P. Prove que P pertence à diagonal BD se, e somente se, as diagonais AC e BD são perpendiculares. PROBLEMA 5 Determine todos os valores de n para os quais existe um conjunto S de n pontos, sem que haja três deles colineares, com a seguinte propriedade: é possível pintar todos os pontos de S de modo que todos os ângulos determinados por três pontos de S, todos da mesma cor ou de três cores diferentes, não sejam obtusos. A quantidade de cores disponíveis é ilimitada. PROBLEMA 6 Encontre todos os pares (a, b) de inteiros positivos tais que 3a = 2b2 + 1. SOLUÇÕES DA TERCEIRA FASE – NÍVEL 1 PROBLEMA 1: SOLUÇÃO DE HELENA VERONIQUE RIOS (SÃO CARLOS – SP) a) Perímetro

( ) ( )3 3 4 4 5 5 4 3 4 3 26.+ + + + + + − + − = O perímetro da figura é 26cm. Área de um triângulo:

24 3 12 6cm2 2 2

b h⋅ ⋅→ = =

Cada triângulo tem 6cm2 de área. Se na figura temos 4 desses triângulos, a área da figura é 4 6,⋅ ou seja, 24cm2. b) 6cm2 – área de cada triângulo Qual o menor múltiplo de 6 que é um quadrado perfeito? 36 ( )6 6⋅ O quadrado deverá ter 236cm e 6cm de lado, se possível. Este quadrado, porém, é impossível de ser formado por causa da forma do triângulo. Teriam de ter dois lados medindo 3cm em cada lado do quadrado, o que seria impossível já que precisariam de 8 lados de 3cm sendo que só tem 6. ( )4 3 6 4 5 6 5 3 6; ;+ ≠ + ≠ + ≠

O próximo menor quadrado possível de ser feito com formas de 26cm é o de lado 12, cuja área é ( )212 12 144cm× .

236cm lado 6− ,

Page 101: Eureka 2011

Sociedade Brasileira de Matemática

EUREKA! N°34, 2011

39

249cm lado 7− , 264cm lado 8− , 281cm lado 9− ,

2100cm lado 10− , 2121cm lado 11− , 2144cm lado 12− ; 49, 64, 81, 100 e 121 não são divisíveis por 6 (área do

triângulo). 144 é divisível por 6 ( )6 24 144⋅ = .

Quadrado de lado 12cm, área 144cm2 com 24 triângulos retângulos de lados 3, 4 e 5 cm.

PROBLEMA 2: SOLUÇÃO DE LUCCA MORAIS DE ARRUDA SIAUDZIONIS (FORTALEZA – CE) a)

Verde

1 5

Vermelho 7

Verde

2

Verde

3

4

6

Vermelho 8

Vermelho 9

A = 7 B = 3 A – B = 4

Page 102: Eureka 2011

Sociedade Brasileira de Matemática

EUREKA! N°34, 2011

40

b)

Verde

1 2

Vermelho

3

Verde

4

Verde

6

5

8

Vermelho

7

Vermelho

9

A = 3 B = 6 A – B = –3

c) Para a ser igual a 4 os dois números que estarão juntos com ele na fileira devem ser ( ) ( ) ( )1 2 2 3 ou 1 3, ; , , . Porém o 3 não pode estar junto com ele na fileira, senão ele não seria pintado de verde. Então uma fileira horizontal é: 1, 2, 4. Porém, para que o 3 seja o B, as outras duas casas verdes teriam que ser 1 e 2. Porém, 1 e 2 estão na mesma fileira, então casas verdes são ( )1 3, , já que o 2 não é verde, a terceira casa verde é um número 4,≥ ocasionando o fato de 3B .≠ Portanto, não é possível. PROBLEMA 3: SOLUÇÃO DE VICTÓRIA MOREIRA REIS COGO (TERESINA – PI) Nesse cubo, podemos formar expressões a partir das placas 3 3 1× × e a partir delas, encontramos o resultado. Veja:

1 3

a 2

2

1

a + 3 + 1 = 2 + 2 + 1 a = 5 – 4 a = 1

c b

1 2

x

2

1 + b + c = 2 + x + 2 1 + b + c = 2 + 3 + 2 b + c = 7– 1 b + c = 6

Page 103: Eureka 2011

Sociedade Brasileira de Matemática

EUREKA! N°34, 2011

41

2

2 3 3

1 x

2 + 2 + x = 3 + 3 + 1 x = 7 – 4 x = 3

Como os resultados só podem ser de 0 a 3, e a única soma que dá 6 é 3 + 3, então: b= 3 e c = 3

1

1 m 2

2 2

1 + 2 + 1 = m + 2 + 2 m = 4 – 4 m = 0

3 b

e

3

3

2

3 + b + e = 3 + 3 + 2 3 + 3 + e = 3 + 3 + 2 e = 8 – 6 e = 2

a 1

d

1

3

m

a + 1 + d = 1 + 3 + m 1 + 1 + d = 1 + 3 + 0 d = 4 – 2 d = 2

1 c

f

2

1

2

1 + c + f = 2 + 1 + 2 1 + 3 + f = 5 f = 5 – 4 f = 1

PROBLEMA 4: SOLUÇÃO DE LUCCA MORAIS DE ARRUDA SIAUDZIONIS (FORTALEZA – CE) Chamamos o primeiro algarismo de A, o segundo de B o terceiro de C e o quarto de D. Testamos os casos: 1º. Caso: o último algarismo é maior que o primeiro. Se A = 1, temos: 1 10 10 8 800⋅ ⋅ ⋅ = Se A = 2, temos: 1 10 10 7 700⋅ ⋅ ⋅ = Se A = 3, temos: 1 10 10 6 600⋅ ⋅ ⋅ = Se A = 4, temos: 1 10 10 5 500⋅ ⋅ ⋅ = Se A = 5, temos: 1 10 10 4 400⋅ ⋅ ⋅ = Se A = 6, temos: 1 10 10 3 300⋅ ⋅ ⋅ = Se A = 7, temos: 1 10 10 2 200⋅ ⋅ ⋅ = Se A = 8, temos: 1 10 10 1 100⋅ ⋅ ⋅ = Total de 3600 casos.

Page 104: Eureka 2011

Sociedade Brasileira de Matemática

EUREKA! N°34, 2011

42

2º. Caso: A D, C B.= > Se B = 0, Temos: 9 1 9 1 81⋅ ⋅ ⋅ = Se B = 1, Temos: 9 1 8 1 72⋅ ⋅ ⋅ = Se B = 2, Temos: 9 1 7 1 63⋅ ⋅ ⋅ = Se B = 3, Temos: 9 1 6 1 54⋅ ⋅ ⋅ = Se B = 4, Temos: 9 1 5 1 45⋅ ⋅ ⋅ = Se B = 5, Temos: 9 1 4 1 36⋅ ⋅ ⋅ = Se B = 6, Temos: 9 1 3 1 27⋅ ⋅ ⋅ = Se B = 7, Temos: 9 1 2 1 18⋅ ⋅ ⋅ = Se B = 8, Temos: 9 1 1 1 9⋅ ⋅ ⋅ = Total de 405 casos. Resposta final: 4005 números. PROBLEMA 5: SOLUÇÃO DA BANCA a) Por exemplo, 2 2 2900 2 3 5 ,= ⋅ ⋅ que tem ( ) ( ) ( )2 1 2 1 2 1 27+ ⋅ + ⋅ + = divisores positivos. b) Não, não existe. Seja n um número com pelo menos 200 divisores. Se o i-ésimo

menor divisor é d, então o i-ésimo maior divisor é n .d

Seja m o centésimo menor

divisor. Temos 100m ≥ e n m,m

> donde 2 10000n m .> ≥ Chegamos perto, mas

isso ainda não resolve o problema. Consideremos o 98º., o 99º. e o 100º. menores divisores de n, que chamaremos de k, l, e m. Note que, se 105m ,≥ teremos como

antes n m,m

> donde 2 2105 11025 11000n m .> ≥ = >

Podemos supor então que 98 104k l m .≤ < < ≤ Como para quaisquer inteiros positivos distintos a, b temos ( )mdc a,b b a ,≤ − e

( ) ( )a bmmc a,b ,

mdc a,b⋅

= concluímos que ( ) ( )( )n mmc k ,l ,m mmc k ,mmc l,m≥ = =

( )( )( )

( )( ) ( )

k mmc l,m k mmc l,mmdc k ,l mdc k ,mmdc k ,mmc l,m

⋅ ⋅= ≥ =

( ) ( ) ( ) ( )( )( ) ( )( )( )98 99 100klm klm .

mdc l ,m mdc k ,l mdc k ,m m l l k m k m l l k m k⋅ ⋅

= ≥ ≥− − − − − −

Page 105: Eureka 2011

Sociedade Brasileira de Matemática

EUREKA! N°34, 2011

43

Como ( ) ( ) 104 98 6m l l k m k ,− + − = − ≤ − = temos ( )( ) 3 3 9m l l k− − ≤ ⋅ = e

( )( )( ) 9 6 54m l l k m k ,− − − ≤ ⋅ = donde 98 99 100 1100054

n .⋅ ⋅≥ >

SOLUÇÕES DA TERCEIRA FASE – NÍVEL 2 PROBLEMA 1: SOLUÇÃO DE LIARA GUINSBERG (SÃO PAULO – SP) Considere o número da forma ABCD. Temos 3 possibilidades:

• A D> ⇒ o número não é abestado. • A D= ⇒ o número é abestado somente se C B> : calculando o número de

casos temos: A D= : 9 possibilidades, pois 0A D ,= ≠ senão o número teria somente 3

algarismos. C D> : 45 possibilidades, pois é o resultado do somatório 9 8 7 1... ,+ + + + já que C pode assumir o valor zero. Totalizando 9 45 405× = números abestados.

• A D< ⇒ o número é abestado, independentemente dos valores B e C. Novamente calculando o número de casos:

A D< : 36 possibilidades, pois é o somatório de 8 7 6 1... ,+ + + + já que A não

pode assumir o valor zero. e B C : 100 possibilidades, já que B pode assumir 10 valores diferentes, assim

como C. Total: 36 100 3600× = números abestados. Finalizando, teremos 405 + 3600 = 4005 números abestados de quatro algarismos. PROBLEMA 2: SOLUÇÃO DE PEDRO MORAIS DE ARRUDA SIAUDZIONIS (FORTALEZA – CE) Seja B AD = α , logo BCD ,= α pois em um paralelogramo os ângulos opostos são iguais. Seja BDA .= β Assim 180ABD .= ° − α − β Veja que o arco 2 2AB BDA= = β e

2 360 2 2AD ABD AD .= ⇒ = ° − α − β

Page 106: Eureka 2011

Sociedade Brasileira de Matemática

EUREKA! N°34, 2011

44

A

D

B

β

180° – α – β α

2α α

F

E

C

360° – 2α – 2β

Note que 2 360 2 360 42

BAD EFBCD EF EF−= ⇒ α = ° − α − ⇒ = ° − α

Com isso 4EBAF .= α Seja O o circuncentro do FCE.∆ Sabemos que 2 2FOE FCE FOE .= ⇒ = α

Como 22 2

EBAF EBAFFOE O ,= α ⇒ = ⇒ ∈Γ pois FOE é ângulo inscrito.

PROBLEMA 3: SOLUÇÃO DA BANCA Chamaremos Arnaldo de A e Bernaldo de B. Suponha, sem perda de generalidade, m ≤ n. Vamos mostrar que para m = 2, A tem estratégia vencedora se e somente se n não é múltiplo de 4; para m ≥ 3, A tem estratégia vencedora se e somente se m e n são ímpares. Estudemos o caso m = 2. Se 4 não divide n, A pode vencer colocando o cavalo na primeira coluna se n = 4k + 1 e na segunda coluna se n = 4k + r, para r = 2 ou r = 3. O cavalo deve ser movido sempre duas colunas à direita em cada jogada, permitindo 2k jogadas a mais. Considere agora o caso n = 4k. Divida o tabuleiro em tabuleiros 2 × 4, e forme pares de casas de modo que é possível mover o cavalo entre casas do mesmo par:

Page 107: Eureka 2011

Sociedade Brasileira de Matemática

EUREKA! N°34, 2011

45

1 2 3 4

3 4 1 2 Como todo o tabuleiro está dividido em pares, B consegue jogar, não importando onde A coloque o cavalo: basta mover o cavalo para a outra casa do par. Quando A jogar, colocará o cavalo numa casa de outro par, e B repete a estratégia. Deste modo, se n = 4k o jogador B tem estratégia ganadora. Isto termina o caso m = 2. O caso m ≥ 3 segue de modo semelhante, dividindo o tabuleiro em vários tabuleiros menores. Considere as seguintes maneiras de se formar pares, além da descrita anteriormente:

1 2 3 4 1 2 3 4 5 6 1 2 3 1 2 3 4 5

3 6 1 5 3 4 1 7 8 9 4 A 1 3 4 6 A 7

2 5 4 6 2 7 8 9 6 5 2 3 4 2 1 7 5 6 Juntando esses tabuleiros se prova que B tem estratégia vencedora para tabuleiros 3 × n, n par e A tem estratégia vencedora para tabuleiros 3 × n, n ímpar: divida o tabuleiro num 3 × 3 ou 3 × 5 e tabuleiros 3 × 4; basta colocar o cavalo na casa marcada com A e seguir a mesma estratégia de B. Para verificar o caso m = 4, basta juntar tabuleiros 4 × 2 se n é par e um tabuleiro 4 × 3 e vários tabuleiros 4 × 2 se n é ímpar. Isso também prova que se A ou B tem estratégia vencedora para um tabuleiro m × n então tem estratégia vencedora também para tabuleiros (m + 4) × n, m ≥ 3. Então basta resolver o problema para m ∈ {3,4,5,6}. Esses tabuleiros resolvem o caso m = 5:

1 2 3 4 5 6 1 2 3 4 5

3 4 1 7 8 9 3 4 5 6 7

2 10 11 9 6 5 2 1 10 8 9

12 13 14 15 7 8 11 A 12 7 6

10 11 12 13 14 15 12 10 11 9 8

Page 108: Eureka 2011

Sociedade Brasileira de Matemática

EUREKA! N°34, 2011

46

Note que B tem estratégia vencedora para o tabuleiro 5 × 4 e A tem estratégia vencedora para o tabuleiro 5 × 3. Então, para o caso 5 × n, n par, juntamos tabuleiros 5 × 4 se n é múltiplo de 4 e um tabuleiro 5 × 6 e tabuleiros 5 × 4 se n é na forma 4k + 2; se n é ímpar, juntamos vários tabuleiros 5 × 4 ao tabuleiro 5 × 3 ou 5 × 5, se n = 4k + 3 ou n = 4k + 1, respectivamente. O caso 6 × n segue diretamente do caso 3 × n se n é par (basta juntar dois tabuleiros 3 × n) e juntando tabuleiros 6 × 4 a um tabuleiro 6 × 3 ou 6 × 5, se n = 4k + 3 ou n = 4k + 1, respectivamente. Com isso, todos os casos estão cobertos. PROBLEMA 4: SOLUÇÃO DE PAULO HENRIQUE OMENA DE FREITAS (SÃO PAULO – SP) Vamos trabalhar com a equação:

( )( )

2

2

2010

2010

a b ca b

b c a

+ =≠

+ =

Usando o método da subtração:

( ) ( )2 2 0a b c b c a+ − + = 2 2 2 2 0a b a c b c b a⇔ + − − =

( ) ( )2 2 0ab a b c a b⇔ − + − =

( ) ( )( ) 0ab a b c a b a b⇔ − + + − =

( )( ) 0a b ab ac bc⇔ − + + = a b⇔ = ou 0ab ac bc .+ + =

Já que, do enunciado, a b,≠ 0ab ac bc .+ + = Colocando c em evidência:

( )c a b ab+ = −

( )2c a b abc⇔ + = − . Colocando a em evidência:

( )a b c bc+ = −

( )2a b c abc⇔ + = − . Assim, temos a igualdade:

( ) ( )2 2 2010c a b abc a b c+ = − = + = . Finalmente, ( )2 2010c a b .+ =

Page 109: Eureka 2011

Sociedade Brasileira de Matemática

EUREKA! N°34, 2011

47

PROBLEMA 5: SOLUÇÃO DE FELLIPE SEBASTIAM S. P. PEREIRA (RIO DE JANEIRO – RJ)

A D

M

P

O

N

Q

B C θ

θ

α α

B

MN

D

Observe que como ABCD é um quadrilátero inscritível temos que os ângulos ACB e BDA são iguais. Temos também que OQCDP é inscritível, logo, como ˆACB e BDA são iguais, segue que os arcos OQ e OP são iguais. Podemos concluir que

os segmentos OQ e OP são iguais também. Analogamente, fazendo tudo com o

quadrilátero ANOMB, chegamos à conclusão que OM e ON são iguais. Agora para provar que O é o centro da circunferência que passa pelos vértices do quadrilátero MNQP (não sabemos ainda se ela existe), basta provarmos que OP ON= , pois assim teremos OM ON OP OQ,= = = concluindo assim que existe uma circunferência de centro O que passa pelos vértices do quadrilátero MNQP. Para provarmos isto façamos o seguinte: chamemos o ângulo ACD de θ . Como o quadrilátero ABCD é inscritível segue que DBA = θ . Temos também que OPN ,= θ pois 180OPD = ° − θ (pois o quadrilátero OPDC é inscritível).

Temos também que PNO = θ , pois NOBA é inscritível. Logo os ângulos ONP e OPN são iguais, donde segue que OM OP= , c.q.d. PROBLEMA 6: SOLUÇÃO DE TADEU PIRES DE MATOS BELFORT NETO (FORTALEZA – CE) Sabemos pela fórmula de Heron que a área de um triângulo é:

( )( )( )S p p a p b p c= − − −

Page 110: Eureka 2011

Sociedade Brasileira de Matemática

EUREKA! N°34, 2011

48

2a b cp + +

=

2 2 2 2a b c a b c a b c a b cS a b c+ + + + + + + + = − − −

2 2 2 2a b c b c a a c b a b cS + + + − + − + − =

( )( )( )( )216S a b c b c a a c b a b c= + + + − + − + − sabemos que a,b,c , e e S a,b c+∈ são lados de um triângulo. Podemos ver que a b c+ + é par. Caso contrário

( ) ( )2 2a b c, a b c a, a b c c+ + + + − + + − e ( ) 2a b c b+ + − seriam ímpares e teriam o produto par, o que é claramente um absurdo. O valor mínimo para essa soma é 4, mas no caso a única tripla de interiros positivos que têm essa soma é ( )1 1 2, , .Mas desobedecem a desigualdade triangular: 1 + 1 não é maior que 2. Essa soma também não pode ser 6. Porque nesse caso o produto teria um fator 3, mas como é um quadrado perfeito teria que ter dois ou mais fatores 3, Assim, ( ) ( )b c a , a c b+ − + − ou ( )a b c+ − teriam esse fator 3. Mas eles são pares e menores que 6, logo não há como isso acontecer (absurdo!). Se 8a b c ,+ + = nenhum dos outros fatores poderia ser 6, caso contrário teria que haver dois fatores iguais a 6. Sem perda de generalidade supomos que

66

b c aa c b

+ − =+ − =

2 12 6 2c c a b c a b.= ⇒ = → + = → > + Absurdo. Assim, temos alguns casos a analisar, pois cada fator só pode ser 2 ou 4.

I) 444

a b ca c bb c a

+ − =+ − =+ − =

12a b c+ + = . Absurdo!

II) 442

a b ca c bb c a

+ − =+ − =+ − =

10a b c+ + = . Absurdo!

Page 111: Eureka 2011

Sociedade Brasileira de Matemática

EUREKA! N°34, 2011

49

III)

8422

a b ca b ca c bb c a

+ + =+ − =+ − =+ − =

( )( )( )( ) 128a b c a b c a c b b c a+ + + − + − + − = .

128 não é da forma 16S2 com S ,+∈ Absurdo!

IV) 222

a b ca c bb c a

+ − =+ − =+ − =

6a b c+ + = . Absurdo!

Podemos ver que a soma não pode ser 10, por um argumento análogo ao do 6. Pois teria que haver outro fator 5, o que faria com que um dos fatores fosse 10, o que é um absurdo! Já que para 12a b c+ + = é possível, basta tornar a= 5, b= 4, c = 3. Vale a desigualdade triangular e a área é dada por

( )( )( )S p p a p b p c= − − − 6 5 4 3p , a , b , c= = = =

6 1 2 3S = ⋅ ⋅ ⋅ 26 6S .= =

Vamos provar que o valor mínimo de ( )( )( )p a p b p c− − − é 6, caso o produto

( )( )( )p p a p b p c− − − fosse menor que 36. Supondo que não fosse, teríamos as seguintes possibilidades:

( )( )( ) 5 5 1 12 2 2

b c a a b c a c bp a p b p c , ,+ − + − + −− − − = ⇒ = = = ⇒

5 1 1 7p = + + = e ( )( )( ) 7 5 35p p a p b p c ,− − − = ⋅ = que não é quadrado perfeito. Absurdo! ( )( )( ) 4p a p b p c− − − = . Para o produto disso com p ser menor que 36 e quadrado perfeito, p seria 4 e já vimos anteriormente que isso é um absurdo. ( )( )( ) 3p a p b p c− − − = . Nesse caso p seria 3, para o produto ser menor que 36 e quadrado perfeito, mas nesse caso 6a b c+ + = e já analisamos esse caso.

( )( )( ) 2p a p b p c− − − = 12

a b c+ −= 1

2a c b+ −

= 22

b c a+ −=

1 1 2 4 8p , S= + + = = . Absurdo!

Page 112: Eureka 2011

Sociedade Brasileira de Matemática

EUREKA! N°34, 2011

50

Logo os valores de ( )( )( )p a p b p c− − − e p são mínimos e portanto o produto é mínimo. Assim, a área mínima é 6. SOLUÇÕES DA TERCEIRA FASE – NÍVEL 3 PROBLEMA 1: SOLUÇÃO DE DAVI COELHO AMORIM (FORTALEZA – CE) Temos que f : → e ( ) ( )f a b f a b+ = ⋅ (1) , a,b∀ irracionais. Logo, temos:

( ) ( ) ( ) ( )( ) ( ) ( ) ( )f a b f a b f a b f a b f a b f a b .+ = ⋅ = − ⋅ − = − − ⇒ + = − −

Lema: Todo número real pode ser representado como a soma de dois números irracionais. 1º. Caso: Número irracional. Seja x um racional e α um irracional. Logo, sendo x ,β = − α β é um irracional, pois se β for racional, xα = − β e α seria racional. Absurdo!

Logo, como ( )x x ,= α + − α = α + β todo racional pode ser escrito como a soma de dois irracionais. 2º. Caso: Número irracional Seja x esse irracional. Vamos supor que para todo 0 1< α < irracional, x ,− α = β onde β é racional. Logo temos:

( )1x

,x

− α = ϕ − − α = θ

onde pq

ϕ = e r , p,q,r ,ss

θ = ∈

Somando obtemos 2 12

p r ps rq sqx x xq s qs

+ +− = + ⇒ = ⇒ é racional. Absurdo!

Logo, todo irracional pode ser escrito como a soma de dois irracionais. Com o lema provado, temos que ( ) ( )f x f x , x= − ∀ ∈ (2) Em (1), fazendo a b,= − temos:

( ) ( )( )

( )2

2 20f f b f b .= − = Seja ( )0f k= ⇒ ( )2 irracional.f b k , b= ∀

Logo, provamos que x +∀ ∈ tal que ( )x , f x k.∉ = Basta provarmos agora

que os números y +∈ tais que y ∈ também satisfazem ( )f y k.= Porém,

isso não é difícil de provar: seja y tal que y ∈ . Temos um 0 1< θ < irracional

tal que ( ) ( ) ( )( )y y f y f y= − θ + θ ⇒ = − θ ⋅ θ e também

Page 113: Eureka 2011

Sociedade Brasileira de Matemática

EUREKA! N°34, 2011

51

( ) ( ) ( ) ( )( )( )y y f y f y= + θ + −θ ⇒ = + θ −θ . Como, para todo x irracional,

( )f x r ,= vamos provar que um dos números ( )y − θ ⋅ θ e ( ) ( )y + θ ⋅ −θ é irracional, fazendo assim com que todo número tenha imagem r. Vamos supor o contrário, ou seja, que os dois são racionais:

( )

( )( )

2

2

2

p py yq q

r ry ys s

p ryq s

− θ θ = θ − θ = ⇒ + θ −θ = − θ − θ =

θ = −

, onde p,q,r ,s ∈

Como y y∈ ⇒ ∈ ⇒tyu

= , onde t ,u ∈

22

t ps rq psu rquu qs qst

− −⇒ ⋅ θ = ⇒ θ = ⇒ θ é racional. Absurdo!

Com isso, provamos que ( )f x k , x= ∀ ∈ , onde k é uma constante qualquer. Para qualquer k, essa função serve, de acordo com a questão, pois

( ) ( )f a b f a b k k+ = ⋅ ⇔ = . OK! PROBLEMA 2: SOLUÇÃO DE CARLOS HENRIQUE DE ANDRADE SILVA (FORTALEZA – CE) Seja d = grau do polinômio p. Agora vamos numerar os primos em ordem crescente, Logo

1 2P ,= 2 3P ,= 3 5P ,= 4 7P ,= ... Então seja b inteiro positivo tal que

12 210 10b b

dP .−+< < Vamos definir k como sendo ( )( )2 2

1 2 2 2 10ada a a b

dk P P ...P .+

+= ⋅ <

Logo ( )2 2n ab d .≤ ⋅ + Podemos supor que para x suficientemente grande

( ) ( )1p x p x .+ > Se isso não ocorre então p é constante ou ( )nlim p n

→∞= −∞ , e então

claramente ( ) ( )p n d k< se a é suficientemente grande, onde ( )d k = número de divisores positivos de k. Então basta provarmos que ( ) ( )( )2 2d k p ab d> + para a suficientemente grande

já que teremos ( )( ) ( )2 2p ab d p n .+ ≥

Então vamos às contas: ( ) ( )2 21 dd k a ;+= + e como o polinômio tem grau d então

( ) 1dp x x +< para x suficientemente grande. Como no nosso problema

Page 114: Eureka 2011

Sociedade Brasileira de Matemática

EUREKA! N°34, 2011

52

( )2 2x ab d= ⋅ + com “a” variável então nosso “x” pode ser tão grande quanto quisermos. Então basta provar que:

( ) ( )( ) ( ) ( ) ( )12 2 21 2 2 1 2 2dda ab d p n a ab d .

+++ > ⋅ + > ↔ + > + Como b, d são constantes então a única variável é “a” e como no lado direito ”a” tem menor grau, então para “a” suficientemente grande a desigualdade é válida, mostrando que existem k, n que satisfazem a condição do enunciado. PROBLEMA 3: SOLUÇÃO DA BANCA Sejam ABCD e EFGH duas faces opostas, AE, BF, CG e DH sendo lados do cubo. Denotaremos por X´ a projeção ortogonal do ponto X no plano. Note que { } { } { }A,G , B,H , C,E e { }D,F são pares de vértices opostos. Suponha, sem perda de generalidade, que A pertence à fronteira da projeção do cubo. Então, considerando a simetria do cubo em relação ao seu centro, o simétrico G de A também pertence à fronteira. Dois dos três vértices vizinhos de A serão projetados em vértices vizinhos de A na fronteira (a menos que, digamos, a face AEHD seja projetada em um segmento, mas nesse caso podemos considerar um vértice degenerado nesse segmento). Suponha, sem perda de generalidade que esses vizinhos são B e D . Então E é interior à projeção. Novamente pela simetria, H´ e F´ pertencem à fronteira da projeção e C´ pertence ao interior da projeção. Finalmente, como AE BF CG DH ,= = = a projeção do cubo é A D H´G F´ B .

As faces ABCD, BCGF e CDHG são projetadas sobre paralelogramos (ou segmentos) A B C´ D , B C´G´ F´ e C´ D H´G . Trace diagonais

e B D ,B G D G . A área da projeção é portanto o dobro da área do triângulo BDG. Esse triângulo é equilátero de lado 2 ,e logo o máximo desejado é

Page 115: Eureka 2011

Sociedade Brasileira de Matemática

EUREKA! N°34, 2011

53

( )22 3

2 34

.⋅

⋅ = Uma projeção ortogonal num plano paralelo ao plano BDG

realiza a igualdade. PROBLEMA 4: SOLUÇÃO DE GUSTAVO LISBÔA EMPINOTTI (FLORIANÓPOLIS – SC) Na verdade a recíproca é verdadeira, mas a implicação direta nem sempre vale, como veremos a seguir. ( )⇐ Suponha AC BD.⊥

Então podemos tornar um sistema de coordenadas em que AC é o eixo x e BD o eixo y. Sejam A = (a, 0), B = (0, b), C = (c, 0) e D = (0, d). Claramente 0abcd .≠

O coeficiente angular de PM é ab

(pois ).AC BD⊥ Como 2 2c dM , , =

a equação

da reta PM é 2 2d a cy x .

b − = −

Analogamente, a equação de PN é

2 2d c ay x .

b − = −

Como ( )p pP x , y PM PN ,= ∩ temos

( ) 02 2p p pc aa x c x x c a − = − → − =

e a c≠

0px P BD→ = → ∈ (eixo y).

( )⇒ Suponha P BD.∈

Então podemos tomar um sistema de coordenadas em que ( ) ( ) ( )0 0 0 0P , , B ,b , D ,d .= = = Sejam ( )a aA x , y= e ( )c cC x , y=

2 2 2 2

c c a ax y d x y dM , , N , .+ + → = =

O coeficiente angular de PM é c

c

y dx+ (claramente 0cx ≠ ) e o de AB, a

a

y bx−

(também temos 0ax ≠ ).

( )( )1c ac a a c

c a

y d y bPM AB y d y b x xx x

+ −⊥ ⇒ ⋅ = − → + − = − ⋅

(*)

Analogamente, ( )( )a c a cPN BC y d y b x x⊥ → + − = − ⋅

Page 116: Eureka 2011

Sociedade Brasileira de Matemática

EUREKA! N°34, 2011

54

( )( ) ( )( )a c c a a c a c a c c ay d y b y d y b y y b y d y bd y y by d y bd→ + − = + − ↔ ⋅ − ⋅ + ⋅ − = ⋅ − + ⋅ −

( ) ( )a c a cy b d y b d y y↔ + = + ↔ = ou 0b d .+ = Se a cy y , AC= é paralelo ao eixo x e, portanto, perpendicular a BD, que é o eixo y.

Se 0b d ,+ = (*) vira 1c a

c a

y b y b ,x x

− −⋅ = −

o que implica BC AB⊥ (pois c

c

y bx−

e a

a

y bx− são, respectivamente, os coeficientes angulares de BC e AB), ou seja,

90ABC .∠ = ° Isso caracteriza todos os contra exemplos: A partir de um triângulo PNM∆ retângulo em P, tome um ponto D dentro do ângulo NPM∠ mas fora do triângulo NPM,∆ e tal que ( ) 90DP,NM .∠ ≠ ° Aplique uma homotetia de centro em D e razão 2. Fazemos N A, M´ C= = e P B.= Temos um quadrilátero ABCD convexo no qual M e o ponto médio de CD; N , o de AD; e P, o de BD. Como PN // AB e PM PN,⊥ temos PM AB⊥ e da mesma forma PN BC.⊥ 90ABC NPM .∠ = ∠ = ° Esse é o contraexemplo, pois pela escolha de

D, P BD∈ mas BD não é perpendicular a AC.

P´= B

N M

D

P N´= A M´= C

PROBLEMA 5: SOLUÇÃO DE ANDRÉ SARAIVA NOBRE DOS SANTOS (FORTALEZA – CE) Se tivermos 4 pontos, todos da mesma cor ou todos de cores diferentes, teremos o seguinte: 1º. Caso: Eles formam um quadrilátero convexo:

Page 117: Eureka 2011

Sociedade Brasileira de Matemática

EUREKA! N°34, 2011

55

a

b d

c

a + b+ c + d = 360° logo, se eles forem diferentes, haverá um deles maior que 90° (pois se todos forem menores, a soma não daria 360°), logo, todos têm que ser de 90°, ou seja, eles têm que formar um retângulo.

a

b

2º. Caso: Eles formam um quadrilátero côncavo: Como a + b > 180°, garanto que um deles é maior que 90°, logo, isso não pode acontecer.

Então, como 4 pontos de cores diferentes têm que formar um retângulo, não temos mais como ter 5 pontos de 5 cores diferentes nesse conjunto. Sendo assim, podemos ter 1, 2, 3 ou 4 cores: 1 cor: só podemos ter até 4 pontos, pois 3 deles só definem a posição do próximo, só que os 4 têm que formar um retângulo: ex:

A A

A A

2 cores: podemos ter até 8 pontos, de cada cor 4: ex:

A A

A A

V V

V V

Page 118: Eureka 2011

Sociedade Brasileira de Matemática

EUREKA! N°34, 2011

56

3 cores: conseguimos uma configuração com até 12 pontos, basta eles ficarem muito afastados, cada retângulo acutângulo, pois assim, ao escolhermos 3 cores diferentes, os ângulos do triângulo vão ser aproximadamente os do triângulo acutângulo: ex:

60°

60° 60°

A

A

A

A

V V

V V L L

L L

4 cores: já mostramos que só conseguimos 4 pontos: de fato, 3 pontos de cores diferentes determinam uma única posição possível para os pontos da outra cor, e logo há apenas um ponto de cada cor. Logo, como checamos todos os casos, vimos que não conseguimos mais de 12 pontos e achamos um exemplo com 12, qualquer n menor também satisfaz, pois tirar pontos de uma configuração faz com que a restante também satisfaça. Logo, n de 0 até 12 é solução, não existindo mais nenhuma. PROBLEMA 6: SOLUÇÃO DA BANCA As soluções são (1,1), (2,2) e (5,11). Se a é par e maior que 2, a equação equivale a (3a/2 – 1) ⋅ (3a/2 + 1) = 2b2. Porém mcd(3a/2 – 1, 3a/2 + 1) = mcd(3a/2 – 1, 2) = 2 e se conclui que 3a/2 + 1 = 4u2 e 3a/2 – 1 = 2v2 ou 3a/2 + 1 = 2u2 e 3a/2 – 1 = 4v2. No primeiro caso, 3a/2 = (2v – 1)(2v + 1), e como mcd(2v – 1, 2v + 1) = mcd(2v – 1, 2) = 1, 2v – 1 = 1⇔ v = 1 e a/2 = 1 ⇔ a = 2 e portanto b = 2.

Page 119: Eureka 2011

Sociedade Brasileira de Matemática

EUREKA! N°34, 2011

57

No segundo caso, 3a/2 = 4v2 + 1 ⇒ 0 ≡ v2 + 1 (mod 3) ⇔ v2 ≡ –1 (mod 3), o que é impossível. Se a é ímpar, a equação é equivalente a 3 ⋅ (3(a–1)/2)2 – 2b2 = 1. Seja c = 3(a–1)/2. Encontremos as soluções de 3c2 – 2b2 = 1 (*).

Como ( )( ) ( ) ( ) 12323123231212

=−+⇒=−+++ kk

e

( ) 232312

kkk

bc +=++

y ( ) 232312

kkk

bc −=−+

(**) (ck, bk) são soluções de (*), para k ≥ 0 inteiro. Suponha que existe uma solução (α, β) distinta com ,α β ∈ . Então existe k ∈ tal que

( ) ( )

( )( )( ) 211396252323

21139625

2323

232323

1

1

1212

+<−+<+⇔

+<+

+<+⇔

+<+<+

+−

k

k

kk

βα

βα

βα

É possível provar por indução que ( )( ) 23625231

φθβα +=−+−k

, θ e φ ambos inteiros satisfazendo 2 23 2 1θ φ− = . Além disso,

02323

1123 >−=+

>>+ φθφθ

φθ , e portanto 023 >> φθ .

Portanto ( )φθ , é solução de (*), com 1 < θ < 9. Porém é possível verificar (testando) que não há soluções com 1 < θ < 9, contradição.

Para k = 0 temos ( )2 13 2 3 2

k,

++ = + o que nos dá a solução ( ) ( )1 1c,b ,= de

(*), e, para k = 1, temos ( )2 13 2 9 3 11 2

k,

++ = + o que nos dá a solução

(c, b) = (9, 11) de (*). Suponha que c > 9, ou seja, k > 1. Por (**) e pelo teorema do binômio temos

∑=

−− +⋅⋅

++⋅+=⋅⋅

++

=k

m

kkmkm kk

mk

c0

1233

122)12(23

1212

(***)

Page 120: Eureka 2011

Sociedade Brasileira de Matemática

EUREKA! N°34, 2011

58

Logo 3 divide 2k + 1. Seja 3t a maior potência de 3 que divide 2k + 1. Como 11 2)12()12(23

312 −− ⋅−⋅⋅+=⋅⋅

+ kk kkkk

, a maior potência de 3 que divide o

segundo termo de (***) é também 3t . Para m > 1, sendo 3s a maior potência de 3

que divide 2m + 1, o m-ésimo termo, mkmmkm

mk

mk

mk −− ⋅⋅

++

=⋅⋅

++

2322

121223

1212

tem pelo menos t + m – s fatores 3 (t de 2k + 1, m de 3m, subtraindo s de 2m + 1). Temos 2m s− ≥ para todo 1m .> De fato, para m = 2 e m = 3 temos s = 0 e, para

( )34 2 1 2m ,m s m log m≥ − ≥ − + ≥ (isso segue da desigualdade

3 1 2 22

r

r , r ,−≥ + ∀ ≥ que pode ser facilmente provada por indução).

Então, como k > 1, todos os termos a partir do terceiro tem pelo menos t + 2 fatores 3, e

Nkkk

Nkkkkctka

tkk

⋅++−+⋅⋅+=⇔

⋅+⋅−⋅⋅++⋅+=+−−

+−

212/)1(

21

3]3)1)(12[(2)12(3

32)12()12(2)12(

Note que como 3 divide 2k + 1 então também divide (2k + 1)(k – 1) + 3; além disso, 9 divide (2k + 1)(k – 1), e portanto a maior potência de 3 que divide (2k + 1)(k – 1) + 3 é 3. Portanto a maior potência de 3 que divide

]3)1)(12[(2)12( 1 +−+⋅⋅+ − kkk k é 3t+1 (t fatores de 2k + 1 e 1 de (2k + 1)(k – 1) + 3). Finalmente, 3(a–1)/2 = c = 3t+1 (1 + 3N), que não é possível pois N > 0. Dessa forma, não há mais soluções.

Page 121: Eureka 2011

Sociedade Brasileira de Matemática

EUREKA! N°34, 2011

59

XXXII OLIMPÍADA BRASILEIRA DE MATEMÁTICA Problemas e soluções da Primeira Fase – Nível Universitário

PROBLEMA 1: Há muito tempo em uma galáxia muito distante, utilizavam-se como referência para viagens espaciais os pontos A, B, C, D, E, F, G, H, vértices de um cubo de ares igual a um ano-luz tendo os quadrados ABCD e EFGH como faces e tendo os segmentos AE, BF, CG e DH como arestas. Uma nave espacial viaja com velocidade constante em trajetória retilínea de B para C. Outra nave viaja com velocidade constante igual ao triplo da velocidade da primeira, em trajetória retilínea de A para G. Sabendo que a primeira atinge o ponto C no mesmo instante em que a segunda atinge o ponto G, determine a menor distãncia entre as naves durante esse deslocamento. PROBLEMA 2: Quantos são os pares ordenados (x, y) com { }0 1 2 142x, y , , ,...,∈ tais que

2 25 7 1x y+ − é múltiplo de 143? PROBLEMA 3: Dados os polinômios com coeficientes complexos em uma variável ( )f x e ( )h x ,

prove que existe um polinômio ( )g x tal que ( ) ( )( )f x g h x= se, e somente se,

existe um polinômio com coeficientes complexos em duas variáveis ( )q x,y tal

que ( ) ( ) ( ) ( ) ( )( )f x f y q x, y h x h y .− = − PROBLEMA 4: Seja n um inteiro positivo. Seja nA o subconjunto do plano definido por ( )1 0x n, y ln x .≤ ≤ ≤ ≤ Seja nB o

polígono convexo de vértices ( ) ( )( ) ( )( ) ( )( ) ( )( ) ( )1 0 1 1 2 2 3 3 0, ,ln , ,ln , ,ln ,..., n,ln n , n, .= Seja n n nC A B ,= − o complemento de nB em relação a nA . a) Calcule as áeas de n nA ,B e nC .Simplifique sua resposta. b) Mostre que a área de nC é menor que 1, para qualquer inteiro positivo n. Obs: ln representa o logaritmo na base e.

Page 122: Eureka 2011

Sociedade Brasileira de Matemática

EUREKA! N°34, 2011

60

PROBLEMA 5: Suponha que temos um grafo com 1 4n + ≥ vértices e queremos pintar suas arestas com duas cores de forma que não haja duas arestas disjuntas da mesma cor. Mostre que há no máximo 2n tais colorações. Observações: Um grafo é formado por um conjunto de vértices e um conjunto de arestas, cada aresta unindo dois vértices distintos e cada para de vértices sendo unido por no máximo uma aresta. Arestas disjuntas são arestas que não têm vértices em comum. PROBLEMA 6: Cada um dos itens a seguir apresenta um valor diferente para a matriz B. Para cada um desses valores, determine quantas matrizes reais A existem tais que

3 3A A B.− =

a) 0 11 0

B =

b) 4 00 4

B =

c) 4 10 4

B =

SOLUÇÕES NÍVEL UNIVERSITÁRIO – PRIMEIRA FASE PROBLEMA 1 Dando coordenadas, suponha sem perda de generalidade que

( ) ( ) ( ) ( )( ) ( ) ( ) ( )0 0 0 1 0 0 11 0 0 1 0

0 0 1 1 0 1 111 0 11

A , , , B , , , C , , , D , , ,

E , , , F , , , G , , , H , , .

= = = =

= = = =

Se as posições (em função do tempo) das duas naves são ( )tα e

( )t ,β respectivamente, se t = 0 é o instante em que ( )t Cα = e ( )t Gβ = e 1t = − é

o instante em que ( )t Bα = temos

( ) ( ) ( ) ( ) ( ) ( )11 0 0 1 0 111 3 111t , , t , , , t , , t , , .α = + β = + Assim o quadrado da distância em função do tempo é

( ) ( ) ( )( ) ( ) ( )( ) ( )( )( ) ( )

2 2 2

22 2 2 2

1 1 3 1 1 3 0 1 3

3 3 1 1 2 3 3 10 2 3 2 3 1

h t t t t t

t t t t t t .

= − + + + − + + − +

= + − + + + = − + +

Page 123: Eureka 2011

Sociedade Brasileira de Matemática

EUREKA! N°34, 2011

61

Temos ( ) ( )20 4 3 2 3h t t .= − +

Para ( ) ( )0 2 3 20 4 3 3 5 3 44 0 265t ,= − − = − + ≈ − temos ( )0 0h t ;= para

0t t< temos ( ) 0h t < e para 0t t> temos ( ) 0h t .> Assim o mínimo do quadrado da distância é

( ) ( )0 29 3 3 44 0 541h t ,= − ≈

e a distância mínima é

( )29 3 3 44 0 7355,− ≈

PROBLEMA 2 Note que 143 11 13.= × Se pN é o número de pares ordenados (x, y) com

{ }0 1 1x, y , ,..., p∈ − tais que 2 25 7 1x y+ − é múltiplo de p, então a resposta do

problema será 11 13N N .⋅ de fato, 2 25 7 1x y+ − é múltiplo de 143 se, e somente se, é múltiplo de 11 e 13. Por outro lado, pelo teorema chinês dos restos, dados pares ordenados ( )x , y com { } ( )0 1 10 e x , y ` , ,..., x ´, y ´∈ com

{ }0 1 12 ,x ´, y ´ , ,...,∈ existe um único par ordenado ( )x, y com { }0 1 142x, y ` , ,...,∈ tal

que ( ) ( ) ( )11 13 11x x mod ,x x" mod , y y mod≡ ≡ ≡ e ( )13y y" mod .≡

Vamos agora calcular 11N e 13N . Os possíveis valores de ( )2 11x mod são 0, 1, 4, 9, 5, 3, sendo cada valor não nulo atingido para duas classes de congruência módulo 11. Assim, 5 é quadrado módulo 11 mas 7 não é, e portanto ( )25 11x mod

assume os valores 0, 1, 3, 4, 5, 9, enquanto ( )27 11x mod assume os valores 0, 2,6,7,8,10 (nos dois casos os valores não nulos são assumidos duas vezes). Temos que 1 é o resultado módulo 11 da soma de números dessas duas listas nas formas 1 + 0,4 + 8 e 5 + 7, o que dá 2 + 4 + 4 = 10 soluções módulo 11 de 2 25 7 1x y ,+ = e portanto 11 10N .= Analogamente, os possíveis valores de ( )2 13x mod são 0, 1, 4, 9, 3, 12, 10, sendo cada valor não nulo atingido para duas classes de congruência módulo 13. Assim, 5 e 7 não são quadrados módulo 13, e portanto ( )25 13x mod e

( )27 13x mod assumen os valores 0, 2, 5, 6, 7, 8, 11 (os valores não nulos são assumidos duas vezes). Temos que 1 é o resultado módulo 13 da soma de dois números dessa lista nas formas 6 + 8,8 + 6 e 7 + 7, o que dá 4 + 4 + 4 = 12 soluções módulo 13 de 2 25 7 1x y ,+ = e portanto 13 12N .=

Page 124: Eureka 2011

Sociedade Brasileira de Matemática

EUREKA! N°34, 2011

62

Assim, a resposta do problema é 11 13 10 12 120N N .⋅ = ⋅ = PROBLEMA 3 Note que, se ( ) 1 0

nn xg x a x ... a a ,= + + + então

( ) ( ) ( ) ( ) ( )( ) )1 2 11 1

n n n n nn ng u g v a u v ... a u v u v a u u v ... v ... a− − −− = − + + − = − + + + + +

= ( ) ( )R u,v u v ,⋅ − para um certo polinômio em duas variáveis ( )R x, y , e logo, se

( ) ( )( )f x g h x ,= então

( ) ( ) ( )( ) ( )( ) ( ) ( )( ) ( ) ( ) ( ) ( ) ( )( )( )f x f y g h x g h y R h x ,h y h x h y q x,y h x h y ,− = − = ⋅ − = −

com ( ) ( ) ( )( ):q x, y R h x ,h y ,= o que mostra a primeira implicação. Vamos provar a volta por indução no grau de f. Se o grau de f for 0, as duas afirmações são verdadeiras. Suponha agora que f não é constante e que

( ) ( ) ( ) ( ) ( )( )f x f y q x, y h x h y− = − (daí segue que h não é constante). Fazendo y

= 0, obtemos ( ) ( ) ( ) ( ) ( )( )0 0 0f x f q x, h x h ,− = − para todo x, e portanto,

( ) ( ) ( ) ( ) ( )( )0 0 0f y f q y, h y h ,− = − para todo y. Substraindo, obtemos

( ) ( ) ( )( ) ( ) ( ) ( ) ( ) ( )( ) ( ) ( ) ( )( )0 0 0 0q x,y h x h y f x f y q x, h x h q y, h y h− = − = − − − =

Assim, ( ) ( )h x h y− divide o polinômio ( ) ( )( ) ( ) ( )( )0 0 0q x, q y, h y h .− − Como

h não é constante e ( ) ( )0h y h− é um polinômio só na variável y, segue que

( ) ( )h x h y− não tem nenhum fator comum (não constante) com ( ) ( )0h y h ,− e

portanto ( ) ( )h x h y− divide o polinômio ( ) ( )0 0q x, q y, .− Seja

( ) ( ) ( ): 0q x q x q .= − Temos ( ) ( ) ( ) ( ) ( )( ) ( ) ( ) ( )( )0 0 0 0f x f q x, h x h q x h x h ,− = − = −

donde o grau de q é menor que o grau de f. Por outro lado, ( ) ( )h x h y− divide o

polinômio ( ) ( ) ( ) ( )0 0q x, q y, q x q y ,− = − e portanto, por hipótese de indução,

existe um polinômio ( )g x tal que ( ) ( )( )q x g h x ,= donde

( ) ( ) ( ) ( ) ( )( )0 0f x f q x h x h= + − = ( ) ( )( ) ( ) ( )( ) ( )( )0 0f g h x h x h g h x ,+ − =

onde ( ) ( ) ( ) ( )( ): 0 0g x f g x x h ,= + − o que completa a demonstração.

Page 125: Eureka 2011

Sociedade Brasileira de Matemática

EUREKA! N°34, 2011

63

PROBLEMA 4

A figura mostra as regiões A5 (abaixo da curva vermelha), B5 (abaixo da poligonal azul), e C5 (entre a poligonal azul e a curva vermelha). Temos

Área ( ) ( ) ( )1

1n

nA ln t dt nln n n= = − +∫ ;

Área ( ) ( ) ( ) ( ) ( ) ( ) ( )1 12 3 12 2nB ln ln ... ln n ln n ln n! ln n= + + + − + = − ;

Área ( ) ( ) ( ) ( )1 12nC nln n n ln n ln n! .= − + + −

Para estimar Área Área ( )nC escreva

( ) ( ) ( ) ( ) ( )( )11

k

k ka ln t ln k t k ln k ln k dt

+= − − − + −∫ ;

note que ka é a área da k-ésima ‘bochechinha” entre a poligonal azul e a curva vermelha. Assim,

Área ( ) 1 2 1n nC a a ... a −= + + + ; Queremos estimar ka para mostrar que a série abaixo converge para S < 1:

1 20 1kS a a ... c ... .< = + + + + < Seja ( ) ( ) ( ) ( ) ( ) ( )( )1ku t ln t ln k t k ln k ln k= − − − + − ; temos ( ) ( )1 0k ku k u k .= + =

Note que ( ) 2"ku t t .−= − Integrando por partes temos

( )1k

k kka u t dt

+= ∫

( )1 1

2k '

kkt k u t dt

+ = − − − ∫

Page 126: Eureka 2011

Sociedade Brasileira de Matemática

EUREKA! N°34, 2011

64

( ) ( )12 2

11 1 1 12 2 2 2

kk' "

k kkk

t k u t t k u t dt+

+ = − − − + − −

( )( )2

1 1 1 18 2 2

k "kk

t k u t dt+ = − − − −

Para 1k t k≤ ≤ + temos ( )( )"ku t dt− donde

212

2

1 1 1 18 2 2 12

k

k ka k t k dt .

k+−

≤ − − − = ∫

Temos portanto

( )2 21 1 1 2 112

S ... k ...≤ + + + +

( )2

1

1 1112 6

t dt+∞ −≤ + ≤∫ ;

Completando a demonstração. Observação: Este problema mostra como obter estimativas como a de Stirling: temos 0 ≤ Área ( ) 1 6nC ≤ donde

( ) ( ) ( )1 10 112 6

nln n n ln n ln n!≤ − + + − ≤

( ) ( ) ( ) ( ) ( )1 5 1 112 6 2

nln n n ln n ln n! nln n n ln n− + + ≤ ≤ − + +

5 3 2n n n nn e e n n! n e e n− −≤ ≤ Sabemos por Stirling que a melhor aproximação é

2n nn! n e n−≈ π ; note que 5 3 22e e .< π < PROBLEMA 5 Suponha que algum vértice do grafo esteja contido em todas as arestas do grafo. Então o grafo é uma estrela com n pontas, e o resultado segue (há exatamente 2n colorações para este exemplo). Suponha que o grafo tenha um vértice x de grau 3≥ (i.e., que pertença a pelo menos 3 arestas) e que exista uma aresta disjunta de x, digamos e. Devido à hipótese sobre o grau de x, para aqualquer aresta e, há uma aresta ( )f f e= que incide em x que é disjunta de e. Então, em qualquer coloração das arestas que

Page 127: Eureka 2011

Sociedade Brasileira de Matemática

EUREKA! N°34, 2011

65

incidem em x, a cor de f define a cor de e (a cor de f é a oposta de e). Assim, há no máximo ( )2 2grau x n≤ colorações. Se o grafo tem um vértice x de grau 2, ligado a dois outros vértices y e z, então para toda aresta e disjunta de x que não seja a (possível) aresta yz, há uma aresta

( )f f e= que incide em x que é disjunta de e, cuja cor determina a cor de e. Assim, as cores de xy, xz e de yz (se existir) determinam todas as outras. Assim, há no máximo 32 2n≤ colorações. Finalmente, se todo vértice tem grau no máximo 1, todas as arestas são disjuntas, e nesse caso, pelas hipótese do problema, o grafo pode máximo duas arestas e há no máximo 22 2n< colorações. PROBLEMA 6 Antes de mais nada vamos esboçar o gráfico de ( ) 3 3f x x x.= −

Vemos que para 2 2y− < < a equação ( )f x y= admite três soluções reais enquanto para 2y < − ou 2y > ela admite uma solução real e duas complexas conjugadas. a) Os autovalores de B são 1 e – 1 donde podemos escrever 1B XDX −= para X inversível e

Page 128: Eureka 2011

Sociedade Brasileira de Matemática

EUREKA! N°34, 2011

66

1 00 1

D .

= −

Sejam 1 2 3c ,c ,c (resp. 1 2 3d ,d ,d ) as soluções reais de ( ) ( )( )1 resp. 1f x f x .= = − Se

( )f A B= temos ( )1f X AX D− = donde 1X AX− é da forma

11 00 j

cX AX

d−

=

Para i, j escolhidos independentemente. Há portanto 9 matrizes reais A que satisfazem ( )f A B.= b) Sejam z,z as raízes complexas de ( ) 4f x .= Seja ( ) 2

1 2v w ,w= ∈ um vetor

linearmente independente com ( )1 2v w ,w= e considere A unicamente definida por

Av zv,Av zv.= = Em outras palavras, 1

1 11 1

2 22 2

00

w w w wzA .

zw w w w

=

Para qualquer tal A temos ( ) 4f A I B.= = Temos além disso A real: há portanto

infinitas matrizes reais A que satisfazem ( )f A B.= c) Se M é diagonalmente então ( )f M também o é. Como B não é diagonalmente então A também não o é. Assim A deve ter autovalor com multiplicidade algébrica igual a 2 logo o único autovalor de A é o único real c com ( ) 4f c .= Além disso qualquer autovetor de A é autovetor de B; como 1e é (a menos de múltiplo escalar) o único autovalor de B então 1e deve ser autovetor de A. Já 2e deve ser autovetor generalizado, isto é, devemos ter 2 2 1Ae ce ye= + (para algum real y). Assim

0c y

Ac

=

( ) ( )23 4 3 3

30 4

c yf A A A

−= − =

e devemos ter ( )21 3 3y c .= − Há portanto uma única solução.

Page 129: Eureka 2011

Sociedade Brasileira de Matemática

EUREKA! N°34, 2011

67

XXXII OLIMPÍADA BRASILEIRA DE MATEMÁTICA Problemas e soluções da Segunda Fase – Nível Universitário

PRIMEIRO DIA PROBLEMA 1:

Calcule ( )

4

0 sen cos cos

/ x dx.x x x

π

+∫

PROBLEMA 2: Qual a maior área possível para a sombra de um cubo de aresta 1? (Obs.: supomos que o sol está a pino, isto é, a sombra é uma projeção ortogonal; o cubo pode estar em qualquer posição). PROBLEMA 3: Sejam 1n e 2n inteiros positivos e 1 2n n n .=

Considere a matriz real simétrica n n,× ( )1

i , j i , j nA a

≤ ≤= , tal que para todo i,

4i ,ia ,=

1 1 1i ,i i ,ia a+ += = − para 1 1i n≤ ≤ − tal que ( )1i + não é múltiplo de 1n ,

1 11i ,i n i n ,ia a ,+ += = −

e as demais entradas i , ja são iguais a 0. Prove que A é invertível e todas as entradas de 1A− são positivas.

Page 130: Eureka 2011

Sociedade Brasileira de Matemática

EUREKA! N°34, 2011

68

SEGUNDO DIA PROBLEMA 4:

Definimos os polinômios ( )( ) ( )1 2 1x x x x ... x jj j !

− − − + =

para todo j natural,

com 10x

.

=

a) Prove que todo polinômio não identicamente nulo pode ser escrito como uma

combinação linear desses xj

de forma única;

b) Seja nk

o coeficiente de xk

no desenvolvimento de nx (como no item a)).

Calcule

111

n nk k

.nk

++

++

PROBLEMA 5: Se F é um subconjunto finito de 3 , denotamos por ( )rV F a vizinhança de raio r de F (i.e., a união das bolas abertas de raio r com centros pertencentes a F). Prove que, se 0 r R,< < vol ( )( ) ( )3

RV F R / r .≤ vol ( )( )rV F . PROBLEMA 6: Prove que se 210 8 10 1n n.+ + tem um fator primo da forma 60k + 7 então n e k são pares.

Page 131: Eureka 2011

Sociedade Brasileira de Matemática

EUREKA! N°34, 2011

69

PROBLEMA 1: SOLUÇÃO DE BRUNO DA SILVA SANTOS (BELFORD ROXO – RJ)

( )24 4

0 0

secsen cos cos tgx 1

x xI dx x dx.x x x

π π= = ⋅

+ +∫ ∫

Fazendo: ( )2sec e ln 1 tgx

1 tgxxx u;dv dx du dx v= = → = = +

+

como u dv u v v du := ⋅ −∫ ∫

( ) ( ) ( )4 4

40 00

ln 1 tgx ln 1 ln2 ln 1 tgx4

I x tgx dx dxπ

π ππ= ⋅ + − + = − +∫ ∫

Seja ( )4

2 0ln 1 tgx .I dx

π= +∫ Fazendo 4u x= π − e du dx := −

( )( )4 4

2 0 0

1ln 1 tg 4 ln 11

tguI u du dutgu

π π −= + π − = + +

∫ ∫

( )( ) ( )4 4

2 0 0ln2 ln 1 ln2 ln 1

4I tgu du tgu du

π ππ= − + = − +∫ ∫

2 2 2ln2 ln24 8

I I I .π π= − → =

Portanto ( )4

0ln2 1 ln2 ln2 ln2

4 4 8 8I ln tgx dx .

ππ π π π= − + = − =∫

ln24

I .π=

PROBLEMA 2: SOLUÇÃO DE RAFAELTUPYNAMBÁ DUTRA (BELO HORIZONTE – MG) Sendo A o vértice do cubo que está mais em baixo (um vértice com altura mínima), as três faces que contêm A estão no escuro. Colocamos os eixos de forma que os vetores unitários normais a essas três faces sejam = (1,0,0);i = (0,1,0);j

= (0,0,1),k e consideramos o vetor unitário ( )x y zN N ,N ,N ,= com 2 2 2 1x y zN N N ,+ + = paralelo à direção dos raios solares.

Projetando as faces escuras do cubo sobre o plano horizontal, vemos que a área da sombra do cubo é igual à soma das áreas das projeções das três faces escuras. Mas a área da projeção de uma face é igual à área da face original (que é 1) multiplicada pelo módulo do produto escalar entre o vetor normal à face e o vetor normal ao

Page 132: Eureka 2011

Sociedade Brasileira de Matemática

EUREKA! N°34, 2011

70

plano de projeção. Assim, a área da sombra é

x y zN i N j N k N N N .⋅ + ⋅ + ⋅ = + +

Como a média aritmética é menor ou igual à média quadrática, temos

2 2 2 13 3 3

x y z x y zN N N N N N+ + + +≤ =

3x y zN N N+ + ≤ .

Logo, a maior área possível é 3 , que ocorre quando a direção N dos raios solares é paralela à reta HA que liga dois vértices opostos do cubo. PROBLEMA 3: SOLUÇÃO DA BANCA Precisamos supor que 2 1n .> De fato, se 1 2n = e 2 1n ,= nem todas as entradas de

1A− são positivas. Pedimos desculpas... Vamos encontrar o inverso de 4

A I X ,= − onde todas as entradas de X são 0 ou

14 . Vamos usar a série ( ) 1 2 3I X I X X X ...−− = + + + + (note que

( )( )2 1n nI X I X X ... X I X +− + + + + = − ). Vamos mostrar inicialmente que essa

série converge. Para isso, vamos mostrar (ao final da solução) que o menor M ∈R tal que Xv M v≤ para todo vetor v em nR (aqui ||.|| denota a norma euclidiana

usual) é menor que 1. Daí segue que k kX v M v≤ para todo vetor v em nR e

todo inteiro positivo k, e logo a série claramente converge. Considere o grafo cujos vértices são 1 2 nv ,v ,...,v e conectamos iv e jv se e somente

se a entrada ijX em X é 14 . Pela definição da matriz A, esse grafo pode ser

decomposto em diversos caminhos, como os seguintes: 11 2 1, n( v ,v ...,v ),−

( )1 1 11 21 1 1 1kn kn , k n( v ,v ...,v ), k n+ + − ≤ ≤ − e 1 1 12 11r r n r n r n n( v ,v ,v ,...v ), r n .+ + + − ≤ ≤

Claramente esse grafo é conexo. Como a entrada ijm em kX é não nula se e somente se existe um caminho de i a j com k lados, para quaisquer i, j existe k tal que a entrada correspondente ijm em kX é não nula (e logo positiva). Isso prova

que todas as entradas de 1A− são positivas. Finalmente, para mostrar que o menor M ∈R tal que Xv M v≤ para todo vetor v em nR é menor que 1, consideremos um vetor 1 2 nv ( a ,a ,...,a )= tal que

Page 133: Eureka 2011

Sociedade Brasileira de Matemática

EUREKA! N°34, 2011

71

Xv M v= . Note que, como X é simétrica e não-negativa, podemos tomar um tal v com Xv=Mv. Temos que, para cada j n≤ , a j-ésima coordenada de Xv é da

forma 1 2

4j j js( j )r r ra a ... a+ + +

, onde 4s( j ) ≤ , cujo quadrado é menor ou igual a

( ) 1 2 1 2

2 2 2 2 2 2

16 4j j js( j ) j j js( j )r r r r r ra a ... a a a ... a

s j+ + + + + +

≤ , e, se vale a igualdade, todos

os jira devem ser iguais. Somando os termos 1 2

2 2 2

4j j js ( j )r r ra a ... a+ + +

para todos os

j n≤ , o resultado é menor ou igual a 2 2 21 2 na a ... a+ + + , pois em cada coluna de X

há no máximo 4 elementos não nulos. Daí segue que 2 2Xv v≤ e portanto 1M ≤ . Suponha por absurdo que M = 1, isto é, que valha a igualdade. Para cada j com

0ja ≠ , todos os jira devem ser iguais a ja (e portanto não nulos). Pela conexidade

do grafo definido acima , deveríamos ter então todos os ja iguais, mas nesse caso não vale a igualdade, pois nem todas as colunas têm 4 entradas não nulas. PROBLEMA 4: SOLUÇÃO DE MATEUS OLIVEIRA DE FIGUEIREDO (FORTALEZA – CE) a)

i) O polinômio xj

possui grau j já que é o produto de j polinômios de grau 1.

ii) Dado n +∈ provemos que todo polinômio de grau n pode ser escrito de

maneira única como combinação linear dos xj

.

Seja nP o espaço vetorial de todos os polinômios de grau n.≤ Uma base trivial para esse espaço é: { }0 1 2 nx ,x ,x ,...,x , já que 0 1

0 1 0 1 20 0nn na x a x ... a x a a a ... a+ + + ≡ ⇒ = = = = = e

qualquer polinômio de grau menor ou igual a n pode ser escrito como combinação linear deles. Assim, a dimensão de nP é n + 1.

iii) Se escrevermos os xj

nessa base temos:

Page 134: Eureka 2011

Sociedade Brasileira de Matemática

EUREKA! N°34, 2011

72

1 2 01 2 0

1 j j jj j

xx b x b x ... b x

j j !− −

− −

= + + + +

Assim, note que para escrever xj

só precisamos dos vetores { }0 1 2 jx ,x ,x ,...,x

pois por i) ele possui grau j e o coeficiente de jx é 1 .j !

Escrevendo matricialmente

( )

1! 1 2

111 ! 2 3

212! 3

0

0 1

0 02

0 1

0

nn j j

nn j j

nn j

M

xn

b b xxc c nx

x xdn

x

x

− −

−− − −

−− −

− = −

Como abaixo da diagonal principal só temos zeros, utilizando Laplace é fácil ver que o determinante da matriz é:

det M = ( ) ( )

1 1 1 1 0! 1 ! 2 ! 1!

...n n n

⋅ ⋅ ≠− −

Como Det M 0≠ a matriz possui inversa, ou seja, os jx podem ser escritos como

combinação linear dos xj

. Logo o conjunto 0 1x x x

S , ,...,n

=

gera nP e

como possui n + 1 elementos é base. Portanto, todo polinômio de nP pode ser escrito como combinação linear dos elementos de S.

Podemos estender a propriedade para 0x

S' jj

= ≥

já que, para j n,>

xdeg n

j

>

e portanto não aparecerá na combinação linear para escrever um

polinômio de nP .

Page 135: Eureka 2011

Sociedade Brasileira de Matemática

EUREKA! N°34, 2011

73

Mas como n foi pego genérico, qualquer polinômio pode ser escrito de manera

única como combinação dos x

.j

b)

i) Vamos escrever x

xi

na base S´.

( )( ) ( )1 2 1!

x x x x x ... x ix

i i⋅ − − − −

= =

( ) ( )( ) ( )1 2 1!

x i i x x x ... x ii

− + − − − +=

( )( )( ) ( )( ) ( )( ) ( )( )

1 2 3 1 1 2 1! !

xi

x x x x ... x i x i x x x ... x ii

i i− − − − + − − − − +

= + =

( )( ) ( )( )( )

( )

( )

1

1 2 1 11

11 !x

i

x x xx x x ... x i ii i i

i i ii

+

− − − + + = + = + + ++

( ) 1

1x x x

x i ii i i

⇒ = + + +

.

ii) Sabemos que:

0

nn

i

n xx .

i i=

=

∑ Multiplicando por x temos:

( )1

0 0

1

1

n ¨ nn

i i

n x n x xx x i i

i i i i i+

= =

= = + + +

∑ ∑ (I)

Por outro lado, 1

1

0

1

nn

i

n xx

i i

++

=

+ =

∑ (II)

Como todo polinômio não nulo pode ser escrito de maneira única por S´, (I) e (II) devem ter os mesmos coeficientes.

Olhando para o coeficiente de

1x

k +

em (I) temos ( )

1n n

k .k k i

+ + +

E em II o coeficiente é 11

nk

++

.

Page 136: Eureka 2011

Sociedade Brasileira de Matemática

EUREKA! N°34, 2011

74

Logo ( ) 1

11 1

n n nk

k k k +

+ + = + + ⇒

1 1

1 11

n nk k

n kk

++

=+ ++

.

PROBLEMA 5: SOLUÇÃO DE RAMON MOREIRA NUNES (FORTALEZA – CE) Vamos proceder por indução em #F. No caso inicial F tem um elemento. Trivial:

( )( ) ( )( ) ( )( )3

39 03r r r

RVol V F r , r Vol V F Vol V F , r R.r

= π ∀ ∈ ⇒ = ⋅ ∀ < <

Agora, suponha que se F possui k pontos então vale o resultado. Se F é um conjunto de ( )1k + pontos, escreva { }1 1k kF x ,...,x ,x .+=

Se { }1 kF x ,...,x ,= vale o resultado para F i.e.

( )( )3

rVol V F

r é função decrescente de r.

Como ( )( ) ( )( ) ( )( ) ( ( ) ( )11

3 3 3 3

decrescente constante

r r r kr r kVol V F Vol V F B xVol V F Vol B x

r r r r++

∩= + −

E a primeira função é decrescente, a segunda é constante igual a 43

,π basta provar

que a última é crescente ( ) ( )( )1

3

r r kVol V F B x.

r+

Para fazer isso suponha sem

perda de generalidade que 1kx + é a origem de 3 i.e. ( )1 0 0 0 0kx , , .+ = =

Considere a homotetia rT de centro O e razão 1 .r

Temos

( )( ) ( )( ) ( )1 1 rr r r rxT B x B ,T V F V F ,r

= =

onde 1 kr

xxF ,..., .r r

=

Como

( ) ( )3

1rVol T A Vol A

r= para qualquer A pois

Page 137: Eureka 2011

Sociedade Brasileira de Matemática

EUREKA! N°34, 2011

75

( )( ) ( )3 3

1 1det T r

r rT A A AVol T A dx dx dx Vol A ,

r r= = = =∫ ∫ ∫ temos

( ) ( )( ) ( ) ( )( )1 13

00

r r

r

Vol V F BVol V F B .

r

∩= ∩ Portanto, basta mostrar que

( ) ( )( )1 1 0rr Vol V F B→ ∩ é crescente. Basta para isso mostrar que

( ) ( ) ( ) ( )1 1 1 10 0r RV F B V F B∩ ⊂ ∩ se r R.< Para isso, usaremos o seguinte lema:

Lema: Dado 3y ∈ fixado, a função x x y→ − é convexa, isto é,

( )( ) ( ) [ ]1 2 21 1 0 1tx t x y t x y t x y ,t , .+ − − ≤ − + − − ∈

Prova: Dados 31 2x ,x ,∈ temos

( )( ) ( ) ( ) ( ) ( )( )1 2 1 2 1 21 1 1 1tx t x y tx t x ty t y t x y t x y .+ − − = + − − − − = − + − −

Pela desigualdade triangular, isso é menor ou igual a ( ) ( )( )1 21t x y t x y .− + − −

Como a norma é homogênea, ( )( ) ( )1 2 1 21 1tx t x y t x y t x y .+ + − ≤ − + − −

Agora voltemos à prova de que ( ) ( ) ( ) ( )1 1 1 10 0r RV F B V F B .∩ ⊂ ∩ Tome

( ) ( )1 1 0ra V F B .∈ ∩ Então, i k∃ ≤ tal que 1ixar

− < e 1a .<

Como R r,> ixR

está no intervalo 0 ix, .r

Vamos usar a convexidade de

x x a ;→ − escolha [ ] ( )0 1 tal que 1 0i ix xrt , t t .R R r

= ∈ = + − ⋅ Temos

( ) ( )1 0 1 1 1 1i ix xa t a t a t t .R r

− ≤ − + − − < ⋅ + − ⋅ = Ou seja, ( )1 1i

Rxa B V F .R

∈ ⊂

Como ( ) ( ) ( )1 1 10 0Ra B ,a V F B .∈ ∈ ∩

Ou seja, acabamos de provar que ( ) ( ) ( ) ( )1 1 1 10 0r RV F B V F B∩ ⊂ ∩ . Como já

vimos, isso implica que ( ) ( )( )1

3

r r kVol V F B xr

r+∩

→ é crescente, e como também

já vimos isso implica que ( )( )

3rVol V F

rr

→ é decrescente. Concluímos.

Page 138: Eureka 2011

Sociedade Brasileira de Matemática

EUREKA! N°34, 2011

76

PROBLEMA 6: SOLUÇÃO DE RÉGIS PRADO BARBOSA (FORTALEZA – CE) Temos n,k +∈ e p primo com 60 7p k= + tal que 210 8 10 1n np + ⋅ + . Queremos

provar que n,k são pares. Temos

( ) ( ) ( )2 22 210 8 10 1 10 2 10 1 6 10 10 1 6 10 10 1n n n n n n n n+ ⋅ + = + ⋅ + + ⋅ = + + ⋅ ⇒ + ≡

( )6 10n mod p≡ − ⋅ ⇒ 6 10 1

n

p − ⋅

=

onde

0 se 1 se é resíduo quadrático módulo .

1 se não é resíduo quadrático módulo .

, p aa , a pp

, a p

= −

Veja que claramente 16 10 2 3 5n n np \ +/ − ⋅ = − ⋅ ⋅ pois 60 7p k= + 2 60 3 60 5 60, ,→ e 2 7 3 7 5 7 2 3 e 5\ , \ , \ p , ./ / / → ≠ Sabemos que o símbolo de Legendre possui a seguinte propriedade:

a b a b .p p p

⋅= ⋅

Logo: 1

6 10 1 2 3 5 1n nn

p p p p p

+ − ⋅ −

= ⋅ =

(simplesmente separei os fatores).

Calculemos cada um deles:

( ) ( ) ( ) ( )1 60 7 1 30 3

2 21 11 1 1 1

p k k

p p

− + −+ − −

→ = − = − → = − = − ⇒

( )1 1p

−= −

(I)

( )2 18

2 1p

.p

− → = −

Temos 2 260 7 3600 2 60 7 49p k p k k= + ⇒ = + ⋅ ⋅ + ⇒

2 21 3600 840 488 8

p k k− + += .

22 2

par

1 450 105 6 450 104 68

p k k k k k. −

⇒ = + + = + + +

Assim,

( ) ( )( )2

21 450 104 681 1

p k k k−+ + +− = − ( ) ( )( ) ( )

221 450 104 6

81 1 1p k k k−

+ +⇒ − = − ⋅ − ⇒ ( )2

1 k

p

= −

(II)

Page 139: Eureka 2011

Sociedade Brasileira de Matemática

EUREKA! N°34, 2011

77

Para calcular 3p

e 5p

precisaremos da lei da Reciprocidade Quadrática: dados

p, q primos ímpares: ( )1 1

2 21p qp q

q p

− −⋅

= −

Assim: com q = 3, ( ) ( ) ( ) ( )3 1 1 1 30 3

2 23 31 1 1

3 3

p kp p .p p

− −⋅ ⋅ + ⋅ = − ⇒ ⋅ = − = −

Assim precisamos de calcular 60 7 1 1 13 3 3 3p k p+ = = = → =

(usando que

( )260 7 1 1 3k mod+ ≡ ≡ ).

Substituindo acima tem-se: ( )3 1 1p

⋅ = − ⇒

( )3 1

p

= −

(III)

Com q = 5, ( ) ( ) ( )5 1 1 2 30 32 2

5 51 1 15 5

p kp p .p p

− −⋅ ⋅ + ⋅ = − ⇒ ⋅ = − =

Assim

precisamos calcular o valor de ( ) ( )60 7 2 1 15 5 5 5p k p+ = = = − → = −

(pois

( )60 7 2 5k mod+ ≡ e os resíduos quadráticos módulo 5 são 0, 1, 4 já que

( )0 1 2 3 4 5x , , , , mod≡ → ( )2 0 1 4 4 1 5x , , , , mod≡ ).

Substituindo acima ( )5 51 1 15p

p p ⋅ = → ⋅ − = →

( )

51

p

= −

(IV)

Juntam-se (I), (II), (III) e (IV) em (*):

( ) ( ) ( ) ( ) ( ) ( ) ( )1

1 11 2 3 51 1 1 1 1 1 1 1

n nk n n k n n

p p p p

++ + +−

⋅ ⋅ = ⇒ − ⋅ − ⋅ − ⋅ − = ⇒ − =

( )1k n n⇒ + + é par.

⇒ se n é ímpar ( )1n→ + par ( )1k n→ + par ( )1k n n⇒ + + ímpar. Absurdo!

Logo n é par ( )1n→ + ímpar.

⇒ se k é ímpar ( )1k n→ + ímpar ( )1k n n→ + + ímpar. Absurdo! Logo k é par . Assim, dados n,k +∈ com 60 7p k= + primo tal que 210 8 10 1n np + ⋅ + então n e

k são pares.

Page 140: Eureka 2011

Sociedade Brasileira de Matemática

EUREKA! N°34, 2011

78

XXXII OLIMPÍADA BRASILEIRA DE MATEMÁTICA Premiados

NÍVEL 1 (6º. e 7º. Anos) NOME CIDADE – ESTADO PRÊMIO Ana Emília Hernandes Dib S.J. do Rio Preto - SP Ouro Pedro Henrique Alencar Costa Fortaleza - CE Ouro Ryunosuke Watanabe Tagami Rio Claro - SP Ouro Helena Veronique Rios São Carlos - SP Ouro Italo Lesione de Paiva Rocha Fortaleza - CE Ouro José Henrique Carvalho Curitiba - PR Ouro Juliana Bacelar de Freitas Brasília - DF Prata Daniel Lima Braga Eusébio - CE Prata Hermes Lins e Nascimento Fortaleza - CE Prata Laís Monteiro Pinto Rio de Janeiro - RJ Prata Lucca Morais de Arruda Siaudzionis Fortaleza - CE Prata Leandro Alves Cordeiro Ribeirão Pires - SP Prata Henrique Gontijo Chiari Belo Horizonte - MG Prata André Akinaga Benites São Paulo - SP Prata Gabriel Diniz Vieira e Sousa Fortaleza - CE Prata Rafael Seiji Uezu Higa São Paulo - SP Prata Adriana de Sousa Figueiredo Porto Alegre - RS Prata Gustavo Figueiredo Serra São Paulo - SP Prata Matheus Uchôa Constante Goiânia - GO Bronze Kristian Holanda Nogueira Manaus - AM Bronze Fábio Itikama São Paulo - SP Bronze Loic Dominguez Fortaleza - CE Bronze Jiang Zhi São Paulo - SP Bronze Ricardo Ken Wang Tsuzuki São Paulo - SP Bronze Ana Caroline Obana da Cruz Curitiba - PR Bronze Ana Paula Lopes Schuch Porto Alegre - RS Bronze José Marcio Machado de Brito Cocal dos Alves - PI Bronze Lucas Bastos Germano Fortaleza - CE Bronze Victória Moreira Reis Cogo Teresina - PI Bronze Thiago Araujo Oliveira Jaboatão dos Guararapes - PE Bronze Gabriel Toneatti Vercelli Osasco - SP Bronze Nathan Bonetti Teodoro Curitiba - PR Bronze Jefferson Daxian Hong São Paulo - SP Bronze Cristóbal Sciutto Rodriguez São Paulo - SP Bronze Aruana Almeida Correa Porto Alegre - RS Bronze Cynthia Lacroix Herkenhoff Vitória - ES Bronze Kaíque Maestrini Sacchi São Paulo - SP Menção Honrosa Igor de Lacerda Curitiba - PR Menção Honrosa

Page 141: Eureka 2011

Sociedade Brasileira de Matemática

EUREKA! N°34, 2011

79

Rafael Reple Geromee São Paulo - SP Menção Honrosa Leonardo de Matos Felippetti Mariano Curitiba - PR Menção Honrosa Gabriel Passamani Correa Vitória - ES Menção Honrosa Daniel de Almeida Souza Brasília - DF Menção Honrosa Diego Teixeira Nogueira Fidalgo Salvador - BA Menção Honrosa Natan Novellu Tu São Paulo - SP Menção Honrosa Ricardo Borsari Brinati São Paulo - SP Menção Honrosa Rafael Neves Vieira Brasília - DF Menção Honrosa Juliano Pecica Negri Piracicaba - SP Menção Honrosa Gustavo Rodrigues Machado Sorocaba - SP Menção Honrosa Zoltan Flamarion Glueck Carvalho Belo Horizonte - MG Menção Honrosa Gabriel Ribeiro Barbosa Fortaleza - CE Menção Honrosa Pedro Henrique Rocha de Freitas Brasília - DF Menção Honrosa Pedro Henrique Sacramento de Oliveira Loureira - SP Menção Honrosa Guilherme Goulart Kowalczuk Porto Alegre - RS Menção Honrosa Pedro de Vasconcellos Oporto Nova Lima - MG Menção Honrosa Aryssa Victoria Shitara São Paulo - SP Menção Honrosa Ives Vaz Caldeira Lopes São Paulo - SP Menção Honrosa Marcos Vinícius de Oliveira Soares Rio de Janeiro - RJ Menção Honrosa Jéssica Carolina Zilio Piracicaba - SP Menção Honrosa João Pedro Graça Melo Vieira Rio de Janeiro - RJ Menção Honrosa Henrique Medici Pontieri Campo Grande - MS Menção Honrosa Gabriel Caino Castilho Rodrigues Salvador - BA Menção Honrosa Tamara P. de A. Moraes Feira de Santana - BA Menção Honrosa Karine Quaresma Lima Taguatinga - DF Menção Honrosa Natália Brasileiro Lins Barbosa Jaboatão dos Guararapes - PE Menção Honrosa Lucki Li São Paulo - SP Menção Honrosa Heloísa Antunes de Medeiros Itamogi - MG Menção Honrosa Iuri Grangeiro Carvalho Fortaleza - CE Menção Honrosa Lara Sampaio Pinheiro de Freitas Olinda - PE Menção Honrosa Maria Júlia Costa Medeiros Fortaleza - CE Menção Honrosa Kevin Korpasch Guarapuana - PR Menção Honrosa Sofía Leite Correia Lima Fortaleza - CE Menção Honrosa João Baptista de Paula e Silva Belo Horizonte - MG Menção Honrosa Bernardo Puetter Schaeffer Rio de Janeiro - RJ Menção Honrosa Júlia Bertelli Joinville - SC Menção Honrosa Rafael Purim de Azevedo Pirassununga - SP Menção Honrosa Pedro Henrique da Silva Dias Porto Alegre - RS Menção Honrosa Marcelo Bandeira de Melo Boavista Teresina - PI Menção Honrosa Gabriel Branco Frizzo Curitiba - PR Menção Honrosa Maria Eduarda Müller Eyng Porto Alegre - RS Menção Honrosa Henrique Martínez Rocamora São Bernardo do Campo - SP Menção Honrosa Felipe Roz Barscevicius Sorocaba - SP Menção Honrosa João Vitor Vaz Oliveira Recife - PE Menção Honrosa Mateus Siqueira Thimoteo Mogi das Cruzes - SP Menção Honrosa

Page 142: Eureka 2011

Sociedade Brasileira de Matemática

EUREKA! N°34, 2011

80

Ebenezeer Pinto Banoeira Neto Fortaleza - CE Menção Honrosa Maria Clara Vasconcelos Andrade Brasília - DF Menção Honrosa Rafael Beck Salvador - BA Menção Honrosa Arthur Monteiro Dos Santos Salvador - BA Menção Honrosa Júlia Wotzasek Pereira São Paulo - SP Menção Honrosa Gabriel Oliveira Rigo Cotia - SP Menção Honrosa Leonardo Galante Barco São Paulo - SP Menção Honrosa Bruno Scatolini São Paulo - SP Menção Honrosa Lucas Pereira Galvão de Barros São Paulo - SP Menção Honrosa Vítor Ossamu Rodrigues Okamura Brasília DF Menção Honrosa

NÍVEL 2 (8º. e 9º. Anos)

NOME CIDADE – ESTADO PRÊMIO Rafael Rodrigues Rocha de Melo Caucaia - CE Ouro Vinícius Canto Costa Rio de Janeiro - RJ Ouro Henrique Vieira G. Vaz São Paulo - SP Ouro Fellipe Sebastiam da Silva P. Pereira Rio de Janeiro - RJ Ouro Roberto Tadeu Abrantes de Araújo Rio de Janeiro - RJ Ouro Pedro Victor Falci de Rezende Santo Antonio - MG Ouro Alessandro A. de Oliveira Pacanowski Rio de Janeiro - RJ Prata Lincoln de Queiroz Vieira Fortaleza - CE Prata Tadeu Pires de Matos Belford Neto Fortaleza - CE Prata Vitor Ramos de Paula Belo Horizonte - MG Prata Francisco Markan N. de Souza Filho Fortaleza - CE Prata Jair Gomes Soares Júnior Montes Claros - MG Prata Breno Soares da Costa Vieira J. dos Guararapes - PE Prata Gabriel José Moreira da Costa Silva Maceió - AL Prata Pedro Morais de Arruda Siaudzionis Fortaleza - CE Prata Gabriel Sena Galvão Guará - DF Prata Fabio da Silva Soares Planaltina - DF Prata Michel Rozenberg Zelazny São Paulo - SP Prata Bruno Eidi Nishimoto Jales - SP Prata Franco Matheus de Alencar Severo Rio de Janeiro - RJ Prata Aimê Parente de Sousa Fortaleza - CE Prata Marcos Paulo Nunes de Lima Silva Rio de Janeiro - RJ Bronze Gabriel N. Coelho de Togni de Souza Rio de Janeiro - RJ Bronze Rafael T. Eugênio Pontes Barone Aracatuba - SP Bronze Murilo Corato Zanarella Amparo - SP Bronze Rodrigo Sanches Angelo São Paulo - SP Bronze Alexandre Perozim de Faveri Neves Paulista - SP Bronze Luíze Mello D'urso Vianna Rio de Janeiro - RJ Bronze Maria Clara Cardoso São Paulo - SP Bronze Liara Guinsberg São Paulo - SP Bronze Lucas Cawai Julião Pereira Caucaia - CE Bronze

Page 143: Eureka 2011

Sociedade Brasileira de Matemática

EUREKA! N°34, 2011

81

Luis Guilherme Gomes Aguiar Rio de Janeiro - RJ Bronze Carlos Adriano Vieira Igarapé - MG Bronze Daniel Santana Rocha Rio de Janeiro - RJ Bronze Raphael Mendes de Oliveira Rio de Janeiro - RJ Bronze Samuel Brasil de Albuquerque Fortaleza - CE Bronze Gustavo Souto Henriques Campelo João Pessoa - PB Bronze Lucas de Moura Herlin Rio de Janeiro - RJ Menção Honrosa Vitor Dias Gomes Barrios Marin Presidente Prudente - SP Menção Honrosa João Pedro Sedeu Godoi Rio de Janeiro - RJ Menção Honrosa Suzane Eberhart Ribeiro da Silva Campo Grande - MS Menção Honrosa Ícaro Sampaio Viana Fortaleza - CE Menção Honrosa Pedro Henrique Bortolozo Maria Colombo - PR Menção Honrosa Fábio Kenji Arai São Paulo - SP Menção Honrosa Guilherme de Oliveira Rodrigues Fortaleza - CE Menção Honrosa Alexandre Mendonça Cardoso Salvador - BA Menção Honrosa Leyberson Pereira Assunção Fortaleza - CE Menção Honrosa Rubens Martins Bezerra Farias Sobral - CE Menção Honrosa João Vítor Fernandes Paiva Rio de Janeiro - RJ Menção Honrosa Bruno Almeida Costa Fortaleza - CE Menção Honrosa Daniel Lima Santanelli Rio de Janeiro - RJ Menção Honrosa Marília Nascimento Monteiro Recife - PE Menção Honrosa Igor Albuquerque Araujo Rio de Janeiro - RJ Menção Honrosa Josué Knorst Picada Café - RS Menção Honrosa Ricardo Vieira Marques Brasília - DF Menção Honrosa Júlio César de Barros Santo André - SP Menção Honrosa Thomas Akio Ikeda Valvassori Mogi das Cruzes - SP Menção Honrosa Gabriel Fazoli Domingos Urupês - SP Menção Honrosa Henrique Luan Gomes Pereira Braga Belem - PA Menção Honrosa Beatriz Yumi Ota São Paulo - SP Menção Honrosa Kiane Sassaki Menezes Rio de Janeiro - RJ Menção Honrosa Eric Gripa Marques Rio de Janeiro - RJ Menção Honrosa Samuel Kuo Chen Shao São Paulo - SP Menção Honrosa Pedro Henrique Jagosenit Vilaça Santa Branca - SP Menção Honrosa Caio de Souza Câmara Manaus - AM Menção Honrosa Lucas David Noveline Belem - PA Menção Honrosa Lucas Rebelo Vieira da Silva Recife - PE Menção Honrosa Elias Brito Oliveira Brasília - DF Menção Honrosa Guilherme Ryu Odaguiri Kobori São Paulo - SP Menção Honrosa Mariana Souza de Araújo Recife - PE Menção Honrosa Francisco Cláudio Coelho Rio de Janeiro - RJ Menção Honrosa Murilo Leão Pereira Belem - PA Menção Honrosa Jadi Diniz Guimarães de Queiroz Recife - PE Menção Honrosa Caio Lima Albuquerque São Paulo - SP Menção Honrosa Carolina Lima Guimarães Vitória - ES Menção Honrosa

Page 144: Eureka 2011

Sociedade Brasileira de Matemática

EUREKA! N°34, 2011

82

Nível 3 (Ensino Médio) NOME CIDADE – ESTADO PRÊMIO Gustavo Lisbôa Empinotti Florianópolis - SC Ouro Marcelo Tadeu de Sá Oliveira Sales Salvador - BA Ouro João Lucas Camelo Sá Fortaleza - CE Ouro Hanon Guy Lima Rossi São Paulo - SP Ouro Maria Clara Mendes Silva Pirajuba - MG Ouro Matheus Secco Torres da Silva Rio de Janeiro - RJ Prata Lucas Lourenço Hernandes São Paulo - SP Prata Deborah Barbosa Alves São Paulo - SP Prata Henrique G. Fiuza do Nascimento Brasília - DF Prata Luiz Filipe Martins Ramos Niterói - RJ Prata André Macieira Braga Costa Belo Horizonte - MG Prata Thiago Saksanian Hallak São Paulo - SP Prata Victor Juca Martins Fortaleza - CE Prata Caíque Porto Lira Fortaleza - CE Prata Gustavo H. F. e Sampaio Braga São José dos Campos-SP Prata Alvaro Lopes Pedroso Santa Isabel - SP Prata André Amaral de Sousa Diadema - SP Prata Marcos Massayuki Kawakami São Paulo - SP Bronze Carlos Henrique de Andrade Silva Fortaleza - CE Bronze Rafael Kazuhiro Miyazaki São Paulo - SP Bronze André Saraiva Nobre dos Santos Fortaleza - CE Bronze Daniel Eiti Nishida Kawai Atibaia - SP Bronze Lucas de Freitas Smaira Guaxupé - MG Bronze Cássio dos Santos Sousa Osasco - SP Bronze Alessandro Macêdo de Araújo Fortaleza - CE Bronze Breno Vieira da Silva Passos Aracaju - SE Bronze Iago Dalmaso Brasil Dias Rio de Janeiro - RJ Bronze Isabella Amorim Gonçalez Marília - SP Bronze Daniel dos Santos Bossle Porto Alegre - SP Bronze Davi Coelho Amorim Fortaleza - CE Bronze Lucas Mestres Mendes Fortaleza - CE Bronze Vinícius Gomes Pereira Rio de Janeiro - RJ Bronze Renan Pablo da Cruz Fortaleza - CE Bronze Jonas Rocha Lima Amaro Fortaleza - CE Bronze Iuri Rezende Souza Mineiros - GO Bronze Matheus Araújo Marins São Gonçalo - RJ Menção Honrosa Felipe Vieira de Paula Fortaleza - CE Menção Honrosa Rafael Farias Marinheiro Recife - PE Menção Honrosa Elvis Falcao de Araujo Fortaleza - CE Menção Honrosa Pablo Almeida Gomes Santana de Pirapama - MG Menção Honrosa Paulo Gabriel Ramos Monteiro Rio de Janeiro - RJ Menção Honrosa Victor de Oliveira Bitarães Betim - MG Menção Honrosa

Page 145: Eureka 2011

Sociedade Brasileira de Matemática

EUREKA! N°34, 2011

83

Daniel Caueh Dunaiski Figueira Leal Curitiba - PR Menção Honrosa Raphael Julio Barcelos Taguatinga - DF Menção Honrosa Fernando Fonseca Andrade Oliveira Belo Horizonte - MG Menção Honrosa Felipe Mendes dos Santos Gama - DF Menção Honrosa Felipe Abella C. Mendonça de Souza João Pessoa - PB Menção Honrosa Francisco Raul Lobo Rodrigues Fortaleza - CE Menção Honrosa Gabriel Leite de Carvalho Fortaleza - CE Menção Honrosa André Austregesilo Scussel Fortaleza - CE Menção Honrosa Victorio Takahashi Chu São Paulo - SP Menção Honrosa Victor José Tiburtius Franco Recife - PE Menção Honrosa Matheus Cavalcante Lima Fortaleza - CE Menção Honrosa Cleberton de Santana Oliveira São Miguel do Aleixo - SE Menção Honrosa Mauro Brito Júnior Fortaleza - CE Menção Honrosa Gabriel José Guimarães Barbosa Pequi - MG Menção Honrosa Lucas Colucci Cavalcante de Souza São Paulo - SP Menção Honrosa Sarah Villanova Borges Juiz de Fora - MG Menção Honrosa Ivan Tadeu Ferreira Antunes Filho Lins - SP Menção Honrosa Dalton Felipe de Menezes São José dos Campos-SP Menção Honrosa Thiago de Paula Vasconcelos Fortaleza - CE Menção Honrosa Jardiel Freitas Cunha Recife - PE Menção Honrosa Ana Beatriz Prudêncio de A. Rebouças Fortaleza - CE Menção Honrosa Rafael Sussumu Yamaguti Miada Valinhos - SP Menção Honrosa Davi Sampaio de Alencar Fortaleza - CE Menção Honrosa Bruno Ferri de Moraes São Paulo - SP Menção Honrosa

Nível Universitário

NOME CIDADE – ESTADO PREMIO

Rafael Tupynambá Dutra Belo Horizonte - MG Ouro

Renan Henrique Finder Rio de Janeiro - RJ Ouro

Regis Prado Barbosa Fortaleza - CE Ouro

Ramon Moreira Nunes Fortaleza - CE Ouro

Thomás Yoiti Sasaki Hoshina Rio de Janeiro - RJ Ouro

Guilherme Rodrigues N. de Souza S.J. dos Campos - SP Prata

Jorge Henrique Craveiro de Andrade Rio de Janeiro - RJ Prata

Rafael Assato Ando Campinas - SP Prata

Gabriel Luís Mello Dalalio S.J. dos Campos - SP Prata

Charles Barbosa de Macedo Brito Rio de Janeiro - RJ Prata

Leonardo Ribeiro de Castro Carvalho S.J. dos Campos - SP Prata

Marcelo Matheus Gauy São José do Rio Preto-SP Prata

Leandro Farias Maia Rio de Janeiro - RJ Prata

Adenilson Arcajo de Moura Júnior Fortaleza - CE Bronze

Paulo André Carvalho de Melo Piedade - RJ Bronze

Joas Elias dos Santos Rocha Muribeca - SE Bronze

Page 146: Eureka 2011

Sociedade Brasileira de Matemática

EUREKA! N°34, 2011

84

Guilherme Lourenço Mejia S.J. dos Campos - SP Bronze

Reinan Ribeiro Souza Santos Lagarto - SE Bronze

Rafael Alves da Ponte Fortaleza - CE Bronze

Davi Lopes Alves de Medeiros Fortaleza - CE Bronze

Luca Mattos Moller Nova Friburgo - RJ Bronze

Renato Rebouças de Medeiros Fortaleza - CE Bronze

Danilo Furlan Kaio São Paulo - SP Bronze

Rafael Endlich Pimentel Vitória - ES Bronze

Paulo Sérgio de Castro Moreira Fortaleza - CE Bronze

Carlos Coelho Lechner Rio de Janeiro - RJ Menção Honrosa

Thiago Ribeiro Ramos Varginha - MG Menção Honrosa

Hugo Fonseca Araújo Rio de Janeiro - RJ Menção Honrosa

Alysson Espíndola de Sá Silveira Fortaleza - CE Menção Honrosa

Jordan Freitas Piva Rio de Janeiro - RJ Menção Honrosa

Érik Fernando de Amorim Araraquara - SP Menção Honrosa

Daniel Ungaretti Borges Belo Horizonte - MG Menção Honrosa

Antônio Deromir Neves Silva Júnior Fortaleza - CE Menção Honrosa

Rafael Parpinel Cavina São Paulo - SP Menção Honrosa

Isaque Santa Brigida Pimentel Barcarena - PA Menção Honrosa

Mateus Oliveira de Figueiredo Fortaleza - CE Menção Honrosa

Davi Dos Santos Lima Maceió - AL Menção Honrosa

Bruno da Silva Santos Belford Roxo - RJ Menção Honrosa

Francisco Osman Pontes Neto Fortaleza - CE Menção Honrosa

Breno Vieira de Aguiar Rio de Janeiro - RJ Menção Honrosa

Ricardo Turolla Bortolotti Rio de Janeiro - RJ Menção Honrosa

Guilherme Philippe Figueiredo São Paulo - SP Menção Honrosa

Daniel de Barros Soares Rio de Janeiro - RJ Menção Honrosa

Hudson do Nascimento Lima Fortaleza - CE Menção Honrosa

Eduardo Fischer Encantado - RS Menção Honrosa

Luty Rodrigues Ribeiro Fortaleza - CE Menção Honrosa

José Leandro Pinheiro Fortaleza - CE Menção Honrosa

Caio Ishizaka Costa S.J. dos Campos - SP Menção Honrosa

Gabriel Caser Brito Rio de Janeiro - RJ Menção Honrosa

Leonardo Donisete da Silva Campinas - SP Menção Honrosa

Alan Anderson da Silva Pereira União dos Palmares - AL Menção Honrosa

Diego Andrés de Barros Lima Barbosa Rio de Janeiro - RJ Menção Honrosa

Renato Dias Costa Rio de Janeiro - RJ Menção Honrosa

Ivan Guilhon Mitoso Rocha Fortaleza - CE Menção Honrosa

Willy George do Amaral Petrenko Rio de Janeiro - RJ Menção Honrosa

Leonardo Borges Avelino Rio de Janeiro - RJ Menção Honrosa

Jose Armando Barbosa Filho Fortaleza - CE Menção Honrosa

Page 147: Eureka 2011

Sociedade Brasileira de Matemática

EUREKA! N°34, 2011

85

AGENDA OLÍMPICA

XXXIII OLIMPÍADA BRASILEIRA DE MATEMÁTICA

NÍVEIS 1, 2 e 3 Primeira Fase – sábado, 18 de junho de 2011

Segunda Fase – sábado, 3 de setembro de 2011 Terceira Fase – sábado, 15 de outubro de 2011 (níveis 1, 2 e 3)

domingo, 16 de outubro de 2011 (níveis 2 e 3 - segundo dia de prova)

NÍVEL UNIVERSITÁRIO Primeira Fase – sábado, 3 de setembro de 2011

Segunda Fase – sábado, 15 e domingo, 16 de outubro de 2011

IV ROMANIAN MASTER OF MATHEMATICS (RMM) 23 a 28 de fevereiro de 2011(Bucareste, Romênia)

ASIAN PACIFIC MATH OLYMPIAD (APMO)

12 de março de 2011

XVII OLIMPÍADA DE MAIO 7 de maio de 2011

XXII OLIMPÍADA DE MATEMÁTICA DO CONE SUL

14 a 20 de agosto de 2011(La Paz, Bolívia)

LII OLIMPÍADA INTERNACIONAL DE MATEMÁTICA 13 a 24 de julho de 2011(Amsterdam, Holanda)

I OLIMPÍADA DE MATEMÁTICA DA LUSOFONIA

20 a 31 de julho de 2011(Coimbra, Portugal)

XVII OLIMPÍADA INTERNACIONAL DE MATEMÁTICA UNIVERSITÁRIA (IMC) 24 a 30 de julho de 2011(Blagoevgrad, Bulgária)

XXV OLIMPÍADA IBEROAMERICANA DE MATEMÁTICA

23 de setembro a 1 de outubro de 2011(São José, Costa Rica)

III COMPETIÇÃO IBEROAMERICANA INTERUNIVERSITÁRIA DE MATEMÁTICA 2 a 8 de outubro de 2011(Quito, Equador)

XIII OLIMPÍADA IBEROAMERICANA DE MATEMÁTICA UNIVERSITÁRIA

26 de novembro de 2011

Page 148: Eureka 2011

Sociedade Brasileira de Matemática

EUREKA! N°34, 2011

86

COORDENADORES REGIONAIS

Alberto Hassen Raad (UFJF) Juiz de Fora – MG Américo López Gálvez (USP) Ribeirão Preto – SP Antonio Carlos Nogueira (UFU) Uberlândia – MG Benedito Tadeu Vasconcelos Freire (UFRN) Natal – RN Bruno Holanda (CAEN – UFC) Fortaleza – CE Carmen Vieira Mathias (UNIFRA) Santa María – RS Claus Haetinger (UNIVATES) Lajeado – RS Cláudio de Lima Vidal (UNESP) S.J. do Rio Preto – SP Denice Fontana Nisxota Menegais (UNIPAMPA) Bagé – RS Disney Douglas Lima de Oliveira (UFAM) Manaus – AM Edson Roberto Abe (Colégio Objetivo de Campinas) Campinas – SP Edney Aparecido Santulo Jr. (UEM) Maringá – PR Fábio Brochero Martínez (UFMG) Belo Horizonte – MG Florêncio Ferreira Guimarães Filho (UFES) Vitória – ES Francinildo Nobre Ferreira (UFSJ) São João del Rei – MG Genildo Alves Marinho (Centro Educacional Leonardo Da Vinci) Taguatingua – DF Herivelto Martins (USP – São Carlos) São Carlos – SP Gilson Tumelero (UTFPR) Pato Branco – PR Ivanilde Fernandes Saad (UC. Dom Bosco) Campo Grande – MS João Benício de Melo Neto (UFPI) Teresina – PI João Francisco Melo Libonati (Grupo Educacional Ideal) Belém – PA Diogo Diniz (UFPB) Campina Grande – PB José Luiz Rosas Pinho (UFSC) Florianópolis – SC José Vieira Alves (UFPB) Campina Grande – PB José William Costa (Instituto Pueri Domus) Santo André – SP Krerley Oliveira (UFAL) Maceió – AL Licio Hernandes Bezerra (UFSC) Florianópolis – SC Luciano G. Monteiro de Castro (Sistema Elite de Ensino) Rio de Janeiro – RJ Luzinalva Miranda de Amorim (UFBA) Salvador – BA Marcelo Dias (Grupo Educacional Etapa) São Paulo – SP Marcelo Antonio dos Santos FACOS Osório – RS Marcelo Rufino de Oliveira (Grupo Educacional Ideal) Belém – PA Newman Simões (Cursinho CLQ Objetivo) Piracicaba – SP Nivaldo Costa Muniz (UFMA) São Luis – MA Osnel Broche Cristo (UFLA) Lavras – MG Uberlândio Batista Severo (UFPB) João Pessoa – PB Raul Cintra de Negreiros Ribeiro (Colégio Anglo) Atibaia – SP Reginaldo de Lima Pereira (Escola Técnica Federal de Roraima) Boa Vista – RR Reinaldo Gen Ichiro Arakaki (UNIFESP) SJ dos Campos – SP Ricardo Amorim (Centro Educacional Logos) Nova Iguaçu – RJ Ronaldo Alves Garcia (UFGO) Goiânia – GO Rogério da Silva Ignácio (Col. Aplic. da UFPE) Recife – PE Rosangela Ramon (UNOCHAPECÓ) Chapecó – SC Sérgio Cláudio Ramos (IM-UFRGS) Porto Alegre – RS Seme Gebara Neto (UFMG) Belo Horizonte – MG Tadeu Ferreira Gomes (UEBA) Juazeiro – BA Tomás Menéndez Rodrigues (U. Federal de Rondônia) Porto Velho – RO Valdenberg Araújo da Silva (U. Federal de Sergipe) São Cristóvão – SE Wagner Pereira Lopes (CEFET – GO) Jataí – GO Wanderson Breder (CEFET – RJ) Nova Friburgo – RJ William Serafim dos Reis (UFT – TO) Arraias – TO